Bất đẳng thức hình học

120
ĐI H¯C THÁI NGUYÊN TRƯ˝NG ĐI H¯C KHOA H¯C Hoàng Ngc Quang M¸T S¨ BT ĐNG THC HÌNH H¯C Chuyên Nghành: PHƯƠNG PHÁP TOÁN SƠ CP MÃ S¨: 60.46.40 LUN VĂN THC SĨ TOÁN H¯C Ngưi hưng d¤n khoa hc: TS. Nguy„n Văn Ngc Thái Nguyên - 2011 www.VNMATH.com

Transcript of Bất đẳng thức hình học

Page 1: Bất đẳng thức hình học

ĐẠI HỌC THÁI NGUYÊN

TRƯỜNG ĐẠI HỌC KHOA HỌC

Hoàng Ngọc Quang

MỘT SỐ BẤT ĐẲNG THỨC HÌNH HỌC

Chuyên Nghành: PHƯƠNG PHÁP TOÁN SƠ CẤP

MÃ SỐ: 60.46.40

LUẬN VĂN THẠC SĨ TOÁN HỌC

Người hướng dẫn khoa học: TS. Nguyễn Văn Ngọc

Thái Nguyên - 2011

www.VNMATH.com

Page 2: Bất đẳng thức hình học

Công trình được hoàn thành tạiTrường Đại học Khoa học - Đại học Thái Nguyên

Người hướng dẫn khoa học: TS. Nguyễn Văn Ngọc

Phản biện 1: . . . . . . . . . . . . . . . . . . . . . . . . . . . . . . . . . . . . . . . . . . . . . . . . . . . . . . .. . . . . . . . . . . . . . . . . . . . . . . . . . . . . . . . . . . . . . . . . . . . . . . . . . . . . . . . . . . . . . . . . . . .

Phản biện 2: . . . . . . . . . . . . . . . . . . . . . . . . . . . . . . . . . . . . . . . . . . . . . . . . . . . . . . .. . . . . . . . . . . . . . . . . . . . . . . . . . . . . . . . . . . . . . . . . . . . . . . . . . . . . . . . . . . . . . . . . . . .

Luận văn sẽ được bảo vệ trước hội đồng chấm luận văn họp tại:Trường Đại học Khoa học - Đại học Thái Nguyên

Ngày .... tháng .... năm 2011

Có thể tìm hiểu tạiThư viện Đại học Thái Nguyên

www.VNMATH.com

Page 3: Bất đẳng thức hình học

1

Mục lục

Mục lục . . . . . . . . . . . . . . . . . . . . . . . . . . . . . 1

Mở đầu . . . . . . . . . . . . . . . . . . . . . . . . . . . . . . 3

Chương 1. Các bất đẳng thức trong tam giác và tứ giác 6

1.1. Các bất đẳng thức đại số cơ bản . . . . . . . . . . . . . . 6

1.2. Các đẳng thức và bất đẳng thức cơ bản trong tam giác . 8

1.2.1. Các đẳng thức cơ bản trong tam giác . . . . . . . 8

1.2.2. Các bất đẳng thức cơ bản trong tam giác . . . . . 10

1.3. Bất đẳng thức trong tam giác . . . . . . . . . . . . . . . 11

1.3.1. Bất đẳng thức về độ dài các cạnh . . . . . . . . . 11

1.3.2. Bất đẳng thức về các đại lượng đặc biệt . . . . . 14

1.4. Các bất đẳng thức sinh ra từ các công thức hình học . . 17

1.5. Bất đẳng thức trong các tam giác đặc biệt . . . . . . . . 23

1.5.1. Các bất đẳng thức trong tam giác đều . . . . . . 23

1.5.2. Các bất đẳng thức trong tam giác vuông và tam

giác cân . . . . . . . . . . . . . . . . . . . . . . . 27

1.6. Các bất đẳng thức khác trong tam giác . . . . . . . . . . 29

1.7. Các bất đẳng thức trong tứ giác . . . . . . . . . . . . . . 40

1.7.1. Các bất đẳng thức cơ bản trong tứ giác . . . . . . 41

1.7.2. Các bất đẳng thức khác trong tứ giác . . . . . . . 45

Chương 2. Bất đẳng thức Ptolemy và các mở rộng 48

2.1. Định lí Ptolemy . . . . . . . . . . . . . . . . . . . . . . . 48

2.2. Bất đẳng thức Ptolemy . . . . . . . . . . . . . . . . . . . 53

2.3. Định lí Bretschneider . . . . . . . . . . . . . . . . . . . . 63

2.4. Định lí Casey . . . . . . . . . . . . . . . . . . . . . . . . 63

2.5. Mở rộng bất đẳng thức Ptolemy trong không gian . . . . 68

www.VNMATH.com

Page 4: Bất đẳng thức hình học

2

Chương 3. Bất đẳng thức Erdos-Mordell và các mở rộng 70

3.1. Bất đẳng thức Erdos-Mordell trong tam giác . . . . . . . 70

3.2. Bất đẳng thức Erdos-Mordell trong tam giác mở rộng . . 79

3.3. Mở rộng bất đẳng thức Erdos-Mordell trong tứ giác . . . 85

3.4. Mở rộng bất đẳng thức Erdos-Mordell trong đa giác . . . 87

3.5. Mở rộng bất đẳng thức Erdos-Mordell trong tứ diện . . . 90

Chương 4. Các bất đẳng thức có trọng 92

4.1. Bất đẳng thức dạng Hayashi và các hệ quả . . . . . . . . 92

4.1.1. Bất đẳng thức Hayashi . . . . . . . . . . . . . . . 92

4.1.2. Các hệ quả của bất đẳng thức hyashi . . . . . . . 94

4.1.3. Bài toán áp dụng . . . . . . . . . . . . . . . . . . 94

4.2. Bất đẳng thức Weizenbock suy rộng và các hệ quả . . . 96

4.2.1. Bất đẳng thức Weizenbock suy rộng . . . . . . . 96

4.2.2. Các hệ quả của bất đẳng thức Weizenbock suy rộng101

4.3. Bất đẳng thức Klamkin và các hệ quả . . . . . . . . . . 105

4.3.1. Bất đẳng thức Klamkin . . . . . . . . . . . . . . 105

4.3.2. Các hệ quả của bất đẳng thức Klamkin . . . . . . 106

4.4. Bất đẳng thức Jian Liu và các hệ quả . . . . . . . . . . 108

4.4.1. Bất đẳng thức Jian Liu . . . . . . . . . . . . . . . 108

4.4.2. Các hệ quả của bất đẳng thức Jian Liu . . . . . . 110

Kết luận . . . . . . . . . . . . . . . . . . . . . . . . . . . . . 116

Tài liệu tham khảo . . . . . . . . . . . . . . . . . . . . . . . 117

www.VNMATH.com

Page 5: Bất đẳng thức hình học

3

Mở đầu

Các bài toán về bất đẳng thức và cực trị hình học thuộc loại những

bài toán khó, làm cho học sinh phổ thông, nhất là phổ thông cơ sở kể cả

học sinh giỏi lúng túng khi gặp các bài toán loại này. Thực sự nó là một

phần rất quan trọng của hình học và những kiến thức về bất đẳng thức

trong hình học cũng làm phong phú hơn phạm vi ứng dụng của toán

học. So với các bất đẳng thức đại số, các bất đẳng thức hình học chưa

được quan tâm nhiều. Một trong những nguyên nhân gây khó giải quyết

vấn đề này là vì phương pháp tiếp cận không phải là các phương pháp

thông thường hay được áp dụng trong hình học và càng không phải là

phương pháp đại số thuần túy. Để giải một bài toán về bất đẳng thức

hình học cần thiết phải biết vận dụng các kiến thức hình học và đại số

một cách thích hợp và nhạy bén.

Luận văn này giới thiệu một số bất đẳng thức hình học từ cơ bản

đến nâng cao và mở rộng. Các bài toán về bất đẳng thức hình học được

trình bày trong luận văn này có thể tạm phân thành các nhóm sau:

I. Nhóm các bài toán mà trong lời giải đòi hỏi nhất thiết phải có

hình vẽ. Phương pháp giải các bài toán nhóm này chủ yếu là "phương

pháp hình học", như vẽ thêm đường phụ, sử dụng tính chất giữa đường

vuông góc và đường xiên, giữa đường thẳng và đường gấp khúc, quan

hệ giữa các cạnh, giữa cạnh và góc trong một tam giác, hay tứ giác v.v..

Bất đẳng thức và cực trị trong hình học phẳng thuộc nhóm này là nội

dung thường gặp trong các kì thi chọn học sinh giỏi toán hay thi vào các

trường chuyên.

II. Nhóm thứ hai gồm các bài toán mà khi giải chúng cần phải sử

dụng các hệ thức lượng đã biết, như các hệ thức lượng giác, hệ thức

đường trung tuyến, đường phân giác, công thức các bán kính, công thức

www.VNMATH.com

Page 6: Bất đẳng thức hình học

4

diện tích của tam giác v.v.. Các bài toán này đã được quan tâm nhiều

và chúng được trình bày khá phong phú trong các tài liệu [4,7], vì thế

luận văn này sẽ không đề cập nhiều đến các bất đẳng thức trong tam

giác có trong các tài liệu trên mặc dù chúng rất hay mà chỉ nêu ra một

số bất đẳng thức cơ bản nhất để tiện sử dụng sau này.

III. Nhóm thứ ba gồm các bài toán liên quan đến các bất đẳng thức

hình học nổi tiếng, đặc biệt là bất đẳng thức Ptolemy và bất đẳng thức

Erdos-Mordell và các bất đẳng thức có trọng như bất đẳng thức Hayshi,

bất đẳng thức Weizenbock, bất đẳng thức Klamkin v.v.. Các bất đẳng

thức này còn ít được giới thiệu bằng Tiếng Việt và thường gặp trong các

đề thi Olympic Quốc tế.

Bản luận văn "Một số bất đẳng thức hình học" gồm có mở đầu,

bốn chương nội dung, kết luận và tài liệu tham khảo.

Chương 1. Các bất đẳng thức trong tam giác và tứ giác.

Chương này trình bày một số bất đẳng thức thuộc nhóm I và nhóm II.

Chương 2. Bất đẳng thức Ptolemy và các mở rộng.

Chương này trình bày đẳng thức Ptolemy, bất đẳng thức Ptolemy và

các bài toán áp dụng. Các bài toán này chủ yếu được trích ra từ các đề

thi vô địch các nước, đề thi vô địch khu vực và đề thi IMO, một số là do

tác giả sáng tác. Ngoài ra, còn trình bày một số mở rộng bất đẳng thức

Ptolemy trong tứ giác và trong tứ diện.

Chương 3. Bất đẳng thức Erdos - Mordell và các mở rộng.

Chương này trình bày bất đẳng thức Edos-Mordell và các bài toán liên

quan. Ngoài ra, còn trình bày một số mở rộng bất đẳng thức này trong

tam giác, trong tứ giác và trong đa giác [11-13].

Chương 4. Các bất đẳng thức có trọng.

Chương này trình bày một số bất đẳng thức liên quan đến tổng khoảng

cách từ một hay nhiều điểm của mặt phẳng đến các đỉnh hoặc các cạnh

của tam giác với các tham số dương tùy ý được gọi là trọng số hay gọi

tắt là trọng. Đó là các bất đẳng thức Hyashi, Weizenbock, Klamkin, Jian

www.VNMATH.com

Page 7: Bất đẳng thức hình học

5

Liu, v.v.. Các bất đẳng thức này còn ít được giới thiệu bằng Tiếng Việt,

một số là kết quả nghiên cứu của các chuyên gia Quốc tế trong lĩnh vực

bất đẳng thức hình học [9,13-14].

Luận văn này được hoàn thành tại trường Đại học Khoa học - Đại

học Thái Nguyên với sự hướng dẫn của TS. Nguyễn Văn Ngọc. Tác giả

xin được bày tỏ lòng biết ơn sâu sắc đối với sự quan tâm hướng dẫn của

Thầy, tới các thầy cô trong Ban Giám hiệu, Phòng Đào tạo và Khoa

Toán-Tin Trường Đại học Khoa học. Đồng thời tác giả xin cảm ơn tới

Sở GD - ĐT tỉnh Yên Bái, Ban Giám đốc, các đồng nghiệp Trung tâm

GDTX - HNDN Hồ Tùng Mậu huyện Lục Yên đã tạo điều kiện cho tác

giả học tập và hoàn thành kế hoạch học tập.

Thái Nguyên, ngày 20 tháng 6 năm 2011.

Tác giả

Hoàng Ngọc Quang

www.VNMATH.com

Page 8: Bất đẳng thức hình học

6

Chương 1Các bất đẳng thức trong tam giácvà tứ giác

Chương này trình bày các bất đẳng thức trong tam giác và tứ giác

từ cơ bản đến nâng cao. Nội dung chủ yếu được hình thành từ các tài

liệu [1-7], [10], [12] và [15].

Kí hiệu ∆ABC là tam giác ABC với các đỉnh là A,B,C. Để thuận

tiện, độ lớn của các góc ứng với các đỉnh A,B,C cũng được kí hiệu tương

ứng là A,B,C.

Độ dài các cạnh của tam giác: BC = a, CA = b, AB = c.

Nửa chu vi của tam giác: p =a+ b+ c

2.

Đường cao với các cạnh: ha, hb, hc.

Đường trung tuyến với các cạnh: ma,mb,mc.

Đường phân giác với các cạnh: la, lb, lc.

Bán kính đường tròn ngoại tiếp và đường tròn nội tiếp: R và r.

Bán kính đường tròn bàng tiếp các cạnh: ra, rb, rc.

Diện tích tam giác ABC: S, SABC hay [ABC].

Để giải được các bài toán bất đẳng thức hình học, trước hết ta cần

trang bị những kiến thức cơ sở đó là các bất đẳng thức đại số cơ bản và

các đẳng thức, bất đẳng thức đơn giản trong tam giác.

1.1. Các bất đẳng thức đại số cơ bản

Định lý 1.1. (Bất đẳng thức AM-GM) Giả sử a1, a2, · · · , an là các số

thực không âm. Khi đó

a1 + a2 + · · ·+ ann

≥ n√a1a2...an. (1.1)

Đẳng thức xảy ra khi và chỉ khi a1 = a2 = · · · = an.

www.VNMATH.com

Page 9: Bất đẳng thức hình học

7

Hệ quả 1.1. Với mọi bộ số dương a1, a2, · · · , an ta có

n√a1a2...an ≥

n1a1

+ 1a2

+ · · ·+ 1an

. (1.2)

Đẳng thức xảy ra khi và chỉ khi a1 = a2 = · · · = an.

Hệ quả 1.2. Với mọi bộ số dương a1, a2, · · · , an ta có

1

a1+

1

a2+ · · ·+ 1

an≥ n2

a1 + a2 + · · ·+ an. (1.3)

Đẳng thức xảy ra khi và chỉ khi a1 = a2 = ... = an.

Hệ quả 1.3. Với mọi bộ số không âm a1, a2, · · · , an và m = 1, 2, · · · tacó

am1 + am2 + · · ·+ amnn

≥(a1 + a2 + · · ·+ an

n

)m. (1.4)

Đẳng thức xảy ra khi và chỉ khi a1 = a2 = · · · = an.

Định lý 1.2. (Bất đẳng thức Cauchy - Schwarz) Cho hai dãy số thựca1, a2, · · · , an và b1, b2, · · · , bn. Khi đó

(a1b1 + a2b2 + · · ·+ anbn)2 ≤

(a21 + a22 + · · ·+ a2n

) (b21 + b22 + · · ·+ b2n

). (1.5)

Đẳng thức xảy ra khi và chỉ khi a1b1 = a2b2

= · · · = anbn.

Định lý 1.3. (Bất đẳng thức Jensen) Cho f(x) là hàm số liên tục và có

đạo hàm cấp hai trên I (a, b) và n điểm x1, x2, · · · , xn tùy ý trên đoạn

I (a, b). Khi đó

i, Nếu f ′′(x) > 0 với mọi x ∈ I (a, b) thì

f(x1) + f(x2) + · · ·+ f(xn) ≥ nf

(x1 + x2 + · · ·+ xn

n

).

ii, Nếu f ′′(x) < 0 với mọi x ∈ I (a, b) thì

f(x1) + f(x2) + · · ·+ f(xn) ≤ nf

(x1 + x2 + · · ·+ xn

n

).

Ở đây I (a, b) nhằm ngầm định là một trong bốn tập hợp (a, b) , [a, b) ,

(a, b] , [a, b].

www.VNMATH.com

Page 10: Bất đẳng thức hình học

8

Định lý 1.4. (Bất đẳng thức Chebyshev) Cho hai dãy số thực đơn điệucùng chiều a1, a2, · · · , an và b1, b2, · · · , bn. Khi đó ta có

a1b1 + a2b2 · · ·+ anbn ≥1

n(a1 + a2 + · · ·+ an) (b1 + b2 + · · ·+ bn) . (1.6)

Nếu hai dãy số thực a1, a2, · · · , an và b1, b2, · · · , bn đơn điệu ngược chiều

thì bất đẳng thức trên đổi chiều.

Định lý 1.5. (Bất đẳng thức Nesbitt) Cho a, b, c là các số thực dương.

Bất đẳng thức sau luôn đúng

a

b+ c+

b

c+ a+

c

a+ b≥ 3

2. (1.7)

Đẳng thức xảy ra khi và chỉ khi a = b = c.

1.2. Các đẳng thức và bất đẳng thức cơ bản trong tam giác

1.2.1. Các đẳng thức cơ bản trong tam giác

Định lý 1.6. (Định lý hàm số sin) Trong tam giác ABC ta có

a

sinA=

b

sinB=

c

sinC= 2R.

Định lý 1.7. (Định lý hàm số cosin) Trong tam giác ABC ta có

a2 = b2 + c2 − 2bc cosA, b2 = c2 + a2 − 2ca cosB, c2 = a2 + b2 − 2ab cosC.

Định lý 1.8. (Các công thức về diện tích) Diện tích tam giác ABC

được tính theo một trong các công thức sau

S =1

2aha =

1

2bhb =

1

2chc (1.8)

=1

2bc sinA =

1

2ca sinB =

1

2ab sinC (1.9)

= pr (1.10)

=abc

4R(1.11)

= (p− a)ra = (p− b)rb = (p− c)rc (1.12)

=√p (p− a) (p− b) (p− c). (1.13)

Công thức (1.13) được gọi là công thức Hê-rông.

www.VNMATH.com

Page 11: Bất đẳng thức hình học

9

Định lý 1.9. (Định lý đường phân giác) Trong một tam giác, đường

phân giác của một góc chia cạnh đối diện thành hai đoạn thẳng tỉ lệ với

hai cạnh kề hai đoạn ấy .

Định lý 1.10. (Công thức đường phân giác) Trong tam giác ABC ta

la =2bc

b+ ccos

A

2, lb =

2ca

c+ acos

B

2, lc =

2ab

a+ bcos

C

2.

Định lý 1.11. (Định lý đường trung tuyến) Trong một tam giác, ba

đường trung tuyến gặp nhau tại một điểm được gọi là trọng tâm của

tam giác. Trên mỗi đường trung tuyến, khoảng cách từ trọng tâm đến

đỉnh bằng hai lần khoảng cách trọng tâm đến chân đường trung tuyến.

Định lý 1.12. (Công thức đường trung tuyến) Trong tam giác ABC ta

m2a =

b2 + c2

2− a2

4, m2

b =c2 + a2

2− b2

4, m2

c =a2 + b2

2− c2

4.

Định lý 1.13. (Công thức bán kính đường tròn nội tiếp) Trong tam

giác ABC ta có

r = (p− a) tanA

2= (p− b) tan

B

2= (p− c) tan

C

2.

Định lý 1.14. (Công thức bán kính đường tròn bàng tiếp) Trong tam

giác ABC ta có

ra = p tanA

2, rb = p tan

B

2, rc = p tan

C

2.

Định lý 1.15. (Các hệ thức lượng giác cơ bản) Với mọi tam giác ABC

ta luôn có các hệ thức sau

sinA+ sinB + sinC = 4 cosA

2cos

B

2cos

C

2, (1.14)

sin 2A+ sin 2B + sin 2C = 4 sinA sinB sinC, (1.15)

cosA+ cosB + cosC = 1 + 4 sinA

2sin

B

2sin

C

2, (1.16)

cos 2A+ cos 2B + cos 2C = −1− 4 cosA cosB cosC, (1.17)

www.VNMATH.com

Page 12: Bất đẳng thức hình học

10

sin2A+ sin2B + sin2C = 2 (1 + sinA sinB sinC) , (1.18)

cos2A+ cos2B + cos2C = 1− 2 cosA cosB cosC, (1.19)

tanA+ tanB + tanC = tanA tanB tanC, (1.20)

cotA

2+ cot

B

2+ cot

C

2= cot

A

2cot

B

2cot

C

2, (1.21)

tanA

2tan

B

2+ tan

B

2tan

C

2+ tan

C

2tan

A

2= 1, (1.22)

cotA cotB + cotB cotC + cotC cotA = 1. (1.23)

Riêng với hệ thức (1.20) thì tam giác ABC cần giả thiết không vuông.

1.2.2. Các bất đẳng thức cơ bản trong tam giác

Định lý 1.16. (Bất đẳng thức tam giác) Trong tam giác ABC ta có

|b− c| < a < b+ c, |c− a| < b < c+ a, |a− b| < c < a+ b.

Định lý 1.17. (Các bất đẳng thức lượng giác cơ bản) Với mọi tam giác

ABC ta luôn có các bất đẳng thức sau

sinA+ sinB + sinC ≤ 3√

3

2, (1.24)

cosA+ cosB + cosC ≤ 3

2, (1.25)

cosA

2+ cos

B

2+ cos

C

2≤ 3√

3

2, (1.26)

sinA

2+ sin

B

2+ sin

C

2≤ 3

2, (1.27)

sinA

2sin

B

2sin

C

2≤ 1

8, (1.28)

cosA cosB cosC ≤ 1

8, (1.29)

sin2A+ sin2B + sin2C ≤ 9

4, (1.30)

tanA

2+ tan

B

2+ tan

C

2≥√

3, (1.31)

tanA+ tanB + tanC ≥ 3√

3, (1.32)

cotA+ cotB + cotC ≥√

3. (1.33)

www.VNMATH.com

Page 13: Bất đẳng thức hình học

11

Riêng với bất đẳng thức (1.32) thì tam giác ABC cần giả thiết không

vuông. Đẳng thức xảy ra trong các bất đẳng thức trên khi và chỉ khi

ABC là tam giác đều.

1.3. Bất đẳng thức trong tam giác

Tam giác là hình đơn giản nhất trong các đa giác, mỗi đa giác bất kì

đều có thể chia thành các tam giác và sử dụng tính chất của nó. Vì vậy,

nghiên cứu các bất đẳng thức trong tam giác sẽ hữu ích trong việc giải

quyết các bất đẳng thức trong đa giác. Trước hết, chúng ta nghiên cứu

các bất đẳng thức cơ bản sau đây:

1.3.1. Bất đẳng thức về độ dài các cạnh

Định lý 1.18. Cho hai đường tròn có bán kính lần lượt là R và R′

(R ≥ R′), khoảng cách giữa tâm của chúng bằng d. Điều kiện cần và đủ

để hai đường tròn đó cắt nhau là R−R′ ≤ d ≤ R +R′.

Chứng minh.

Hình 1.1 Hai đường tròn không cắt nhau.

Rõ ràng nếu hai đường tròn ở ngoài nhau (hình 1.1 A) thì ta có

R + R′ < d. Nếu hai đường tròn chứa nhau (hình 1.1 B) thì ta cũng có

ngay d < R−R′. Nếu hai đường tròn cắt nhau tại một điểm M thì theo

bất đẳng thức về ba cạnh của tam giác OO′M , với O và O′ lần lượt là

tâm của đường tròn bán kính R và R′, ta có R−R′ ≤ d ≤ R +R′.

Đảo lại, nếu R − R′ ≤ d ≤ R +R′ thì hai đường tròn đã cho không

thể ngoài nhau hoặc chứa nhau được (nếu không phải có R + R′ < d

hoặc d < R−R′). Do đó chúng chỉ có thể cắt nhau.

www.VNMATH.com

Page 14: Bất đẳng thức hình học

12

Định lý 1.19. Các số dương a, b, c là độ dài 3 cạnh của một tam giác

khi và chỉ khi a+ b > c, b+ c > a, c+ a > b.

Chứng minh. Nếu a, b, c là độ dài 3 cạnh của tam giác thì theo bất đẳng

thức về 3 cạnh của tam giác ta có a+ b > c, b+ c > a, c+ a > b.

Ngược lại, nếu có a, b và c là 3 số thực dương thỏa mãn a+b > c, b+c >

a, c + a > b, thì ta có thể chọn hai điểm A và B trên mặt phẳng cách

nhau một khoảng c. Lấy A và B làm tâm dựng hai đường tròn bán kính

tương ứng là a và b. Từ các bất đẳng thức a+ b > c, b+ c > a, c+ a > b

ta có |a− b| < c < a + b. Theo định lý 1.18 thì hai đường tròn tâm A

và B phải cắt nhau tại một điểm C. Vậy a, b, c là độ dài các cạnh của

tam giác ABC theo cách dựng trên.

Định lý 1.20. Cho trước tam giác ABC và một điểm M ở trong tam

giác. Khi đó ta có MB +MC < AB + AC.

Chứng minh.

Hình 1.2

Kéo dài BM về phía M cắt

cạnh AC tại điểm N . Theo định

lý 1.19 ta có

MB +MC < MB +MN +NC

=BN +NC < AB + AN +NC

=AB + AC.

Bài toán 1.1. Cho M là một điểm nằm trong tam giác ABC. Chứng

minh rằng

p < MA+MB +MC < 2p.

Trong đó p là nửa chu vi của tam giác ABC.

Giải. Áp dụng định lý 1.19 cho các tam giácMAB,MBC vàMCA ta có

AB < MA+MB, BC < MB+MC, CA < MC+MA. Cộng theo vế ba

bất đẳng thức trên rồi chia cả hai vế cho 2 ta được p < MA+MB+MC.

Mặt khác, theo định lý 1.20 ta cóMA+MB < CA+CB, MB+MC <

AB+AC, MC +MA < BC +BA. Cộng theo vế ba bất đẳng thức trên

và đem chia cả hai vế cho 2 ta được AM +BM + CM < 2p.

www.VNMATH.com

Page 15: Bất đẳng thức hình học

13

Định lý 1.21. Trong một tam giác ứng với góc lớn hơn là cạnh dài hơn

và ngược lại.

Chứng minh. Xét tam giác ABC. Ta chứng minh nếu ABC > ACB thì

AC > AB và ngược lại.

Hình 1.3

Thật vậy, trong góc ABC ta kẻ tia Bx

tạo với cạnh BC góc bằng góc ACB. Do

ABC > ACB, nên Bx cắt cạnh AC tại

điểm D và tạo thành tam giác cân DBC,

do đó DB = DC. Mặt khác, trong tam

giác ABD ta có AD + DB > AB. Do đó

AC = AD +DC = AD +DB > AB.

Phần ngược lại của định lý là hiển nhiên. Vì nếu ABC < ACB thì

ta phải có AC < AB là điều vô lí.

Bài toán 1.2. Chứng minh rằng đường vuông góc AH hạ từ điểm A

xuống đường thẳng d cho trước luôn nhỏ hơn đường xiên AB.

Giải. Tam giác AHB là tam giác vuông tại H, do đó AHB = 900 >

ABH. Theo định lý trên, ta có AB > AH.

Sự tương ứng giữa độ lớn cạnh và góc còn đúng cho cạnh của hai tam

giác khác nhau. Dùng định lý 1.21 ta dễ dàng chứng minh kết quả sau.

Định lý 1.22. Cho trước hai tam giác ABC và A′B′C ′ có hai cặp cạnh

bằng nhau AB = A′B′ và AC = A′C ′. Ta có bất đẳng thức BAC >

B′A′C ′ khi và chỉ khi BC > B′C ′.

Chứng minh.

Hình 1.4

Trước hết, giả sử rằng BAC >

B′A′C ′, ta sẽ chứng minh BC > B′C ′.

Không mất tính tổng quát giả sử

AB ≥ AC. Ta đem hình tam giác

ABC đặt chồng lên hình tam giác

A′B′C ′ sao cho A ≡ A′, C ≡ C ′

và đỉnh B, B′ nằm cùng phía so với

đường thẳng đi qua AC.

www.VNMATH.com

Page 16: Bất đẳng thức hình học

14

Do AB = A′B′, nên ta có ABB′ = AB′B. Vì CBB′ < ABB′ và

CB′B > AB′B, nên ta có CBB′ < CB′B. Theo định lý 1.21, ta có

CB > CB′, hay là CB > C ′B′.

Nếu như BAC = B′A′C ′ thì ta cũng dễ dàng thấy rằng BC = B′C ′,

do ∆ABC và ∆A′B′C ′ (c.g.c). Vậy ta có BAC > B′A′C ′ khi và chỉ khi

BC > B′C ′.

Bài toán 1.3. Cho tam giác ABC và AM là trung tuyến. Chứng minh

rằng BAC ≥ 900 khi và chỉ khi AM ≤ 12BC.

Giải. Gọi A′ là điểm đối xứng với A qua trung điểm M của cạnh BC.

Tứ giác ABA′C là tứ giác có hai đường chéo cắt nhau tại trung điểm

của mỗi đường nên ABA′C là hình bình hành.

Hình 1.5

Xét hai tam giác ABA′ và ABC có

cạnh AB là cạnh chung và có cặp cạnh

A′B và AC bằng nhau. Theo định lý

1.22, ta có BC ≥ AA′ khi và chỉ khi

BAC ≥ ABA′. Do BAC + ABA′ =

1800 , cho nên BAC ≥ ABA′ khi và

chỉ khi BAC ≥ 900. Tóm lại, AM =12AA

′ ≤ 12BC khi và chỉ khi BAC ≥

900.

Định lý 1.23. Trong những đường xiên nối một điểm M cho trước với

điểm N trên một đường thẳng d cho trước, đường xiên nào có hình chiếu

dài hơn thì dài hơn.

1.3.2. Bất đẳng thức về các đại lượng đặc biệt

Trong một tam giác, mối quan hệ giữa các cạnh dẫn đến mối quan hệ

với các đại lượng đặc biệt. Với đường cao ta dễ thấy là đường cao tương

ứng với cạnh lớn hơn thì ngắn hơn. Đối với đường trung tuyến và đường

phân giác ta cũng sẽ chứng minh rằng ứng với cạnh dài hơn là đường

trung tuyến và đường phân giác ngắn hơn.

Định lý 1.24. Trong tam giác ABC ứng với cạnh dài hơn là đường cao,

đường trung tuyến và đường phân giác ngắn hơn.

www.VNMATH.com

Page 17: Bất đẳng thức hình học

15

Chứng minh. Giả sử c < b, ta sẽ chứng minh rằng hb < hc,mb < mc và

lb < lc.

Hình 1.6

Vì c < b nên suy ra hb = S2b <

S2c = hc.

Để chứng minhmb < mc, ta gọiM,N và

P là trung điểm của các cạnh AB,AC và

BC, tương ứng (hình 1.6). Áp dụng định

lý 1.22 cho ∆PAB và ∆PAC là hai tam

giác có hai cặp cạnh bằng nhau (AP chung

và BP = CP ), ta có APB < APC. Gọi G

là trọng tâm của tam giác ABC. Xét hai

tam giác GPB và GPC là hai tam giác có hai cặp cạnh bằng nhau

(GP chung và PB = PC). Do có APB < APC, nên BG < CG. Vậy

mb =3

2BG <

3

2CG = mc.

Hình 1.7

Gọi phân giác của góc B là BL và phân

giác xuất phát từ C là CK. Theo định lý

đường phân giác ta có LCLA = a

c ⇒ CL =aba+c . Tương tự KB

KA = ab ⇒ BK = ac

a+b .

Do c < b, nên BK < CL. Dựng hình

bình hành BKCT (hình 1.7), ta có BTC =

BKC = A+ C2 và ta có BTC < BLC.

Mặt khác, vì TC = BK, và BK < CL

nên TC < CL. Trong tam giác TLC, ứng

với cạnh lớn hơn là góc lớn hơn theo định

lí 1.21, cho nên CLT < CTL. Từ các bất đẳng thức BLC > BTC và

CLT < CTL, ta có BLT < BTL. Theo định lý 1.21 ta có BT > BL

mà CK = BT suy ra CK > BL.

Định lý 1.25. Trong tam giác ABC ta luôn có

ma ≥ la ≥ ha.

Chứng minh. Gọi H là chân đường cao, L là chân đường phân giác vàM

là chân đường trung tuyến xuất phát từ đỉnh A. Ta chứng minh rằng L

nằm trên đoạn thẳng nối HM , và áp dụng định lý 1.23 để có bất đẳng

thức cần chứng minh.

www.VNMATH.com

Page 18: Bất đẳng thức hình học

16

Hình 1.8

Định lí hiển nhiên đúng cho trường hợp

tam giác ABC cân tại đỉnh A. Để tiện

chứng minh trong trường hợp tam giác

không cân tại A, không mất tính tổng quát

ta giả sử AB < AC. Gọi A′ đối xứng với A

qua M , ta có BACA′ là hình bình hành.

Trong tam giác AA′C ta có AC >

A′C = AB và do đó theo định lý 1.21 ta

có BAM = MA′C > CAM và đó điểm L

nằm trong góc BAM . Mặt khác, do BAH phụ với góc B và CAH phụ

với góc C, cho nên BAH < CAH. Do đó L phải nằm trong góc CAH.

Tóm lại, điểm L nằm giữa điểm H và điểm M và ta có HM > HL.

Theo định lý 1.23, ta có AH < AL < AM .

Định lý 1.26. Đường trung tuyến AM của tam giác ABC nhỏ hơn nửa

tổng các cạnh AB và AC cùng xuất phát từ một đỉnh A.

Chứng minh. Gọi A′ là điểm đối xứng với điểm A qua điểm M , ta có

ABA′C là hình bình hành. Do đó AM =1

2AA′ <

1

2(A′C + AC) =

1

2(AB + AC).

Bài toán 1.4. Chứng minh rằng nếu M là điểm nằm trên đường phân

giác ngoài của góc C của tam giác ABC (M khác C) thì MA+MB >

CA+ CB.

Giải.

Hình 1.9

Giả sử A′ là điểm đối xứng

với điểm A qua đường phân giác

ngoài của góc C. Khi đó các điểm

A′, C,B thẳng hàng và MA′ =

MA. Do đó MA+MB = MA′+

MB > A′B = CA′ + CB =

CA+ CB.

www.VNMATH.com

Page 19: Bất đẳng thức hình học

17

1.4. Các bất đẳng thức sinh ra từ các công thức hình học

Định lý 1.27. (Công thức Euler) Gọi R và r lần lượt là bán kính của

đường tròn ngoại tiếp và đường tròn nội tiếp tam giác ABC, d là khoảng

cách giữa tâm hai đường tròn đó. Ta có

d2 = R2 − 2Rr. (1.34)

Chứng minh.

Hình 1.10

Gọi O, I lần lượt là tâm đường

tròn ngoại tiếp, nội tiếp ∆ABC.

Biết rằng đường tròn ngoại tiếp

tam giác BCI có tâm D là trung

điểm của cung_

BC. Gọi M là

trung điểm của BC và Q là hình

chiếu của I trên OD. Khi đó

OB2 −OI2 =OB2 −DB2 +DI2 −OI2

=OM 2 −MD2 +DQ2 −QO2

= (MO +DM) (MO −DM) + (DQ+QO) (DQ−QO)

=DO (MO −DM +DQ+OQ) = R (2MQ) = 2Rr.

Vậy OI2 = R2 − 2Rr, nghĩa là d2 = R2 − 2Rr.

Hệ quả 1.4. (Bất đẳng thức Euler) Kí hiệu R, r lần lượt là bán kính

đường tròn ngoại tiếp và bán kính đường tròn nội tiếp tam giác ABC.

Khi đó

R ≥ 2r. (1.35)

Đẳng thức xảy ra khi và khi tam giác ABC đều.

Bài toán 1.5. Cho tam giác ABC với R là bán kính đường tròn ngoại

tiếp, r là bán kính đường tròn nội tiếp và p là nửa chu vi tam giác ABC.

Chứng minh rằng r ≤ p

3√

3≤ R

2.

www.VNMATH.com

Page 20: Bất đẳng thức hình học

18

Giải. Ta có [ABC] = abc4R = pr, suy ra 2p = a+b+c ≥ 3 3

√abc = 3 3

√4Rrp.

Do đó 8p3 ≥ 27(4Rrp) ≥ 27(8r2p), vì R ≥ 2r. Vậy p ≥ 3√

3r.

Bất đẳng thức thứ hai, p

3√3≤ R

2 tương đương với a + b + c ≤ 3√

3R.

Sử dụng định lí hàm số sin, bất đẳng thức này tương đương với sinA+

sinB + sinC ≤ 3√3

2 . Bất đẳng thức này đúng vì hàm số f(x) = sinx là

hàm lồi trên (0, π), do đó sinA+sinB+sinC3 ≤ sin

(A+B+C

3

)= sin 600 =

√32 .

Định lý 1.28. (Công thức Leibniz) Cho tam giác ABC với độ dài các

cạnh là a, b, c. Gọi G là trọng tâm và (O,R) là đường tròn ngoại tiếp

tam giác. Khi đó

OG2 = R2 − 1

9

(a2 + b2 + c2

). (1.36)

Chứng minh.

Hình 1.11

Để chứng minh bài toán này ta sử dụng

định lí Stewart "Nếu L là điểm nằm trên

cạnh BC của ∆ABC và nếu AL = l, BL =

m,LC = n, thì a(l2 +mn) = b2m+ c2n".

Áp dụng định lí Stewart cho ∆OAA′,

trong đó A′ là trung điểm của BC, ta

được AA′(OG2 + AG.GA′) = A′O2.AG +

AO2.GA′. Vì AO = R,AG = 23AA

′, GA′ =13AA

′ nên OG2 + 29A′A2 = 2

3A′O2 + 1

3R2.

Mặt khác, vì A′A2 = 2(b2+c2)−a24 và A′O2 = R2 − a2

4 , ta được

OG2 =

(R2 − a2

4

)2

3+

1

3R2 − 2

9

(2(b2 + c2)− a2

4

)=R2 − a2

6− 2(b2 + c2)− a2

18= R2 − a2 + b2 + c2

9.

Hệ quả 1.5. (Bất đẳng thức Leibniz) Cho tam giác ABC với độ dài

các cạnh là a, b, c. (O,R) là đường tròn ngoại tiếp tam giác. Ta có bất

đẳng thức sau

9R2 ≥ a2 + b2 + c2. (1.37)

Đẳng thức xảy ra khi và chỉ khi O là trọng tâm của tam giác ABC.

www.VNMATH.com

Page 21: Bất đẳng thức hình học

19

Bài toán 1.6. Cho tam giác ABC với độ dài các cạnh là a, b, c. Chứng

minh rằng 4√

3 [ABC] ≤ 9abca+b+c .

Giải. Sử dụng bất đẳng thức Leibniz với lưu ý 4R. [ABC] = abc ta có

9R2 ≥ a2 + b2 + c2 ⇔ a2b2c2

16[ABC]2 ≥ a2+b2+c2

9 ⇔ 4 [ABC] ≤ 3abc√a2+b2+c2

. Mặt

khác, Bất đẳng thức Cauchy-Schwarz cho a+ b+ c ≤√

3√a2 + b2 + c2.

Do đó 4√

3 [ABC] ≤ 9abca+b+c .

Bài toán 1.7. Giả sử đường tròn nội tiếp tam giác ABC tiếp xúc các

cạnh AB,BC,CA tại D,E, F , tương ứng.Kí hiệu p là nửa chu vi của

tam giác ABC. Chứng minh rằng EF 2 + FD2 +DE2 ≤ p2

3 .

Giải. Thấy rằng đường tròn nội tiếp tam giác ABC là đường tròn ngoại

tiếp tam giác DEF . Áp dụng bất đẳng thức Leibniz cho tam giác DEF ,

ta được EF 2 + FD2 + DE2 ≤ 9r2. Mặt khác, theo bài toán 1.5 ta có

p2 ≥ 27r2. Do đó EF 2 + FD2 +DE2 ≤ p2

3 .

Định lý 1.29. (Định lí Euler) Cho tam giác ABC nội tiếp trong đường

tròn tâm O, bán kính R. M là một điểm bất kì nằm trong mặt phẳng

tam giác. Gọi X, Y, Z lần lượt là hình chiếu vuông góc của M lên các

cạnh BC,CA,AB. Khi đó diện tích của tam giác XY Z được tính theo

diện tích tam giác ABC và khoảng cách MO bởi công thức sau

[XY Z] =1

4

∣∣∣∣1− MO2

R2

∣∣∣∣ [ABC] . (1.38)

Chứng minh.

Hình 1.12

Kéo dài AM,BM,CM cắt đường tròn

ngoại tiếp tại các điểm X ′, Y ′, Z ′ tương

ứng. Ta có ZXM = MBZ (tứ giác

BZMX nội tiếp), MBZ = ABY ′ (B,Z,A

thẳng hàng và B,M, Y ′ thẳng hàng),

ABY ′ = AX ′Y ′ (cùng chắn cung_

AY ′).

Từ đó suy ra ZXM = AX ′Y ′. Tương tự

Y XM = AX ′Z ′. Từ đó suy ra ZXY =

Z ′X ′Y ′. Ta sẽ kí hiệu hai góc này là X và

www.VNMATH.com

Page 22: Bất đẳng thức hình học

20

X ′. Ta có

[XY Z] =1

2XY.XZ. sinX

=1

2MC. sinC.MB sinB. sinX (định lí hàm số sin)

=1

2MB.MY ′.

MC

MY ′sinB. sinC. sinX

=1

2

∣∣MO2 −R2∣∣ BCZ ′Y ′

. sinB sinC. sinX(phương tích, ∆MBC ∼ ∆MZ ′Y ′)

=1

2

∣∣MO2 −R2∣∣ .BC. sinC. sinB.sinX ′

Y ′Z ′

=1

8

∣∣∣∣1− MO2

R2

∣∣∣∣AC.BC. sinC (vì sinB =AC

2R,sinX ′

Y ′Z ′=

1

2R)

=1

4

∣∣∣∣1− MO2

R2

∣∣∣∣ [ABC] .

Chú ý 1.1.

1) Tam giác XY Z nêu trong định lí được gọi là tam giác Pedal.

2) NếuM nằm trên đường tròn ngoại tiếp tam giác ABC thì [XY Z] =

0. Điều đó có nghĩa là tam giác XY Z suy biến thành đường thẳng, đó

chính là đường thẳng Euler.

3) NếuM ≡ I (I là tâm đường tròn nội tiếp tam giác ABC) thì XY Z

là tam giác nội tiếp đường tròn tâm I, bán kính r có các góc X, Y, Z

tương ứng bằng π2 −

A2 ,

π2 −

B2 ,

π2 −

C2 . Bằng các phép biến đổi sơ cấp từ

công thức (1.38) sẽ suy ra công thức Euler OI2 = R2 − 2Rr.

Hệ quả 1.6. Cho tam giác ABC và một điểm M bất kì nằm trong mặt

phẳng tam giác. Gọi X, Y, Z lần lượt là hình chiếu vuông góc của M lên

các cạnh BC,CA,AB. Khi đó

[XY Z] ≤ 1

4[ABC] . (1.39)

Đẳng thức xảy ra khi và chỉ khi M là tâm của đường tròn ngoại tiếp

tam giác ABC.

www.VNMATH.com

Page 23: Bất đẳng thức hình học

21

Bài toán 1.8. Cho tứ giác lồi ABCD nội tiếp trong đường tròn tâm O

(với O nằm bên trong tứ giác). Gọi MNPQ là tứ giác mà các đỉnh lần

lượt là hình chiếu của giao điểm 2 đường chéo của tứ giác ABCD đến

các cạnh AB,BC,CD,DA. Chứng minh rằng

[MNPQ] ≤ 1

2[ABCD] .

Giải. Gọi K là giao điểm 2 đường chéo AC và BD của tứ giác ABCD.

Hình 1.13

Dễ thấy KMN là tam giác Pedal dựng

từ điểm K của tam giác ABC. Do đó

áp dụng hệ quả 1.6 ta được [KMN ] ≤14 [ABC]. Làm tương tự cho các tam giác

KNP,KPQ,KQM và cộng các kết quả lại

[KMN ] + [KNP ] + [KPQ] + [KQM ]

≤1

4([ABC] + [BCD] + [CDA] + [DAB]) .

Suy ra [MNPQ] ≤ 12 [ABCD].

Đẳng thức xảy ra khi và chỉ khi K là tâm của đường tròn ngoại tiếp tứ

giác ABCD.

Bài toán 1.9. (Balkan, 1999) Cho ABC là một tam giác nhọn và

L,M,N là các chân đường cao hạ từ trọng tâm G của ∆ABC tới các

cạnh BC,CA,AB, tương ứng. Chứng minh rằng

4

27<

[LMN ]

[ABC]≤ 1

4.

Giải. Ta có tam giác LMN là tam giác Pedal dựng từ trọng tâm

G của tam giác ABC. Áp dụng định lí 1.12, ta có [LMN ] =14

∣∣∣1− OG2

R2

∣∣∣ [ABC] = R2−OG2

4R2 [ABC] (vì G nằm trong tam giác ABC).

+ Dễ thấy [LMN ] ≤ 14 [ABC]. Đẳng đẳng thức xảy ra khi và chỉ khi

G ≡ O hay khi và chỉ khi tam giác ABC đều.

+ Ta chứng minh bất đẳng thức còn lại. Thật vậy, để ý rằng OG = 13OH.

Vì tam giác nhọn, H nằm trong tam giác và OH ≤ R nên

[LMN ]

[ABC]=R2 − 1

9OH2

4R2≥R2 − 1

9R2

4R2=

2

9>

4

27.

www.VNMATH.com

Page 24: Bất đẳng thức hình học

22

Định lý 1.30. (Công thức hình bình hành) Cho tứ giác ABCD, gọi x

là khoảng cách giữa trung điểm của hai đường chéo AC và BD. Ta có

AB2 +BC2 + CD2 +DA2 = AC2 +BD2 + 4x2. (1.40)

Chứng minh. Gọi M,N lần lượt là trung điểm của AC và BD. Áp dụng

công thức đường trung tuyến ta có

x2 =NA2 +NC2

2− AC2

4=

AB2+DA2

2 − BD2

4 + BC2+CD2

2 − BD2

4

2− AC2

4

hay AB2 +BC2 + CD2 +DA2 = AC2 +BD2 + 4x2.

Hệ quả 1.7. (Bất đẳng thức hình bình hành) Cho tứ giác ABCD. Ta

AB2 +BC2 + CD2 +DA2 ≥ AC2 +BD2. (1.41)

Đẳng thức xảy ra khi và chỉ khi tứ giác ABCD là hình bình hành.

Bài toán 1.10. (Địa trung Hải, 2000) Cho P,Q,R, S là trung điểm của

các cạnh BC,CD,DA,AB, tương ứng, của tứ giác lồi ABCD. Chứng

minh rằng

4(AP 2 +BQ2 + CR2 +DS2

)≤ 5

(AB2 +BC2 + CD2 +DA2

).

Giải. Ta biết công thức đường trung tuyến XM của tam giác XY Zlà XM2 = 1

2XY2 + 1

2XZ2 − 1

4Y Z2. Ta thay bộ (X, Y, Z,M) bằng

(A,B,C, P ), (B,C,D,Q), (C,D,A,R), (D,A,B, S) vào trong công thứcnày và cộng 4 công thức lại với nhau để thu được công thức thứ 5. Nhâncả hai vế của công thức thứ 5 với 4, ta được

4(AP 2 +BQ2 + CR2 +DS2

)= AB2 +BC2 + CD2 +DA2 + 4

(AC2 +BC2

).

Do đó ta chỉ cần chứng minh bất đẳng thức

AC2 +BC2 ≤ AB2 +BC2 + CD2 +DA2.

Đây là bất đẳng thức hình bình hành. Dấu "=" xảy ra khi và chỉ khi tứ

giác ABCD là hình bình hành. Điều phải chứng minh.

www.VNMATH.com

Page 25: Bất đẳng thức hình học

23

1.5. Bất đẳng thức trong các tam giác đặc biệt

1.5.1. Các bất đẳng thức trong tam giác đều

Tam giác đều có một số tính chất đặc biệt, nói chung không còn đúng

trong một tam giác tùy ý. Trong mục này, ta chỉ nghiên cứu một số bất

đẳng thức trong tam giác đều ABC liên quan mối quan hệ giữa pa, pb, pcvới PA, PB, PC (trong đó pa, pb, pc lần lượt là khoảng cách từ P đến

các cạnh BC,CA,AB).

Bài toán 1.11. Cho ABC là tam giác đều cạnh a, gọi P là một điểm

nằm trong tam giác. Chứng minh rằng

Hình 1.14

i)1

pa+

1

pb+

1

pc≥ 6√

3

a,

ii)1

pa + pb+

1

pb + pc+

1

pc + pa≥ 3√

3

a.

Giải. Gọi D,E, F là chân đường vuông

góc của P lên các cạnh BC,CA,AB tương

ứng. Ta có [ABC] = [BCP ] + [CAP ] +

[ABP ], do đó ah = apa + apb + apc. Vì

h =

√3

2a nên pa+pb+pc =

√3

2a. Áp dụng

bất đẳng thức (1.2), ta được

1

pa+

1

pb+

1

pc≥ 9

pa + pb + pc=

6√

3

a.

Lại áp dụng bất đẳng thức (1.2) có1

pa + pb+

1

pb + pc+

1

pc + pa≥ 9

pa + pb + pb + pc + pc + pa=

3√

3

a.

Bài toán 1.12. Cho tam giác đều ABC cạnh a và P là một điểm tùy

ý nằm trong tam giác. Chứng minh rằng

PA2.PB2.PC2 ≥ 8a3

3√

3papbpc. (1.42)

Trước hết ta chứng minh bổ đề sau

www.VNMATH.com

Page 26: Bất đẳng thức hình học

24

Bổ đề 1.1. Cho P là một điểm tùy ý nằm trong tam giác đều ABC

cạnh a. Kí hiệu α = BPC, β = CPA,γ = APB, ta có đẳng thức

PA2.PB2.PC2 =a3papbpc

sinα sin β sin γ. (1.43)

Chứng minh. Viết lại diện tích tam giác BPC theo hai cách ta được

BP.CP. sinα = a.pa, tương tự CP.AP. sin β = a.pb, AP.BP. sin γ = a.pc.

Nhân theo vế 3 đẳng thức này, ta thu được đẳng thức (1.43).

Giải. Gọi f(x) = ln(sinx), x ∈ (0, π) .Vì f ′′(x) = − 1sin2 x

< 0, nên f là

hàm lồi. Áp dụng bất đẳng thức Jensen, ta có

ln(sinα) + ln(sin β) + ln(sin γ)

3≤ ln(sin

α + β + γ

3).

Suy ra sinα. sin β. sin γ ≤ 3√3

8 . Thay bất đẳng thức này vào (1.43) ta

được bất đẳng thức (1.42).

Bài toán 1.13. Cho tam giác đều ABC cạnh a và P là một điểm tùy

ý nằm trong tam giác. Chứng minh rằng

PA.PB.PC ≥ 8papbpc. (1.44)

Giải. Vì [BPC] + [CPA] + [APB] = [ABC] nên1

2apa +

1

2apb +

1

2apc =

a2√

3

4. Suy ra pa + pb + pc =

a√

3

2. Do đó papbpc ≤

(pa+pb+pc

3

)3= a3

√3

72 .

Thay bất đẳng thức này vào (1.42), ta được bất đẳng thức (1.44).

Bài toán 1.14. Cho tam giác đều ABC cạnh a và P là một điểm tùy

ý nằm trong tam giác. Chứng minh rằng

PA.PB + PB.PC + PC.PA ≥ a2. (1.45)

Giải. Vì α2 + β2 + γ

2 = 1800 nên

cosα + cos β + cos γ ≥ 3

2, (1.46)

dấu đẳng thức xảy ra khi và chỉ khi α = β = γ = 1200.

Bây giờ, áp dụng định lí hàm số cosin cho tam giác PAB, ta có a2 =

www.VNMATH.com

Page 27: Bất đẳng thức hình học

25

PA2 +PB2− 2PA.PB. cos γ hay cos γ = PA2+PB2−a22PA.PB . Tương tự cosα =

PB2+PC2−a22PB.PC , cos β = PC2+PA2−a2

2PC.PA .

Cộng theo vế 3 bất đẳng thức trên và sử dụng (1.46), ta được

PA2 + PB2 − a2

2PA.PB+PB2 + PC2 − a2

2PB.PC+PC2 + PA2 − a2

2PC.PA+

3

2≥ 0,

Do đó(PA2.PB + PB2.PA+ PA.PB.PC

)+(PC2.PB + PB2.PC + PA.PB.PC

)+

+(PA2.PC + PC2.PA+ PA.PB.PC

)− a2 (PA+ PB + PC) ≥ 0,

tương đương với

(PA+ PB + PC)(PA.PB + PB.PC + PC.PA− a2

)≥ 0.

Từ đó có bất đẳng thức PA.PB + PB.PC + PC.PA ≥ a2.

Dấu đẳng thức xảy ra khi và chỉ khi P ≡ O.

Tam giác Pompeiu

Năm 1936, nhà toán học Rumani, Dimitrie Pompeiu phát hiện kết

quả đơn giản nhưng đẹp sau đây trong hình học phẳng Euclide

Định lý 1.31. (Định lý Pompeiu). Cho P là một điểm tùy ý nằm

trong mặt phẳng chứa tam giác đều ABC. Khi đó các khoảng cách

PA, PB, PC là độ dài các cạnh của một tam giác. Tam giác này suy

biến nếu điểm P nằm trên đường tròn ngoại tiếp tam giác ABC.

Chứng minh.

Hình 1.15

Xét trường hợp điểm P không nằm trên

đường tròn ngoại tiếp tam giác ABC. Áp

dụng bất đẳng thức Ptolemy (Xem định lí

2.2) cho 4 điểm A,B, P, C ta có

PA.BC < PC.AB + PB.AC,

PB.AC < PA.BC + PC.AB,

PC.AB < PA.BC + PB.AC.

Vì tam giác ABC đều nên AB = BC =

CA = l, do đó PA < PC + PB, PB < PA+ PC, PC < PA+ PB. Vậy

PA, PB, PC là độ dài 3 cạnh của một tam giác.

www.VNMATH.com

Page 28: Bất đẳng thức hình học

26

Chú ý 1.2.

+ Tam giác với độ dài các cạnh bằng PA, PB, PC được gọi là tam

giác Pompeiu.

+ Khi P nằm ở trong tam giác ABC, tam giác Pompeiu có thể được

xây dựng một cách dễ dàng như sau: Quay tam giác ABP quanh tâm A,

một góc 600, được tam giác AB′C. Khi đó AP = AB′ = PB′, BP = CB′,

tam giác Pompeiu sẽ là ∆PCB′.

Bài toán 1.15. Cho tam giác đều ABC cạnh a và P là một điểm tùy

ý nằm trong tam giác. Chứng minh rằng

3Rp

√3 ≥ PA+ PB + PC ≥ a

√3. (1.47)

Trong đó Rp là bán kính đường tròn ngoại tiếp tam giác Pompeiu có độ

dài các cạnh bằng PA, PB, PC.

Giải. Gọi M là trung điểm của BC, theo bất đẳng thức tam giác AP +

PM ≥ AM , mặt khác lại có PM ≤ PB+PC2 . Suy ra 2PA+ PB + PC ≥

a√

3, tương tự 2PB+PC +PA ≥ a√

3, 2PC +PA+PB ≥ a√

3. Cộng

theo vế 3 bất đẳng thức này ta được PA+ PB + PC ≥ a√

3.

Áp dụng PA2 + PB2 + PC2 ≤ 9R2p và bất đẳng thức Cauchy-Schwarz,

ta có 9R2p ≥ PA2 + PB2 + PC2 ≥ 1

3 (PA+ PB + PC)2.

Suy ra 3Rp

√3 ≥ PA+ PB + PC.

Bài toán 1.16. Cho P là một điểm nằm trong tam giác đều ABC cạnh

a. Gọi T là diện tích của tam giác Pompeiu với độ dài các cạnh bằng

PA, PB, PC. Khi đó

T =

√3

12

(a2 − 3d2

). (1.48)

Trong đó d = OP (O là tâm của tam giác đều ABC).

Giải.

Ta thực hiện 3 phép quay tương tự như hình 1.15, đó là quay tam

giác APB quanh tâm A một góc 600, quay tam giác BPC quanh tâm

B một góc 600 và quay tam giác CPA quanh tâm C một góc 600. Ta

sẽ được một hình lục giác AB′CA′BC ′, trong đó các tam giác Pompeiu

PBA′, PAC ′, PCB′ có cùng diện tích T .

www.VNMATH.com

Page 29: Bất đẳng thức hình học

27

Hình 1.16

Vì ∆APC = ∆BA′C, ∆APB =

∆AB′C, ∆AC ′B = ∆BPC nên diện tích

của lục giác = 2. [ABC] =2a2√

3

4. Mặt

khác ∆APB′, BPC ′, CPA′ là các tam giác

đều. Do đó

2a2√

3

4= 3T+

PA2√

3

4+PB2√

3

4+PC2√

3

4

Luôn có

PA2+PB2+PC2 = 3OP 2+OA2+OB2+OC2

Vì OP = d,OA = OB = OC =a2√

3

3nên

PA2 + PB2 + PC2 = 3d2 + a2. Từ đó ta được công thức (1.48).

Nhận xét 1.1. Từ công thức (1.47) và (1.48) ta có các hệ quả sau:

1) T ≤√

3

12a2. Đẳng thức xảy ra khi và chỉ khi P ≡ O.

2) rp ≤ a6 ≤

Rp

2 , trong đó Rp, rp lần lượt là bán kính vòng tròn ngoại

tiếp, nội tiếp tam giác Pompeiu với độ dài các cạnh bằng PA, PB, PC.

1.5.2. Các bất đẳng thức trong tam giác vuông và tam giác

cân

Bài toán 1.17. Cho góc vuông xAy. B là điểm trên tia Ax, C là điểm

trên tia Ay (B 6= A;C 6= A). Chứng minh rằng AB +√

3AC ≤ 2BC.

Giải.

Hình 1.17

Trong góc xAy vẽ tia Az sao cho

xAz = 300, do đó yAz = 600. Vẽ

BH⊥Az,CK⊥Az(H,K ∈ Az), Az cắt

BC tại I. Xét ∆ABH có AHB =

900, BAH = 300 nên là nửa tam giác đều,

cạnh AB. Suy ra BH = 12AB mà BH ≤

BI. Do đó

AB ≤ 2BI. (1.49)

www.VNMATH.com

Page 30: Bất đẳng thức hình học

28

Xét ∆ACK có AKC = 900, CAK = 600 nên là nửa tam giác đều, cạnh

AC. Suy ra CK = AC√3

2 mà CK ≤ IC. Do đó

√3AC ≤ 2IC. (1.50)

Cộng theo vế (1.49) và (1.50) được AB +√

3AC ≤ (BI +CI) = 2BC.

Bài toán 1.18. Cho tam giác ABC cân tại A. D là điểm trên cạnh BC,

E là điểm trên trên tia đối của tia CB sao cho CE = BD. Chứng minh

rằng AD + AE > 2AB.

Giải.

Hình 1.18

Trên tia đối của tia BC lấy điểm

F sao cho BF = BD. Áp dụng

định lí 1.26 ta có AD + AF > 2AB.

Mặt khác, xét ∆ABF và ∆ACE có

AB = AC, ABF = ACE (vì ABC =

ACB, ABC+ABF = ACE+ACB =

1800), BF = CE. Do đó ∆ABF =

∆ACE (c.g.c). Suy ra AF = AE. Vậy

AD + AE > 2AB.

Bài toán 1.19. (Iran, 2005) Cho tam giác ABC vuông tại A. Gọi D là

giao điểm của phân giác trong của góc A với cạnh BC và Ia là tâm của

đường tròn bàng tiếp cạnh BC của tam giác ABC. Chứng minh rằngAD

DIa≤√

2− 1.

Giải.

Hình 1.19

Gọi E,F lần lượt là điểm tiếp xúc của

đường tròn bàng tiếp với các cạnh AB,AC

tương ứng. Ta có ra = IaE = AF = FIa =

p, trong đó p là nửa chu vi của tam giác

ABC. Hơn nữa, ADDIa

= hara. Vì aha = bc,

ta có ADDIa

= hara

= bcap =

(bac4R

) (4Rra2

) (1rp

)=

4Rra2 . Vì 2R = a và 2r = b + c − a nên

www.VNMATH.com

Page 31: Bất đẳng thức hình học

29

ADDIa

= b+c−aa = b+c

a − 1. Mặt khác a =√b2 + c2 ≥

√(b+c)2

2 = b+c√2hay

b+ c ≤ a√

2. Do đó ADDIa≤√

2− 1.

Bài toán 1.20. (Rumani, 2007) Cho ABC là một tam giác vuông cân

tại A. Với điểm P tùy ý nằm trong tam giác, xét đường tròn tâm A và

bán kính AP cắt các cạnh AB và AC tại M và N , tương ứng. Hãy xác

định vị trí của P để MN +BP + CP đạt giá trị nhỏ nhất.

Giải.

Hình 1.20

Xét điểm Q trên đường trung trực

của BC thỏa mãn AQ = AP . Gọi S

là giao điểm của BP và tiếp tuyến với

đường tròn tại Q. Khi đó SP +PC ≥SC. Do đó BP + PC = BS + SP +

PC ≥ BS + SC.

Mặt khác, BS+SC ≥ BQ+QC, nên

BP + PC đạt giá trị nhỏ nhất nếu

P ≡ Q.

Gọi T là trung điểm của MN . Vì

∆AMQ cân và MT là một trong

những chiều cao của nó, khi đóMT =

ZQ trong đó Z là chân đường vuông

góc hạ từ Q xuống AB. Khi đó MN + BQ + QC = 2(MT + QC) =

2(ZQ+QC) đạt giá trị nhỏ nhất khi Z,Q,C thẳng hàng và điều này có

nghĩa CZ là chiều cao. Bằng phép đối xứng, BQ cũng là chiều cao và

do đó P là trực tâm.

1.6. Các bất đẳng thức khác trong tam giác

Bài toán 1.21. (Bất đẳng thức Weitzenbock, IMO 1961) Gọi a, b, c là

độ dài các cạnh của một tam giác. Chứng minh rằng

a2 + b2 + c2 ≥ 4√

3S. (1.51)

Giải. Theo bất đẳng thức AM - GM ta có

S =√p(p− a)(p− b)(p− c) ≤

√p[(p−a)+(p−b)+(p−c)

3

]3=

√3

9p2.

www.VNMATH.com

Page 32: Bất đẳng thức hình học

30

Do đó, theo bất đẳng thức Cauchy - Schwarz ta có

4√

3S ≤ 4√

3

[√3

9

(a+ b+ c

2

)2]

=1

3(a+ b+ c)2

≤ 1

3

(a2 + b2 + c2

) (12 + 12 + 12

)= a2 + b2 + c2.

Bài toán sau cho ta một kết quả mạnh hơn bất đẳng thức Weitzenbock.

Bài toán 1.22. (Bất đẳng thức Hadwiger - Finsler) Gọi a, b, c là độ dài

các cạnh của một tam giác. Chứng minh rằng

a2 + b2 + c2 ≥ 4√

3S + (a− b)2 + (b− c)2 + (c− a)2 (1.52)

Đẳng thức xảy ra khi và chỉ khi a = b = c.

Giải. Vì a2 = b2 + c2 − 2bccosA và S = 12bcsinA nên ta có

a2 + b2 + c2 − 4√

3S = 2(b2 + c2)− 2bccosA− 2√

3bcsinA

=2(b2 + c2)− 4bccos(π

3− A

)≥ 2(b2 + c2 − 2bc) = 2 (b− c)2 . (1.53)

Mặt khác, không mất tính tổng quát ta giả sử b ≤ a ≤ c. Khi đó ta có

bất đẳng thức

(b− c)2 ≥ (a− b)2 + (c− a)2. (1.54)

Vì bất đẳng thức trên tương đương với bất đẳng thức (a− b)(a− c) ≤ 0.

Từ bất đẳng thức (1.53) và (1.54) ta được bất đẳng thức (1.52).

Đẳng thức xảy ra khi và chỉ khi

cos(π3 − A

)= 1[

a = b

a = c

⇔ a = b = c.

Bài toán 1.23. (Điểm Torricelli) Tìm điểm O trong tam giác ABC cho

trước sao cho tổng khoảng cách từ điểm O tới ba đỉnh của tam giác là

nhỏ nhất có thể.

Giải. Xét làm hai trường hợp:

a) Tam giác ABC có ba góc nhỏ hơn 1200.

www.VNMATH.com

Page 33: Bất đẳng thức hình học

31

Dựng tam giác đều BCD ở phía ngoài của tam giác ABC. Gọi T là

giao điểm của đường tròn ngoại tiếp tam giác BCD với AD. Dễ chứng

minh rằng T nhìn ba cạnh của tam giác ABC dưới ba góc bằng nhau.

Ta chứng minh rằng với một điểm O tùy ý ở trong tam giác ABC khác

điểm T thì ta có OA+OB +OC ≥ TA+ TB + TC.

Hình 1.21

Điểm T được gọi là điểm

Torricelli của tam giác ABC

và có tổng các khoảng cách

tới các đỉnh của tam giác

ABC nhỏ nhất.

Thật vậy, theo định lí

Pompeiu, ta có OB +OC ≥OD, do đó

OA+OB +OC ≥ OA+OD ≥ AD. (1.55)

Mặt khác, vì T nằm trên đường tròn ngoại tiếp tam giác đều BCD nên

TA+ TB + TC = TA+ TD = AD. (1.56)

Từ (1.55) và (1.56) suy ra OA + OB + OC ≥ TA + TB + TC. Đẳng

thức xảy ra khi và chỉ khi O ≡ T .

b) Tam giác ABC có một góc, chẳng hạn B > 1200

Hình 1.22

Dựng tam giác đều BCD ở phía ngoài

tam giác ABC. Do B > 1200, cho nên với

điểm O tùy ý ở trong tam giác ABC, điểm

B nằm trong tam giác ODA. Theo định lí

Pompeiu, ta có OB +OC ≥ OD.

Mặt khác, theo định lí 1.20 đối với tam

giác ODA, ta có OA + OD ≥ BA + BD.

Từ đó ta có OA + OB + OC ≥ OA + OD ≥ BA + BD = BA + BC.

Như vậy, khi O ≡ B, tổng khoảng cách từ O tới các đỉnh của tam giác

ABC là nhỏ nhất có thể.

Tóm lại, trong trường hợp tam giác ABC có một đỉnh không nhỏ hơn

1200, thì chính đỉnh này là đỉnh cần tìm.

www.VNMATH.com

Page 34: Bất đẳng thức hình học

32

Bài toán 1.24. Cho tam giác ABC nội tiếp đường tròn (O) với

H là trực tâm và AD,BE,CE là các đường cao. Kí hiệu D′ =

AD ∩ (O), E ′ = BE ∩ (O), F ′ = CF ∩ (O). Chứng minh rằng

(i)AD

HD+BE

HE+CF

HF≥ 9, (ii)

HD

HA+HE

HB+HF

HC≥ 3

2,

(iii)AD

DD′+BE

EE ′+CF

FF ′≥ 9, (iv)

AD

AD′+BE

BE ′+CF

CF ′≥ 9

4.

Giải.

Hình 1.23

+ Chứng minh i).

Ta có [HBC][ABC] = HD

AD ,[HCA][ABC] = HE

BE ,[HAB][ABC] =

HFCF nên HD

AD + HEBE + HF

CF = 1. Bây giờ, áp

dụng bất đẳng thức (1.2) ta được ADHD +

BEHE + CF

HF ≥9

HDAD+HE

BE+HFCF

= 9.

+ Chứng minh ii).

Ta có HDHA = HD

AD−HD = [HBC][ABC]−[HBC] =

[HBC][HCA]+[HAB] . tương tự, HE

HB = [HCA][HAB]+[HBC] ,

HFHC = [HAB]

[HBC]+[HCA] . Cộng 3 bất đẳng thức

này, sau đó áp dụng bất đẳng thức Nesbitt

ta được bất đẳng thức HDHA + HE

HB + HFHC ≥

32 .

+ Chứng minh iii).

Dễ chứng minh được HD = DD′, HE = EE ′ và HF = FF ′. Thay vào

(i) ta được bất đẳng thức ADDD′ + BE

EE′ + CFFF ′ ≥ 9.

+ Chứng minh iv).

Ta có AD′

AD = AD+DD′

AD = 1 + HDAD . Tương tự, BE

BE = 1 + HEBE ,

CF ′

CF = 1 + HFCF .

Cộng 3 đẳng thức này ta được AD′

AD + BE′

BE + CF ′

CF = 3 + HDAD + HE

BE + HFCF =

4. Bây giờ áp dụng bất đẳng thức (1.2) ta được ADAD′ + BE

BE′ + CFCF ′ ≥

9AD′AD +BE′

BE +CF ′CF

= 94 .

Bài toán 1.25. (Tạp chí THTT số 266) Giả sử M là điểm nằm trong

tam giác ABC. Gọi A1, B1, C1 lần lượt là hình chiếu của M trên các

đường thẳng BC,CA,AB. Chứng minh rằng

MA2

(MB1 +MC1)2 +

MB2

(MC1 +MA1)2 +

MC2

(MA1 +MB1)2 ≥ 3.

www.VNMATH.com

Page 35: Bất đẳng thức hình học

33

Đẳng thức xảy ra khi nào ?

Giải.

Hình 1.24

Ta có

MB1 +MC1

MA=MB1

MA+MC1

MA

= sin MAB1 + sin MAC1

≤2 sinMAB1 + MAC1

2= 2 sin

A

2.

Suy ra MAMB1+MC1

≥ 12 sin A

2

. Tương tự ta cóMB

MC1+MA1≥ 1

2 sin B2

, MCMA1+MB1

≥ 12 sin C

2

. Vậy

MA2

(MB1 +MC1)2 +

MB2

(MC1 +MA1)2 +

MC2

(MA1 +MB1)2

≥1

4

(1

sin2 A2

+1

sin2 B2

+1

sin2 C2

)

≥3

43

√1

sin2 A2 sin2 B

2 sin2 C2

≥ 3

43

√1(18

)2 = 3.

Đẳng thức xảy ra khi và chỉ khi ∆ABC đều và M là tâm của tam giác.

Bài toán 1.26. (Trích đề thi học sinh giỏi Quốc gia, 1991) Cho tam

giác ABC với trọng tâm là G và nội tiếp trong đường tròn bán kính R,

các đường trung tuyến của tam giác này xuất phát từ các đỉnh A,B,C

kéo dài cắt đường tròn lần lượt tại D,E, F . Chứng minh rằng

3

R≤ 1

GD+

1

GE+

1

GF≤√

3

(1

AB+

1

BC+

1

CA

).

Giải. Gọi M,N,P lần lượt là trung điểm của cạnh BC,CA,AB. Từ

AM.MD = BM.MC có MD = a2

4ma. Suy ra

GD = GM +MD =ma

3+

a2

4ma(1.57)

www.VNMATH.com

Page 36: Bất đẳng thức hình học

34

Hình 1.25

Từ (1.57), áp dụng bất đẳng thức AM−GM ta được GD ≥ 2

√a2

12 = a√3⇒ 1

GD ≤√3

BC . Tương tự, có 1GE ≤

√3

CA ,1GF ≤

√3

AB .

Cộng theo vế 3 bất đẳng thức này ta được1GD + 1

GE + 1GF ≤

√3(

1AB + 1

BC + 1CA

).

Lại từ (1.57) có GAGD =

2ma

3(ma

3 + a2

4ma

) =

8m2a

4m2a + 3a2

. Áp dụng công thức đường

trung tuyến được GAGD =

2b2 + 2c2 − a2

a2 + b2 + c2.

Tương tự tính GBGE và GC

GF rồi cộng lại được GAGD+GB

GE+GCGF = 3a2+3b2+3c2

a2+b2+c2 = 3.

Từ đó ADGD + BE

GE + CFGF = 6. Để ý rằng AD,BE,CF đều không lớn hơn

2R, thay vào ta được 1GD + 1

GE + 1GF ≥

3R .

Bài toán 1.27. (Tây ban nha, 1998) Một đường thẳng chứa trọng tâm

G của tam giác ABC cắt cạnh AB tại P và cạnh CA tại Q. Chứng minh

rằng PBPA .

QCQA ≤

14 .

Giải. Vì PBPA .QCQA ≤

14

(PBPA + QC

QA

)2, ta sẽ chứng minh PB

PA + QCQA = 1.

Hình 1.26

Vẽ BB′, CC ′ song song với trung

tuyến AA′ mà B′, C ′ nằm trên PQ.

Các tam giác APG và BPB′ đồng

dạng ; tam giác AQG và CQC ′ cũng

đồng dạng, do đó PBPA = BB′

AG và QCQA =

CC ′

AG . Cộng 2 đẳng thức này lại với lưu

ý rằng AG = 2GA′ = BB′ + CC ′ ta

được PBPA + QC

QA = 1. Từ đó được bất

đẳng thức cần chứng minh.

Bài toán 1.28. (Ba Lan, 1999) Cho D là một điểm trên cạnh BC

của tam giác ABC sao cho AD > BC. Điểm E trên CA thỏa mãnAE

EC=

BD

AD −BC. Chứng minh rằng AD > BE.

Giải.

www.VNMATH.com

Page 37: Bất đẳng thức hình học

35

Hình 1.27

Lấy F trên AD sao cho AF = BC

và Gọi E ′ là giao điểm của BF và

AC. Áp dụng định lí hàm số sin cho

tam giác AE ′F,BCE ′ và BDF , ta

được AE′

E′C = AF. sin AFE′

sin AE′F. sin BE′C

BC. sin CBE′=

sin BFD

sin DBF= BD

FD = AEEC . Do đó E ′ ≡ E.

Lấy G trên BD sao cho BG = AD

và H là giao điểm của GE với đường

thẳng qua A và song song với BC. Dễ thấy các tam giác ECG và EAH

đồng dạng nên AHCG = AE

EC = BDAD−BC = BD

BG−BC = BDCG , suy ra AH = DB.

Do đó BDAH là một hình bình hành, suy ra BH = AD và ∆BHG cân.

Vậy BH = BG = AD > BE.

Bài toán 1.29. (Tạp chí THTT số 265) Gọi AD,BE,CF là các đường

phân giác trong của tam giác ABC. Chứng minh rằng

p(DEF ) ≥ 1

2p(ABC),

trong đó kí hiệu p(XY Z) là chu vi của tam giác XY Z. Đẳng thức xảy

ra khi nào ?

Giải.

Hình 1.28

Từ tính chất của đường

phân giác BE ta có AECE = c

a ,

suy ra AEb = AE

AE+CE = ca+c .

Do đó AE = bca+c . Tương tự

AF = bca+b .

Theo định lí cosin

trong ∆AEF và ∆ABC

EF 2 =AE2 + AF 2 − 2AF.AF. cosA

=

(bc

a+ c

)2

+

(bc

a+ b

)2

− 2b2c2

(a+ c) (a+ b).b2 + c2 − a2

2bc

www.VNMATH.com

Page 38: Bất đẳng thức hình học

36

=a2bc

(a+ c) (a+ b)− abc (a+ b+ c) (b− c)2

(a+ c)2 (a+ b)2

Suy ra EF 2 ≤ a2bc(a+c)(a+b) . Từ đó

EF 2 ≤ a2bc

4√ac.√ab

=1

4.√ac.√ab ≤ 1

4

(√ac+

√ab

2

)2

≤ 1

16

(a+ c

2+a+ b

2

)2

=1

16

(2a+ b+ c

2

)2

.

Do đó EF ≤ 2a+b+c8 . Tương tự FD ≤ 2b+c+a

8 , DE ≤ 2c+a+b8 .

Cộng theo vế 3 bất đẳng thức trên được DE+EF +FD ≤ 12 (a+ b+ c).

hay p(DEF ) ≥ 12p(ABC).

Bài toán 1.30. (IMO, 1991 ) Cho tam giác ABC. Gọi I là tâm đường

tròn nội tiếp tam giác. Đường phân giác trong của các góc A,B,C lần

lượt cắt các cạnh đối diện tương ứng tại L,M,N . Chứng minh rằng

1

4≤ AI.BI.CI

AL.BM.CN≤ 8

27.

Giải.

Hình 1.29

Sử dụng tính chất đường phân

giác có BLLC = c

b , để ý rằng BL +

LC = a, ta được BL = acb+c và

LC = abb+c . Tiếp tục, áp dụng tích

chất đường phân giác cho phân

giác BI của góc ABL ta thu đượcILAI = BL

AB = ac(b+c)c = a

b+c . Do đóALAI = AI+IL

AI = 1 + ILAI = 1 + a

b+c =a+b+cb+c . Khi đó, AIAL = b+c

a+b+c . Tương

tự, BIBM = c+a

a+b+c ,CICN = a+b

a+b+c . Do đó bất đẳng thức cần chứng minh đưa

về dạng chứa các biến a, b và c

1

4<

(b+ c)(c+ a)(a+ b)

(a+ b+ c)3≤ 8

27.

www.VNMATH.com

Page 39: Bất đẳng thức hình học

37

Áp dụng bất đẳng thức AM −GM có

(b+ c)(c+ a)(a+ b) ≤(

(b+ c) + (c+ a) + (a+ b)

3

)3

≤ 8

27(a+ b+ c)3 .

Bất đẳng thức phải được chứng minh.

Để chứng minh bất đẳng thức trái, trước hết để ý rằng

(b+ c)(c+ a)(a+ b)

(a+ b+ c)3=

(a+ b+ c)(ab+ bc+ ca)− abc(a+ b+ c)3

. (1.58)

Biết rằng a+ b+ c = 2p, ab+ bc+ ca = p2 + r2 + 4rR, abc = 4Rrp thay

vào (1.58) được

(b+ c)(c+ a)(a+ b)

(a+ b+ c)3=

2p(p2 + r2 + 4rR)− 4Rrp

8p3=

=2p2 + 2pr2 + 4Rrp

8p3=

1

4+

2r2 + 4Rr

8p2>

1

4.

Bài toán 1.31. (IMO Shorlist, 1996) Cho ABC là tam giác đều và

P là một điểm trong nó. Các đường thẳng AP,BP,CP cắt các cạnh

BC,CA,AB tại các điểm A1, B1, C1, tương ứng. Chứng minh rằng

A1B1.B1C1.C1A1 ≥ A1B.B1C.C1A.

Giải. Áp dụng định lí hàm số cosin cho ∆CA1B1, ta được

A1B21 = A1C

2 +B1C2 − A1C.B1C ≥ A1C.B1C.

Tương tự, B1C21 ≥ B1A.C1A,C1A1 ≥ C1B.A1B. Nhân 3 bất đẳng thức

này, ta được

A1B21 .B1C

21 .C1A

21 ≥ A1C.B1C.B1A.C1A.C1B.A1B. (1.59)

Bây giờ, các đường thẳng AA1, BB1, CC1 đồng quy, vì vậy áp dụng định

lí Ceva ta có A1B.B1C.C1A = AB1.BC1.CA1, thay vào (1.59) ta thu

được bất đẳng thức cần chứng minh. Đẳng thức xảy ra khi và chỉ khi

CA1 = CB1, BA1 = BC1 và AB1 = AC1. Điều này xảy ra khi và chỉ khi

P là tâm của đường tròn ngoại tiếp tam giác ABC.

www.VNMATH.com

Page 40: Bất đẳng thức hình học

38

Bài toán 1.32. (IMO Shorlist, 1999) Cho tam giác ABC và M là một

điểm nằm trong nó. Chứng minh rằng

min MA,MB,MC+MA+MB +MC < AB +BC + CA.

Trước hết, ta chứng minh bổ đề sau:

Bổ đề 1.2. Nếu M là một điểm nằm trong tứ giác lồi ABCD thì

MA+MB < AD +DC + CB.

Chứng minh.

Hình 1.30 Hình 1.31

Gọi N là giao điểm của AM và CD (Hình 1.30). Khi đóMA+MB <

MA + MN + NB ≤ AN + NC + CB ≤ AD + DN + NC + CB =

AD +DC + CB.

Giải. Lấy D,E, F theo thứ tự là trung điểm của BC,CA,AB (Hình

1.31). Xét điểm M nằm trong tam giác, M sẽ thuộc ít nhất hai trong ba

hình thang ABDE,BCEF,CAFD. Không mất tính tổng quát, giả sử

M ∈ ABDE,BCEF . Áp dụng bổ đề trên ta có

MA+MB < AE + ED +DB,

MB +MC < BF + FE + EC.

Cộng theo vế hai bất đẳng thức trên, ta được

MB + (MA+MB +MC) < AB +BC + CA.

Vậy min MA,MB,MC+MA+MB +MC < AB +BC + CA.

www.VNMATH.com

Page 41: Bất đẳng thức hình học

39

Bài toán 1.33. (IMO Shorlist, 2002) Cho tam giác ABC và F là một

điểm trong nó thỏa mãn AFB = BFC = CFA . Các đường thẳng BF

và CF cắt các cạnh AC và AB tại D và E, tương ứng. Chứng minh

rằng

AB + AC ≥ 4DE.

Trước hết, ta chứng minh bổ đề sau:

Bổ đề 1.3. Cho tam giác ABC, các điểm P và Q nằm trên các tia FD,

FE tương ứng, sao cho PF ≥ λDF,QF ≥ λEF , trong đó λ > 0. Nếu

PFQ ≥ 900 thì PQ ≥ λDE.

Chứng minh. Đặt PFQ = θ. Vì θ ≥ 900, ta có cos θ ≤ 0. Bây giờ, áp

dụng định lý hàm số cosin, ta có PQ2 = PF 2 +QF 2− 2PF.QF. cos θ ≥(λDF )2 + (λEF )2 − 2 (λDF ) . (λEF ) . cos θ = (λDE)2. Do đó PQ ≥λDE.

Giải. Lưu ý rằng AFE = BFE = CFD = AFD = 600. Gọi P,Q là giao

điểm của các đường thẳng BF,EF với đường tròn ngoại tiếp tam giác

CFA,AFB tương ứng. Khi đó, dễ dàng thấy cả hai tam giác CPA,AQB

là đều. Gọi P1 là chân đường vuông góc hạ từ F xuống cạnh AC và giả

sử đường trung trực của AC cắt đường tròn ngoại tiếp tam giác CFA

tại P và P2.

Hình 1.32

Gọi M là trung điểm

của AC. Khi đó PDDF =

PMFP1

≥ PMMP2

= 3 vì

vậy PF ≥ 4DF . Tương

tự, ta có QF ≥ 4EF .

Áp dụng hệ quả trên

với λ = 4 và θ =

DEF = 1200 ta được

PQ ≥ 4DE. Cuối cùng,

áp dụng bất đẳng thức

tam giác, AB + AC = AQ+ AP ≥ PQ ≥ 4DE.

Bài toán 1.34. (IMO, 2006) Cho tam giác ABC với I là tâm đường

tròn nội tiếp. Một điểm P nằm trong tam giác thỏa mãn PBA+ PCA =

www.VNMATH.com

Page 42: Bất đẳng thức hình học

40

PBC + PCB. Chứng minh rằng AP > AI và đẳng thức xảy ra khi và

chỉ khi P ≡ I.

Giải. Đặt A = α, B = β, C = γ. Vì PBA + PCA + PBC + PCB =

β + γ nên từ điều kiện bài toán ta có PBC + PCB =β + γ

2. Suy ra

BPC = 900 +α

2.

Hình 1.33

Mặt khác, BIC = 1800 − β + γ

2=

900 +α

2. Do đó BPC = BIC, và vì

P và I nằm cùng phía với BC nên

các điểmB, I, P, C cùng nằm trên một

đường tròn. Nói cách khác, P nằm

trên đường tròn ω ngoại tiếp tam giác

BCI. Gọi Ω là đường tròn ngoại tiếp

tam giác ABC. Rõ ràng tâm của ω

là trung điểm M của cung BC của

Ω. Đây cũng là giao điểm thứ hai của

phân giác AI và ω. Từ tam giác APM

ta có AP +PM ≥ AM = AI+IM = AI+PM . Do đó AP ≥ AI. Đẳng

thức xảy ra khi và chỉ khi P nằm trên đoạn AM , điều này xảy ra khi và

chỉ khi P ≡ I.

1.7. Các bất đẳng thức trong tứ giác

Kí hiệu ABCD là tứ giác lồi với các đỉnh là A,B,C,D được vẽ theo

một chiều nhất định nào đó (cùng chiều kim đồng hồ hay ngược chiều

kim đồng hồ). Để đơn giản, độ lớn của góc ứng với các đỉnh A,B,C,D

cũng được kí hiệu là A,B,C,D.

Độ dài các cạnh của tam giác: AB = a,BC = b, CD = c,DA = d.

Nủa chu vi của tứ giác: p =a+ b+ c+ d

2.

Độ dài các đường chéo: AC = m,BD = n.

Diện tích của tứ giác: S = SABCD hay [ABCD]

www.VNMATH.com

Page 43: Bất đẳng thức hình học

41

1.7.1. Các bất đẳng thức cơ bản trong tứ giác

Bài toán 1.35. Cho tứ giác ABCD, có AB+BD ≤ AC +DC. Chứng

minh AB < AC.

Giải. Gọi O là giao điểm của AC và BD . Xét các tam giác OAB và

ODC ta có

Hình 1.34

AB < OA + OB, DC < OC + OD.

Do đó

AB + CD < (OA+OC) + (OB +OD)

=AC +BD. (1.60)

Mặt khác, theo giả thiết ta có

AB +BD ≤ AC +DC. (1.61)

Cộng theo vế (1.60) và (1.61) ta được 2AB+DC +BD < 2AC +BD+

DC. Suy ra AB < AC.

Bài toán 1.36. Tổng hai đường chéo của một tứ giác lồi ABCD nhỏ

hơn chu vi của tứ giác và lớn hơn nửa chu vi của nó.

Giải. Gọi O là giao điểm của AC và BD, ta có

AC +BD = (OA+OB) + (OC +OD) > AB + CD. (1.62)

AC +BD = (OA+OD) + (OB +OC) > AD +BC. (1.63)

Cộng theo vế các bất đẳng thức (1.62) và (1.63) ta thu được

AC +BD >AB +BC + CD +DA

2.

Mặt khác, AC < AB + BC và AC < DA + CD. Cộng theo vế hai bất

đẳng thức này, ta có

AC <AB +BC + CD +DA

2. (1.64)

Tương tự,

BD <AB +BC + CD +DA

2. (1.65)

Cộng theo vế hai bất đẳng thức (1.64) và (1.65) , ta thu được

AC +BD < AB +BC + CD +DA.

www.VNMATH.com

Page 44: Bất đẳng thức hình học

42

Bài toán 1.37. Cho tứ giác ABCD, gọi M,N,P,Q lần lượt là trung

điểm của các cạnh AB,BC,CD,DA. Chứng minh rằng

a)MP ≤ AD +BC

2. (1.66)

b)MP +NQ ≤ AB +BC + CD +DA

2. (1.67)

Giải.

Hình 1.35

a) Gọi I là trung điểm của BD, MI

là đường trung bình của ∆ABD. Suy ra

MI = AD2 . IP là đường trung bình của

∆DBC, suy ra IP = BC2 . Xét 3 điểm

I,M, P ta có MP ≤ MI + IP , suy ra

MP ≤ AD+BC2 .

b) Áp dụng ý a) ta có NQ ≤ AB+DC2 .

Do đó MP + NQ ≤ AD+BC2 + AB+DC

2 =AB+BC+CD+DA

2 .

Bài toán 1.38. Cho hình vuông ABCD cạnh a. M,N là hai điểm ở

trong hình vuông đã cho. Chứng minh rằng MN ≤ a√

2.

Giải.

Hình 1.36

Vì ABCD là hình vuông cạnh

a nên AC = AB√

2 = a√

2.

Vẽ đường tròn (O) ngoại tiếp

hình vuông ABCD. Ta có đường

kính của (O) là a√

2, M và N là

hai điểm nằm trong O.

Gọi M ′N ′ là dây cung đi qua M

và N . Ta có MN ≤ M ′N ′ mà

M ′N ′ ≤ a√

2 (đường kinh là dây

cung lớn nhất trong đường tròn).

Do đó MN ≤ a√

2.

Bài toán 1.39. Cho hình thang ABCD có đáy nhỏ là AB và C + D ≤900. GọiM và N lần lượt là trung điểm của các cạnh AB và CD. Chứng

minh rằng MN ≤ CD−AB2 .

www.VNMATH.com

Page 45: Bất đẳng thức hình học

43

Giải.

Hình 1.37

Qua M vẽ đường thẳng song song

với AD cắtDC tại E và qua M vẽ

đường thẳng song song với BC cắt

DC tại F . Suy ra D = E1, C = F1.

Suy ra E1 + F1 = D + C ≤ 900 ⇒EMF ≥ 900. Theo bài toán 1.3 tam

giác MEF có EMF ≥ 900 và MN là

trung tuyến nên MN ≤ EF2 .

Mặt khác, có AB//DC và AD//ME nên ADEM là hình bình hành.

Suy ra DE = AM = 12AB, tương tự FC = MB = 1

2AB. Do đó

EF = CD − AB.

Vậy MN ≤ CD−AB2 .

Bài toán 1.40. Cho tứ giác ABCD, M là một điểm thuộc cạnh CD

(M khác C,D). Chứng minh rằng

MA+MB < max CA+ CB;DA+DB . (1.68)

Giải.

Hình 1.38

Gọi A′ là điểm đối xứng của A qua CD.

A′B cắt CD ở P .

Vì M thuộc đoạn CD nên M thuộc

∆A′BC hoặc ∆A′BD. Theo định lí 1.20

ta có [MA′ +MB < CA′ + CB

MA′ +MB < DA′ +DB

⇒[MA+MB < CA+ CB

MA+MB < DA+DB

Do đó MA+MB < max CA+ CB;DA+DB.

Chú ý 1.3. Từ bài toán 1.40 ta có các kết quả sau:

1) Cho tứ giác ABCD, M là một điểm thuộc cạnh CD (M có

thể trùng với C hoặc D). Ta có bất đẳng thức MA + MB ≤max CA+ CB;DA+DB.

www.VNMATH.com

Page 46: Bất đẳng thức hình học

44

2) Cho ABCD là hình chữ nhật và điểm M nằm trên cạnh CD. Ta có

bất đẳng thức MA+MB ≤ CA+ CB.

3) Cho ABCD là hình vuông cạnh a và điểm M nằm trên cạnh CD. Ta

có bất đẳng thức MA+MB ≤ (1 +√

2)a.

Đẳng thức trong các bất đẳng thức trên xảy ra khi và chỉ khi M ≡ C

hoặc M ≡ D.

Bài toán 1.41. (Đề thi vào lớp 10 chuyên toán-tin, ĐHSP, ĐHQG Hà

Nội 1998-1999) Cho hình chữ nhật ABCD và điểm M nằm trong hình

chữ nhật và có thể nằm trên các cạnh của ABCD. Chứng minh rằng

MA+MB +MC +MD ≤ AB + AC + AD.

Giải.

Hình 1.39

Qua M vẽ đường thẳng song song với

AD cắt AB, DE lần lượt tại E, F . Áp

dụng chú ý 1.3 của bài toán 1.40 vào các

hình chữ nhật AEFD,EBCF và ABCD

ta cóMA+MD ≤ EA+ED,MB+MC ≤EB + EC,ED + EC ≤ AD + AC. Do đó

MA+MB +MC +MD ≤ (EA+EB) +

(ED + EC) ≤ AB + AC + AD.

Ta có bài toán tổng quát hơn sau đây

Bài toán 1.42. (Tuyển tập 5 năm tạp chí THTT) Cho tứ giác ABCD,

M là một điểm trong tứ giác. Đặt dA = AB + AC + AD, dB = BC +

BD + BA, dC = CD + CA + CB, dD = DA + DB + DC. Chứng minh

rằng

MA+MB +MC +MD < max dA; dB; dC ; dD .

Giải. Kéo dài AM một đoạn MB′ bằng MB. Qua M kẻ đường trung

trực của BB′. Đường này theo thứ tự cắt hai cạnh tứ giác tại I, J . Có

thể xảy ra một trong ba trường hợp hình (A), (B), (C). Vì trong các hình

(B), (C) bài toán được chứng minh tương tự nhưng đơn giản hơn trong

trường hợp (A) nên ở đây ta chỉ chứng minh trong trường hợp (A). Không

mất tính tổng quát giả sử rằng IC + ID = max IC + ID, JC + JD .

www.VNMATH.com

Page 47: Bất đẳng thức hình học

45

Hình 1.40

Áp dụng bài toán 1.40 cho tứ giác CIJD ta cóMC+MD < IC+ID.

Lại có MA+MB = MA+MB′ = AB′ < IA+ IB′ = IA+ IB. Do đó

MA+MB +MC +MD < IA+ IB + IC + ID = IA+ ID +BC. Áp

dụng bài toán 1.40 cho tứ giác ABCD ta có

IA+ ID < max CA+ CD;BA+BD .

Vậy MA+MB +MC +MD < max CA+ CD;BA+BD+BC =

= max BC +BD +BA;CD + CA+ CB = max dA; dC≤max dA; dB; dC ; dD .

Chú ý 1.4. Từ bài toán 1.42 ta có các kết quả sau:

1) Cho hình chữ nhật ABCD có độ dài các cạnh là a, b và độ dài đường

chéo là c. M là một điểm nằm bên trong hình chữ nhật đó. Ta có bất

đẳng thức MA+MB +MC +MD < a+ b+ c.

2) Cho hình vuông ABCD cạnh và M là một điểm nằm bên trong hình

vuông đó. Ta có bất đẳng thức MA+MB +MC +MD < (2 +√

2)a.

1.7.2. Các bất đẳng thức khác trong tứ giác

Bài toán 1.43. (IMO shorlist) Diện tích của một tứ giác với các cạnh

a, b, c và d là S. Chứng minh rằng

S ≤ a+ c

2.b+ d

2.

Giải. Trước tiên, giả sử tứ giác ABCD không lồi. Khi đó một trong các

đường chéo của nó, chẳng hạn BD sẽ không có điểm chung với phần

trong của tứ giác.

www.VNMATH.com

Page 48: Bất đẳng thức hình học

46

Hình 1.41

Lấy đối xứng điểm C qua BD cho ta

một tứ giác lồi ABC ′D có cùng cạnh nhưng

diện tích lớn hơn diện tích tứ giác ABCD.

Do đó không mất tính tổng quát, ta có thể

giả thiết rằng tứ giác ABCD lồi.

Bây giờ ta chia tứ giác bởi đường chéo AC

thành hai tam giác ABC và ADC. Ta có

[ABC] ≤ ab2 , [ADC] ≤ dc

2 . Do đó

S = [ABC] + [ADC] ≤ ab+ cd

2. (1.69)

Làm tương tự với đường chéo BD, ta có

S = [BAD] + [BCD] ≤ bc+ da

2. (1.70)

Cộng theo vế các bất đẳng thức (1.69) và (1.70), ta được

2S ≤ ab+ bc+ cd+ da

2=

(a+ c) (b+ d)

2hay S ≤ a+ c

2.b+ d

2.

Bài toán 1.44. (Olympic Tây Ban Nha, 2000) Chứng minh rằng trong

tất cả các tứ giác lồi có diện tích bằng 1, thì tổng độ dài các cạnh và

các đường chéo lớn hơn hoặc bằng 2(2 +√

2).

Giải. Gọi a, b, c, d lần lượt là độ dài các cạnh và e, f lần lượt là độ dài

các đường chéo của tứ giác. Ta sẽ chứng minh a + b + c + d ≥ 4 và

e+ f ≥ 2√

2.

Gọi S là diện tích tứ giác. Ta biết rằng S = 12ef sin θ, trong đó θ là

góc giữa hai đường chéo. Vì S = 1 nên ef ≥ 2. Áp dụng bất đẳng thức

AM −GM suy ra e+ f ≥ 2√

2. Đẳng thức xảy ra khi và chỉ khi e = f .

Mặt khác, theo bài toán trên có S ≤ a+c2 .

b+d2 . Áp dụng AM −GM và

sử dụng S = 1 ta suy ra a+ b+ c+ d ≥ 4. Đẳng thức xảy ra khi và chỉ

khi a = b = c = d.

Từ đó suy ra bất đẳng thức cần chứng minh. Cả hai đẳng thức xảy

ra khi và chỉ khi tứ giác lồi là hình vuông.

www.VNMATH.com

Page 49: Bất đẳng thức hình học

47

Bài toán 1.45. (Olympic Địa trung hải, 1998) Cho ABCD là một hình

vuông nội tiếp đường tròn. M là một điểm trên cung_

AB. Chứng minh

rằng MC.MD ≥ 3√

3MA.MB.

Giải.

Hình 1.42

Đặt α = ACM, β = BDM . Khi đó ta

có α + β = π4 và MA.MB

MC.MD = tanα. tan β.

Bây giờ, để ý rằng

tanα. tan β. tan γ ≤ tan3

(α + β + γ

3

),

ở đây γ = π4 .

Suy ra tanα. tan β ≤ 13√3.

Do đó MC.MD ≥ 3√

3MA.MB.

Bài toán 1.46. (IMO, shorlist 1996) Cho ABCD là tứ giác lồi. Kí

hiệu RA, RB, RC và RD là bán kính đường tròn ngoại tiếp tam giác

DAB,ABC,BCD và CDA tương ứng. Chứng minh rằng RA + RC >

RB +RD nếu và chỉ nếu A+ C > B + D.

Giải. Gọi X = AC ∩BD. Trong hai góc AXB và AXD có ít nhất một

góc lớn hơn hay bằng 900. Ta giả sử AXB ≥ 900. Đặt α = CAB, β =

ABD, α′ = BDC, β′ = DCA. Các góc này đều nhọn và α+ β = α′+ β′.

Hơn nữa RA =AD

2 sin β,RB =

BC

2 sinα,RC =

BC

2 sinα′, RD =

AD

2 sin β′.

Bây giờ, ta xét 3 trường hợp sau:

1. Nếu B + D = 1800 thì ABCD là tứ giác nội tiếp đường tròn và dễ

có RA +RC = RB +RD.

2. Nếu B + D > 1800 thì D nằm trong đường tròn ngoại tiếp tam

giác ABC. Điều này dẫn đến β > β′, α < α′, suy ra RA < RD và

RC < RB. Do đó RA +RC < RB +RD.

3. Nếu B + D < 1800 thì D nằm ngoài đường tròn ngoại tiếp tam

giác ABC. Điều này dẫn đến β < β′, α > α′, suy ra RA > RD và

RC > RB. Do đó RA +RC > RB +RD.

www.VNMATH.com

Page 50: Bất đẳng thức hình học

48

Chương 2Bất đẳng thức Ptolemy và các mởrộng

Chương này trình bày định lí Ptolemy, bất đẳng thức Ptolemy và các

bài toán áp dụng. Ngoài ra còn trình bày một số mở rộng của bất đẳng

thức này trong tứ giác và mở rộng trong tứ diện. Nội dung chủ yếu được

hình thành từ các tài liệu [1], [5], [8], [10] và [12].

2.1. Định lí Ptolemy

Định lý Ptolemy hay Đẳng thức Ptolemy miêu tả quan hệ giữa độ

dài bốn cạnh và hai đường chéo của một tứ giác nội tiếp đường tròn.

Định lý này mang tên nhà toán học và thiên văn học người Hy Lạp cổ

đại Ptolemy.

Định lý 2.1. Cho ABCD là tứ giác nội tiếp đường tròn. Khi đó

AC.BD = AB.CD + AD.BC. (2.1)

Chứng minh.

Hình 2.1

Lấy điểm M thuộc đường chéo BD sao

cho MCD = BCA. Khi đó, dễ thấy các

tam giác ABC và DMC đồng dạng nhau,

suy raCD

MD=CA

AB⇔ CD.AB = CA.MD.

Cũng dễ chứng minh rằng hai tam giác

BCM và ACD đồng dạng, do đó ta cóBC

BM=AC

AD⇔ BC.AD = AC.BM. Cộng

theo vế hai bất đẳng thức trên, ta thu được

CD.AB +BC.AD = AC.BM + CA.MD

= AC.BD.

www.VNMATH.com

Page 51: Bất đẳng thức hình học

49

Bài toán 2.1. Cho tam giác ABC nội tiếp đường tròn (O). Đường phân

giác của BAC cắt (O) tại D khác A. Gọi K,L lần lượt là hình chiếu của

B,C trên AD. Chứng minh rằng AD ≥ BK + CL.

Giải.

Hình 2.2

Do ∆BKA,∆CLA là các tam

giác vuông nên ta có BK + CL =

(AB + AC) sinA

2. Mặt khác, tứ giác

ABDC nội tiếp nên theo đẳng thức

ptolemy ta có AB.DC + AC.BD =

AD.BC mà DC = BD =BC

2 cos D2

=

BC

2 cos A2

nên AB + AC = 2AD. cos A2 .

Suy ra BK + CL = AD. sinA ≤ AD.

Bài toán 2.2. (Tạp chí THTT số 261) Cho tam giác ABC. Các đường

phân giác trong xuất phát từ A,B,C cắt đường tròn ngoại tiếp tam giác

ABC tại A1, B1, C1 tương ứng. Chứng minh rằng

AA1.BB1.CC1 ≥ 16R2r.

Giải.

Hình 2.3

Áp dụng định lí Ptolemy cho tứ giác

ABA1C ta có AA1.BC = AB.CA1 +

AC.BA1 hay AA1.a = c.CA1 + b.BA1.

Do AA1 là phân giác góc BAC cho nên

A1 là điểm chính giữa cung_

BC và do đó

A1B = A1C. Suy ra a.AA1 = (b + c)A1B.

Mặt khác theo định lí hàm số sin, ta cóA1B

sin A1AB= 2R, suy ra A1B = 2R sin A

2 .

Vậy AA1 = b+ca .2R sin A

2 . Tương tự BB1 =c+ab .2R sin B

2 , CC1 = a+bc .2R sin C

2 . Để ý

rằng r = 4R. sin A2 . sin

B2 . sin

C2 và (a + b)(b + c)(c + a) ≥ 8abc. Suy

ra AA1.BB1.CC1 =8R3(a+ b)(b+ c)(c+ a)

abcsin A

2 sin B2 sin C

2 ≥ 16R2r.

www.VNMATH.com

Page 52: Bất đẳng thức hình học

50

Bài toán 2.3. Cho tam giác ABC và điểm M bất kỳ nằm trên đường

tròn (O) ngoại tiếp ABC. Chứng minh rằng

MA

a+MB

b+MC

c≥ min

a

b+b

a;b

c+c

b;c

a+a

c

≥ 2.

Giải.

Hình 2.4

Hiển nhiên ta có bất đẳng thức

phải. Ta chứng minh bất đẳng thức

trái bằng phương pháp phản chứng.

Giả sử MAa + MB

b + MCc <

minab + b

a ;bc + c

b ;ca + a

c

. Không

giảm tính tổng quát, giả sử M nằm

trên cung_

BC không chứa A.

Do tứ giác ABMC nội tiếp, nên

áp dụng đẳng thức Ptolemy ta có

MA.BC = MC.AB + MB.AC. Suy

ra MA = MB.b+MC.ca . Khi đó

MA

a+MB

b+MC

c=MB. ba +MC. ca

a+MB.ab +MC.ac

a

=MB.

(ab + b

a

)+MC

(ac + c

a

)a

≥MB.

(ab + b

a

)+MC

(ac + c

a

)MB +MC

. (2.2)

Mặt khác, MAa + MB

b + MCc < min

ab + b

a ;bc + c

b ;ca + a

c

suy ra MA

a + MBb + MC

c < ab + b

a và MAa + MB

b + MCc < a

c + ca .

Từ đó

MA

a+MB

b+MC

c=

MB.

(MA

a+MB

b+MC

c

)+MC.

(MA

a+MB

b+MC

c

)MB +MC

<

MB.

(a

b+b

a

)+MC

(ac+c

a

)MB +MC

. (2.3)

(2.2) và (2.3) mâu thuẫn, do đó giả sử là sai. Vậy được bất đẳng thức

cần chứng minh.

Bài toán 2.4. (Ailen, 1995) Cho ba điểm A,X,D thẳng hàng với X

nằm giữa A và D. Gọi B là điểm thỏa mãn ABX = 1200 và C là điểm

www.VNMATH.com

Page 53: Bất đẳng thức hình học

51

nằm giữa B và X. Chứng minh rằng

2AD ≥√

3 (AB +BC + CD) .

Giải.

Hình 2.5

Dựng tam giác đều AXO sao cho

B và O khác phía với AX. Dễ thấy

ABXO là tứ giác nội tiếp đường tròn

đường kính2√3AX. Áp dụng định lý

Ptolemy cho tứ giác này, ta được

AB.OX +BX.AO = AX.BO.

Vì OX = AO = AX nên

AX (AB +BX) = AX.BO ≤ AX.2√3AX. Suy ra 2AX ≥

√3 (AB +BX). Do đó 2AD = 2 (AX +XD) ≥

√3 (AB +BX)

+2XD ≥√

3 (AB +BC + CX) +√

3XD ≥√

3 (AB +BC + CD) .

Bài toán 2.5. (Vô địch Toán lần thứ 2, Hồng Kông, 1999) Gọi I và O

lần lượt là tâm các đường tròn nội và ngoại tiếp tam giác ABC. Giả

sử tam giác ABC không đều. Chứng minh AIO ≤ 900 nếu và chỉ nếu

2BC ≤ AB + CA.

Giải.

Hình 2.6

Kéo dài AI cắt đường tròn ngoại tiếp

∆ABC tại D. Vì AD là phân giác của

BAC nên DB = DC. Mặt khác, ta có

DIB = DAB + ABI = DCB + IBC =

DBC + IBC = DBI Suy ra DB = DI =

DC. Áp dụng đẳng thức Ptolemy cho tứ

giác ABCD, ta có AD.BC = AB.CD +

AC.BD = DI (AB + AC).

Do đó AIO ≤ 900 ⇔ AI ≥ ID ⇔ 2 ≤AD

DI=AB + AC

BC⇔ 2BC ≤ AB + AC.

www.VNMATH.com

Page 54: Bất đẳng thức hình học

52

Bài toán 2.6. (Chọn đội tuyển Singapore, 2002) Cho ABCD là tứ giác

nội tiếp đường tròn. Chứng minh rằng

|AC −BD| ≤ |AB − CD| .

Giải.

Hình 2.7

Gọi E,F lần lượt là trung điểm của

AC,BD. Áp dụng định lí công thức đường

trung tuyến. Ta có

AB2 +BC2 + CD2 +DA2

=2BE2 +1

2AC2 + 2DE2 +

1

2AC2

=2

(2EF 2 +

1

2DB2

)+ AC2.

Vậy AC2 +BD2 + 4EF 2 = AB2 +BC2 +

CD2 +DA2. Áp dụng định lí Ptolemy cho

tứ giác ABCD, ta có AC.BD = AB.CD + AD.BC. Từ đó suy ra

(AC −BD)2 + 4EF 2 = (AB − CD)2 + (AD −BC)2 . (2.4)

Mặt khác, gọi M là trung điểm của AB, xét tam giác MEF , ta có

EF ≥ |MF −ME| ⇒ EF ≥ 1

2|AD −BC|. Do đó

4EF 2 ≥ (AD −BC)2 . (2.5)

Từ (2.4) và (2.5) thu được bất đẳng thức cần chứng minh.

Bài toán 2.7. (IMO shorlist, 1996) Cho tam giác ABC và O là một điểm

bất kỳ trong tam giác. Gọi (H), (I), (K) theo thứ tự là các đường tròn

ngoại tiếp các tam giác BOC,COA,AOB. Dựng A′ = AO ∩ (H), B′ =

BO ∩ (I), C ′ = CO ∩ (K). Chứng minh các bất đẳng thức sau

a) OA′.OB′.OC ′ ≥ 8.OA.OB.OC.

b)OA′

OA+OB′

OB+OC ′

OC≥ 6.

Giải.

www.VNMATH.com

Page 55: Bất đẳng thức hình học

53

Hình 2.8

Đặt

x = sin BOC ′ = sin COB′,

y = sin COA′ = sin AOC ′,

z = sin AOB′ = sin BOA′.

Khi đó ta dễ dàng chứng minh được

CA′

BC=y

x,

BA′

BC=z

x. (2.6)

Mặt khác, vì tứ giác A′OBC nội tiếp

nên theo đẳng thức Ptolemy ta có

OA′ =OB.CA′ +OC.BA′

BC. (2.7)

Cộng theo vế (2.6) và (2.7), sau đó áp dụng bất đẳng thức AG − GM ,

ta được

OA′ =y

x.OB +

z

x.OC ≥

√yz.OB.OC

x. (2.8)

Tương tự, ta có

OB′ =z

y.OC +

x

y.OA ≥

√zx.OC.OA

y, (2.9)

OC ′ =x

z.OA+

y

z.OB ≥

√xy.OA.OB

z. (2.10)

Nhân theo vế ba bất đẳng thức (2.8), (2.9) và (2.10) ta chứng minh được

phần a.

Mặt khác, ta có OA′

OA + OB′

OB + OC ′

OC =(yx .OBOA + z

x .OCOA

)+(zy .OCOB + x

yOAOB

)+(

xz .OAOC + y

zOBOC

)=(yx .OBOA + x

y .OAOB

)+(zy .OCOB + y

zOBOC

)+(xz .OAOC + z

xOCOA

)≥ 6.

Phần b được chứng minh.

2.2. Bất đẳng thức Ptolemy

Định lý 2.2. (Bất đẳng thức Ptolemy) Với 4 điểm A,B,C,D bất kỳ

trên mặt phẳng, ta có

AB.CD + AD.BC ≥ AC.BD. (2.11)

Chứng minh.

www.VNMATH.com

Page 56: Bất đẳng thức hình học

54

Hình 2.9

Dựng điểm E sao cho tam giác

BCD đồng dạng với tam giác BEA.

Khi đó theo tính chất của tam giác

đồng dạng, ta cóBA

EA=BD

CD. Suy ra

BA.CD = EA.BD. (2.12)

Mặt khác, hai tam giác EBC và ABD

cũng đồng dạng do cóBA

BD=BE

BCvà EBC = ABD. Từ đó

EC

BC=AD

BD.

Suy ra

AD.BC = EC.BD. (2.13)

Cộng theo vế (2.12) và (2.13) ta suy ra

AD.CD + AD.BC = BD. (EA+ EC) ≥ BD.AC.

Dấu bằng xảy ra khi và chỉ khi A,E,C thẳng hàng, tức là khi A và D

cùng nhìn BC dưới một góc bằng nhau, và khi đó tứ giác ABCD nội

tiếp trong một đường tròn.

Bất đẳng thức Ptolemy có nhiều ứng dụng trong các bài toán bất

đẳng thức hình học, đặc biệt là trong các bài toán so sánh độ dài các

đoạn thẳng. Trước hết ta xét ứng dụng bất đẳng thức Ptolemy trong

việc chứng minh một số kết quả kinh điển của hình học phẳng.

Bài toán 2.8. (Bất đẳng thức Erdos - Modell) Cho tam giác ABC. M

là một điểm bất kỳ nằm trong tam giác. Đặt R1 = MA,R2 = MB,R3 =

MC; r1, r2, r3 lần lượt là khoảng cách từ M đến BC,CA,AB. Khi đó ta

có bất đẳng thức

R1 +R2 +R3 ≥ 2 (r1 + r2 + r3) .

Giải. Nối dài AM cắt đường tròn nội tiếp tam giác ABC tại A′. Áp

dụng định lí Ptolemy cho tứ giác nội tiếp ABA′C, ta có

AB.CA′ + AC.BA′ = BC.AA′.

Hạ A′D vuông góc với AC và A′E vuông góc với AB thì rõ ràng A′B ≥A′E,A′C ≥ A′D. Do đó a.AA′ ≥ c.A′D+b.A′E hay 1 ≥ A′D

AA′.c

a+A′E

AA′.b

a.

www.VNMATH.com

Page 57: Bất đẳng thức hình học

55

Hình 2.10

NhưngA′D

AA′=

r2R1

vàA′E

AA′=

r3R1

nên

từ đó R1 ≥ r2.c

a+ r3.

b

a. Tương tự ta

có các đánh giá cho R2 và R3. Do đó

R1+R2+R3 ≥ r1

(c

b+b

c

)+r2

( ca

+a

c

)+

r3

(b

a+a

b

)≥ 2 (r1 + r2 + r3). Dấu bằng

xảy ra khi và chỉ khi tam giác ABC đều và

M trùng với tâm O của tam giác.

Bài toán 2.9. (Điểm Torricelli) Cho tam giác ABC. Hãy tìm điểm M

trong mặt phẳng tam giác ABC sao cho MA + MB + MC đạt giá trị

nhỏ nhất. Điểm M tìm được được gọi là điểm Torricelli của tam giác

ABC.

Giải. Trên cạnh BC, dựng ra phía ngoài tam giác đều BCA′. Áp dụng

bất đẳng thức Ptolemy cho tứ giác MBA′C ta có

BM.CA′ + CM.BA′ ≥ BC.MA′.

Hình 2.11

Do CA′ = BA′ = BC nên ta

được BM + CM ≥ MA′. Suy ra

AM+BM+CM ≥MA+MA′ ≥ AA′

tức là MA + MB + MC ≥ AA′ (là

hằng số)

Dấu bằng xảy ra khi và chỉ khi

i) Tứ giác BMCA′ nội tiếp.

ii) M nằm giữa A và A′.

Dễ thấy ta có thể tìm được điểm M

thỏa mãn cả hai điều kiện này khi và

chỉ khi tất cả các góc của tam giác ABC đều không lớn hơn 1200.

Trường hợp tam giác ABC có một góc lớn hơn 1200, chẳng hạn A > 1200

thì điểm M cần tìm chính là điểm A (xem lời giải ở bài toán 1.23)

Nhận xét 2.1.

www.VNMATH.com

Page 58: Bất đẳng thức hình học

56

1) Phương pháp trên có thể áp dụng cho bài toán tổng quát hơn: "Cho

tam giác ABC và các số thực dương m,n, p. Hãy tìm điểm M trong mặt

phẳng tam giác sao cho m.MA + n.MB + p.MC đạt giá trị nhỏ nhất"

đạt giá trị nhỏ nhất.

2) Việc dựng tam giác đều BCA′ ra phía ngoài trong lời giải bài toán

Torricenlli chính là một cách làm mẫu mực để áp dụng được bất đẳng

thức Ptolemy. Ý tưởng chung là: Để dánh giá tổng p.MA+ q.MB, ta có

thể dựng điểm N sao cho p.NA = q.NB. Sau đó áp dụng bất đẳng thức

ptolemy cho tứ giác ANMB ta được NA.MB + NB.MA ≥ AB.MN .

Từ đó

p.NA.MB + p.NB.MA ≥ p.AB.MN

⇔q.NB.MB + p.NB.MA ≥ p.AB.MN

⇔p.MA+ q.MB ≥ p.AB.MN

NB.

Chú ý rằng điểm N là cố định, như thế p.MA+ q.MB đã được đánh

giá thông qua MN . Ý tưởng này là chìa khóa để giải hàng loạt các bài

toán cực trị hình học.

Bài toán 2.10. Cho điểm M nằm trong góc nhọn xOy. Hai điểm A,B

lần lượt thay đổi trên Ox,Oy sao cho 2OA = 3OB. Tìm vị trí của A,B

sao cho 2MA+ 3MB đạt giá trị nhỏ nhất.

Giải. Áp dụng bất đẳng thức Ptolemy cho tứ giác OAMB, ta có

OA.MB +OB.MA ≥ OM.AB.

Từ đó 2OA.OB + 2OB.MA ≥ 2OM.AB ⇔ 3OB.MB + 2OB.MA ≥2OM.AB ⇔ 2MA+ 3MB ≥ 2OM.

(ABOB

). Vì tam giác OAB luôn đồng

dạng với chính nó nên ABOB là một đại lượng không đổi. Từ đó suy ra

2MA + 3MB đạt giá trị nhỏ nhất bằng 2OM.(ABOB

). Dấu bằng xảy ra

khi và chỉ khi tứ giác OAMB nội tiếp.

Bài toán 2.11. (IMO, 2001) Cho tam giác ABC với trọng tâm G và

độ dài các cạnh a = BC, b = CA, c = AB. Tìm điểm P trên mặt phẳng

tam giác sao cho đại lượng AP.AG + BP.BG + CP.CG đạt giá trị nhỏ

nhất và tìm giá trị nhỏ nhất đó theo a, b, c.

www.VNMATH.com

Page 59: Bất đẳng thức hình học

57

Giải. Vẽ đường tròn ngoại tiếp tam giác BGC. Kéo dài trung tuyến AL

cắt đường tròn này tại K. Gọi M,N là trung điểm các cạnh AC,AB

tương ứng. Áp dụng định lí hàm số sin cho các tam giác BGL,CGL, ta

BG

sin BLG=

BL

sin BGK,

CG

sin CLG=

CL

sin CGK. (2.14)

Hình 2.12

Nhưng L là trung điểm của BC

và sin BLG = sin CLG, nên từ (2.14)

ta có BGCG = sin CGK

sin BGK. Ta có BK =

2R. sin BGK,CK = 2R. sin CGK,

trong đó R là bán kính đường tròn

ngoại tiếp tam giácBCG. Do đó CKBK =

BGCG hay BG

CK = CGBK . Tương tự AG

BG =sin BGN

sin AGN= sin BGN

sin CGK. Hơn nữa BC =

2R. sin BGC = 2R. sin BGN . Từ đóBGCK = AG

BC . Như vậy BGCK = CG

BK = AGBC .

Bây giờ, áp dụng bất đẳng bất đẳng

thức Ptolemy cho tứ giác PBKC ta có PK.BC ≤ BP.CK + CP.BK.

Từ đó PK.AG ≤ BP.BG+CP.CG. Suy ra (AP + PK)AG ≤ AP.AG+

BP.BG+CP.CG và cuối cùng AK.AG ≤ AP.AG+BP.BG+CP.CG.

Dấu bằng xảy ra khi và chỉ khi (1) P nằm trên cung_

BGC (để có đẳng

thức ở bất đẳng thức Ptolemy) và (2) P nằm trên AK (để có đẳng thức

trong bất đẳng thức tam giác). Do đó giá trị này đạt được khi P ≡ G.

Dễ dàng tính được rằng AG2 +BG2 + CG2 =

(a2 + b2 + c2

)3

.

Nhận xét 2.2. Có thể thấy đây là trường hợp đặc biệt của bài toán

Torricelli tổng quát. Chú ý rằng từ ba đoạn AG,BG,CG có thể dựng

được một tam giác ∆, ta chỉ cần dựng tam giác BCK đồng dạng với

tam giác ∆ là được. Cách giải nêu trên chỉ ra cách dựng tường minh cho

điểm K.

Bài toán 2.12. Cho ABCD là hình vuông và P là một điểm tùy ý

trong mặt phẳng. Trong số các khoảng cách PA, PB, PCvà PD, gọi M

www.VNMATH.com

Page 60: Bất đẳng thức hình học

58

là giá trị lớn nhất và m là giá trị nhỏ nhất. Chứng minh rằng

PA+ PB + PC + PD ≥(

1 +√

2)M +m.

Đẳng thức xảy ra khi P nằm trên đường tròn ngoại tiếp hình vuông.

Giải. Không giảm tính tổng quát, giả sử M = PD, thì dễ dàng thấy

m = PB. Áp dụng bất đẳng thức Ptolemy cho tứ giác PADC, ta được

PA.CD + PC.AD ≥ PD.AC

với đẳng thức xảy ra khi và chỉ khi A,P,C,D nằm trên một đường tròn.

Vì AD = CD = AC/√

2, nên PA+ PC ≥√

2PD. Do đó

PA+ PB + PC + PD ≥(

1 +√

2)PD + PB =

(1 +√

2)M +m.

Đẳng thức xảy ra khi P nằm trên đường tròn ngoại tiếp hình vuông.

Bài toán 2.13. (Tạp chí THTT số 259) Cho thất giác đều

A1A2A3A4A5A6A7 và điểm M bất kì. Chứng minh rằng

MA1 +MA3 +MA5 +MA7 ≥MA2 +MA4 +MA6.

Giải. Đặt A1A2 = a,A1A3 = b, A1A4 = c. Áp dụng bất đẳng thức

Ptolemy cho tứ giác A1A2A3M và A5A6A7M , ta được

(MA1 +MA3) a ≥MA2.b, (2.15)

(MA5 +MA7) a ≥MA6.b. (2.16)

Cộng theo vế 2 bất đẳng thức trên có

(MA1 +MA3 +MA5 +MA7) a ≥ (MA2 +MA6) .b. (2.17)

Tiếp tục áp dụng bất đẳng thức Ptolemy cho tứ giác A2A4A6M , được

(MA2 +MA6) b ≥MA4.c. (2.18)

Từ (2.17) và (2.18) có

(MA1 +MA3 +MA5 +MA7) a ≥MA4.c. (2.19)

www.VNMATH.com

Page 61: Bất đẳng thức hình học

59

Từ (2.17) và (2.19) có

(MA1 +MA3 +MA5 +MA7)(ab+a

c

)≥MA2 +MA4 +MA6. (2.20)

Áp dụng định lí Ptolemy cho tứ giác nội tiếp A1A3A4A5 có ab+ac = bc,

suy ra(ab + a

c

)= 1. Thay vào (2.20) được

MA1 +MA3 +MA5 +MA7 ≥MA2 +MA4 +MA6.

Đẳng thức xảy ra khi các đẳng thức ở (2.15), (2.16) và (2.18) xảy ra

nghĩa là M thuộc cung nhỏ_

A1A7 của đường tròn ngoại tiếp đa giác

A1A2A3A4A5A6A7.

Bài toán 2.14. (Olympic Toán học 30/4, bài đề nghị, 2000) Chứng tỏ

rằng trong tam giác ABC ta có

ab

mamb+

bc

mbmc+

ca

mcma≥ 4,

với a, b, c là độ dài ba cạnh và ma,mb,mc là độ dài các đường trung

tuyến tương ứng của tam giác đó.

Giải.

Hình 2.13

Điều phải chứng minh tương

đương với

abmc + bcma + camb ≥ 4mambmc.

Áp dụng bất đẳng thức Ptolemy cho

tứ giác CMGN , ta có

GC.MN ≤ GN.MC +GM.NC,

từ đó2

3mc

c

2≤ 1

3mba

2+

1

3ma

b

2hay 2mcc ≤ mba+mab. Suy ra

2mcc2 ≤ acmb + bcma. (2.21)

Mặt khác, áp dụng bất đẳng thức Ptolemy cho tứ giác ABMN ta được

AM.BN ≤ AN.BM + AB.MN , suy ra mamb ≤ab

4+c2

2, tức là

4mambmc ≤ abmc + 2c2mc. (2.22)

Từ (2.21) và (2.22) suy ra điều phải chứng minh.

www.VNMATH.com

Page 62: Bất đẳng thức hình học

60

Bài toán 2.15. (Trung Quốc, 1988) Cho tứ giác ABCD nội tiếp đường

tròn (O,R). Các tia AB,BC,CD,DA cắt đường tròn (O, 2R) lần lượt

tại các điểm A′, B′, C ′, D′ tương ứng. Chứng minh rằng

A′B′ +B′C ′ + C ′D′ +D′A′ ≥ 2 (AB +BC + CD +DA) .

Khi nào dấu đẳng thức xảy ra ?

Giải. Áp dụng bất đẳng thức Ptolemy cho tứ giác OAD′A′, ta có

OD′.AA′ ≤ OA.A′D′ + OA′.AD′. Suy ra 2AA′ ≤ A′D′ + 2AD′ hay

2AB + 2BA′ ≤ A′D′ + 2AD′. Tương tự với ba bất đẳng thức khác, ta

đi đến

2 (AB +BC + CA+ AD) + 2 (BA′ + CB′ +DC ′ + AD′)

≤ (A′B′ +B′C ′ + C ′D′ +D′A′) + 2 (BA′ + CB′ +DC ′ + AD′) .

Từ đó suy ra bất đẳng thức cần chứng minh.

Hình 2.14

Đẳng thức xảy ra khi tất cả các tứ giác

OAD′A′, OBA′B′, OCB′C ′, ODC ′D′ đều

nội tiếp. Nếu OAD′A′ nội tiếp thì OAA′ =

OD′A′ = OA′D′ = 1800 − OAD′ = OAD.

Do đó OA là phân giác của DAB. Tương

tự, điểm O nằm trên các phân giác khác

của ABCD. Suy ra OAB = OAD =

ODA = ODC = OBA. Từ đó, AOB =

BOC = COD = DOA. Vậy A,B,C,D

cách đều nhau trên đường tròn, nghĩa là

ABCD là hình vuông. Ngược lại, nếu ABCD là một hình vuông thì

hiển nhiên đẳng thức xảy ra.

Tóm lại, đẳng thức xảy ra khi và chỉ khi ABCD là một hình vuông.

Bài toán 2.16. (IMO shorlist, 1997) Cho lục giác lồi ABCDEF có

AB = BC,CD = DE,EF = FA. Chứng minh rằng

BC

BE+DE

DA+FA

FC≥ 3

2.

Khi nào dấu đẳng thức xảy ra ?

www.VNMATH.com

Page 63: Bất đẳng thức hình học

61

Giải. Áp dụng bất đẳng thức Ptolemy cho tứ giác ABCE, ta

được EA.BC + EC.AB ≥ BE.AC. Sử dụng AB = BC, ta được

BC (EA+ EC) ≥ BE.AC, hay BCBE ≥

ACEA+EC .

Tương tự, áp dụng bất đẳng thức Ptolemy cho các tứ giác

ACDE,EFAC và sử dụng CD = DE,EF = FA, ta được DEDA ≥

CEAC+AE ,

FAFC ≥

EACE+CA . Cộng theo vế ba bất đẳng thức trên và áp dụng bất đẳng

thức Nesbitt ta thu được bất đẳng thức cần chứng minh.

Để có dấu bằng ta phải có dấu bằng xảy ra ở ba bất đẳng thức

Ptolemy và ở bất đẳng thức Nesbitt. Dấu bằng xảy ra ở bất đẳng thức

Nesbitt khi tam giác EAC đều, như thế EAC = 600. Vì ABCE là tứ

giác nội tiếp nên góc B phải bằng 1200. Tương tự, góc D và F cũng

bằng 1200. Mặt khác, thấy rằng các tam giác ABC,CDE,EFA bằng

nhau (g.c.g). Như vậy, lục giác có tất cả các cạnh đều bằng nhau và tất

cả các góc bằng 1200, vậy nó là lục giác đều. Ngược lại, hiển nhiên là với

lục giác đều, ta có dấu bằng xảy ra.

Bài toán 2.17. (IMO, 1995) Cho ABCDEF là lục giác sao cho AB =

BC = CD, DE = EF = FA và BCD = EFA = 600. G và H là hai

điểm tùy ý. Chứng minh rằng

AG+BG+GH +DH + EH ≥ CF.

Giải.

Hình 2.15

Từ giả thiết ta có BCD và

AEF là các tam giác đều. Lấy C ′

và F ′ lần lượt đối xứng với C và

F qua BE. Khi đó CF = C ′F ′

và ta được DEF ′ và ABC ′ là các

tam giác đều. Áp dụng bất đẳng

thức Ptolemy cho tứ giác F ′DHE

và C ′AGB, ta có

F ′H.DE ≤ F ′E.DH + F ′D.EH

hay F ′H ≤ DH + EH

và C ′G.AB ≤ C ′A.BG+ C ′B.AG

www.VNMATH.com

Page 64: Bất đẳng thức hình học

62

hay C ′G ≤ BG+ AG. Do đó

AG+BG+GH +DH + EH ≥ C ′G+GH +HF ′ ≥ C ′F ′ = CF.

Bài toán 2.18. (New Zealand, 1998) Cho M,A1, A2, · · · , An (n ≥ 3) là

các điểm phân biệt trong mặt phẳng thỏa mãn A1A2 = A2A3 = · · · =

An−1An = AnA1. Chứng minh rằng

1

MA1.MA2+

1

MA2.MA3+

1

MAn−1.MAn≥ 1

MA1.MAn.

Xác định dấu đẳng thức xảy ra khi nào?

Giải. Với mỗi số nguyên k giữa 1 và n − 1 , áp dụng bất đẳng thức

Ptolemy cho tứ giác MA1AkAk+1 ta được

MA1.AkAk+1 + A1Ak.MAk+1 ≥ A1Ak+1.MAk, (2.23)

(với k = 1 suy biến thành đẳng thức). Chia cả hai vế bất đẳng thức trên

cho MA1.MAk.MAk+1 ta được

AkAk+1

MAk.MAk+1≥ A1Ak+1

MA1.MAk+1− A1Ak

MA1.MAk.

Lấy tổng n− 1 bất đẳng thức này, ta được

A1A2

MA1.MA2+

A2A3

MA2.MA3+ · · ·+ An−1An

MAn−1.MAn≥

A1An

MA1.MAn− A1A1

MA1.MA1=

A1An

MA1.MAn.

Vì A1A2 = A2A3 = · · · = An−1An = AnA1 nên ta được bất đẳng thức

cần chứng minh.

Đẳng thức xảy ra khi và chỉ khi trong (2.23) dấu đẳng thức xảy ra với

tất cả các giá trị k = 1, 2, · · · , n− 1. Điều này xảy ra khi và chỉ khi theo

thứ tự A1, Ak, Ak+1,M nằm trên một đường tròn. Trong trường hợp này

A1A2 · · · , An là một đa giác đều và M thuộc cung_

AkAn ngắn nhất.

www.VNMATH.com

Page 65: Bất đẳng thức hình học

63

2.3. Định lí Bretschneider

Định lý 2.3. Cho tứ giác ABCD, gọi a, b, c, d lần lượt là độ dài các

cạnh AB,BC,CD.DA và m,n là độ dài các đường chéo AC,BD. Khi

đó ta có

m2n2 = a2c2 + b2d2 − 2abcd. cos(A+ C). (2.24)

Chứng minh.

Hình 2.16

Trên cạnh AB ra phía ngoài dựng tam

giác BAK đồng dạng với tam giác ACD,

trong đó BAK = DCA, ABK = CAD,

còn trên cạnh AD ra phía ngoài dựng tam

giác DMA đồng dạng với tam giác ABC,

trong đó DAM = BCA, ADM = CAB.

Từ các tam giác đồng dạng này suy ra

AK =ac

m,AM =

bd

m,KB = DM =

ad

m.

Ngoài ra, KBD+ MDB = CAD+ ABD+ BDA+ CAB = 1800, nghĩa

là tứ giác KBDM là hình bình hành. Do đó KM = BD = n.

Mặt khác KAM = A + C. Áp dụng định lí hàm số cosin cho tam giác

KAM , ta có m2n2 = a2c2 + b2d2 − 2abcd. cos(A+ C).

Chú ý 2.1. Vì 0 < φ < 2π nên −1 ≤ cosφ < 1, do đó từ kết quả

trên suy ra |ac− bd| < mn ≤ ac + bd. Dấu "=" xảy ra khi và chỉ khi

cosφ = −1⇔ φ = π ⇔ ABCD là tứ giác nội tiếp.

Như vậy định lí Ptolemy và cả bất đẳng thức Ptolemy đều là hệ quả

của định lí Bretschneider.

2.4. Định lí Casey

Định lý 2.4. Cho tứ giác ABCD nội tiếp đường tròn (O). Bốn đường

tròn α, β, γ, δ tiếp xúc với (O) lần lượt tại A,B,C,D. Gọi a là độ dài

đoạn tiếp tuyến chung của hai đường tròn α, β, với α, β cùng tiếp xúc

ngoài hoặc cùng tiếp xúc trong với (O). Nếu hai đường tròn α, β có một

www.VNMATH.com

Page 66: Bất đẳng thức hình học

64

đường tròn tiếp xúc ngoài (O) và một đường tròn tiếp xúc trong với (O)

thì a là độ dài tiếp tuyến chung trong của α, β. Các đại lượng b, c, d, x, y

là độ dài các tiếp tuyến chung của các cặp đường tròn xác định tương

tự (hình 2.17). Khi đó ta có

xy = ac+ bd. (2.25)

Chứng minh.

Hình 2.17

Ta chứng minh cho trường hợp

α, β, γ, δ đều tiếp xúc ngoài với (C).

Các trường hợp còn lại được chứng

minh tương tự.

Gọi Ra, Rb, Rc, Rd, R lần lượt là bán

kính của các đường tròn α, β, γ, δ, (O).

Đặt A′OB′ = AOB = α, ta có a2 =

A′B′2−(Ra−Rb)2; A′B′2 = (R+Ra)

2+

(R +Rb)2 − 2(R +Ra)(R +Rb) cosα.

Suy ra

a2 = 2(R +Ra)(R +Rb)(1− cosα), (2.26)

AB2 = 2R2 − 2R2 cosα = 2R2(1− cosα). (2.27)

Từ (2.26) và (2.27) suy ra a =

√(R +Ra)(R +Rb)

RAB. Chứng minh

tương tự ta có

b =

√(R +Rb)(R +Rc)

RBC; c =

√(R +Rc)(R +Rd)

RCD;

d =

√(R +Rd)(R +Ra)

RDA; x =

√(R +Ra)(R +Rc)

RAC;

y =

√(R +Rb)(R +Rd)

RBD.

Áp dụng định lí Ptolemy cho tứ giác ABCD ta thu được (2.25)

Nhận xét 2.3. Đẳng thức (2.25) vẫn đúng khi các bán kính Ra =

0, Rb = 0, Rc = 0, Rd = 0; Khi Ra = Rb = Rc = Rd = 0 thì định lý

www.VNMATH.com

Page 67: Bất đẳng thức hình học

65

Casey trở thành định lí Ptolemy. Phải chăng định lí Casey được suy ra

bằng cách "nở" các điểm A,B,C,D.

Định lí Casey là một mở rộng của định lí Ptolemy, nó sẽ giúp ích cho

việc giải một số bài toán liên quan đến đường tròn tiếp xúc nhau, độ

dài tiếp tuyến và dây cung. Sau đây là một số bài toán áp dụng định lý

Casey.

Bài toán 2.19. Cho hai đường tròn (O1) và (O2) tiếp xúc ngoài nhau

tại I và cùng tiếp xúc trong với đường tròn O. Một tiếp tuyến chung

ngoài của O1 và (O2) cắt O tại B và C, trong khi đó tiếp tuyến chung

trong của chúng cắt O tại điểm A cùng phía với I. Chứng minh rằng I

là tâm đường tròn nội tiếp tam giác ABC.

Giải.

Hình 2.18

Giả sử BC tiếp xúc (O1) tại X và

(O2) tại Y và cắt AI cắt BC tại D.

Đặt BC = a, CA = b, AB = c, BX =

x,CY = y, AI = z,DX = DI =

DY = u. Áp dụng định lý Casey cho

bộ 4 đường tròn (A), (O1), (B), (C) và

(A), (O2), (C), (B) ta có

az + bx = c (2u+ y) , (2.28)

az + cy = b (2u+ x) . (2.29)

Lấy (2.28)trừ (2.29) theo vế, ta được

bx− cy = u (c− b). Từ đóx+ u

y + u=c

b,

tức làBD

CD=AB

AC. Suy ra AD là phân giác của góc A và BD =

ac

b+ c.

Mặt khác, cộng hai vế (2.28) và (2.29) theo vế, ta được az = u (b+ c).

Suy raz

u=b+ c

a, tức là

AI

ID=

AB + AC

BD +DC=AB

BD, do đó BI là phân

giác của góc B. Bài toán được chứng minh.

Bài toán 2.20. Cho tam giác ABC nội tiếp đường tròn (O,R) và

BAC = 600. Đường tròn (O′, R′) tiếp xúc trong với (O,R) và tiếp xúc

với hai cạnh AB,AC. Chứng minh rằng R′ ≤ 2

3R.

www.VNMATH.com

Page 68: Bất đẳng thức hình học

66

Giải. Gọi X, Y lần lượt là tiếp điểm của AB,AC với đường tròn (O′, R′).

Đặt l = AX = AY = XY (∆ABC cân có BAC = 600). Đường tròn

(O,R) tiếp xúc trong với các đường tròn (A), (C), (O′R′), (B) nên áp

dụng định lí Casey ta có b(c− l) + c(b− l) = al.

Hình 2.19

Suy ra

l =2bc

a+ b+ c=bc

p=bc

Sr =

2bc

bc sinAr

=4√3r ≤ 2√

3R (vìR ≥ 2r). (2.30)

Mặt khác, lại có

R′ = O′X =XY

2 sin 600=

l√3

(2.31)

Từ (2.30) và (2.31) suy ra R′ ≤ 2

3R.

Bài toán 2.21. Cho ba đường tròn (C), (C1), (C2) trong đó hai đường

tròn (C1), (C2) tiếp xúc trong với (C) lần lượt tại B,C và (C1), (C2) tiếp

xúc ngoài với nhau tại D. Tiếp tuyến chung trong của (C1), (C2) cắt (C)

tại A,A1, AB cắt (C1) tại điểm thứ hai là K, AC cắt (C2) tại điểm thứ

hai là L. Chứng minh rằng1

DA+

1

DA1=

2

KL.

Giải. Đặt M = (C1) ∩ BC,N = (C2) ∩ BC. Vẽ tia tiếp tuyến BT với

hai đường tròn (C1), (C). Ta chứng minh KL là tiếp tuyến chung của

hai đường tròn (C1), (C2).

Hình 2.20

Thật vậy, ta có MKB = MBT =

CBT = CAB. Suy ra MK//AC. Do đó

MKL = KLA. Ta có ∆KAL ∼ ∆CAB

(vì AK.AB = AL.AC = AD2 ⇒ AK

AC=

AL

AB) suy ra KLA = CBA, do đó KL là

tiếp tuyến của (C1). Tương tự KL là tiếp

tuyến của (C2)

Áp dụng định lí Casey cho bộ 4 đường

tròn (A), (C1), (D), (C2) tiếp xúc trong với

www.VNMATH.com

Page 69: Bất đẳng thức hình học

67

đường tròn (C), ta có AD.A1D + AD.A1D = AA1.KL ⇒2

KL=

AA1

AD.A1D=

1

AD+

1

A1D.

Bài toán 2.22. Cho hai đường tròn (C1), (C2) tiếp xúc trong với đường

tròn (C) lần lượt tại M,N . Hai đường tròn (C1), (C2) cắt nhau hoặc

tiếp xúc ngoài với nhau. Trục đẳng phương của (C1), (C2) cắt (C) tại

hai điểm A,B. Đường thẳng AM,AN cắt (C1), (C2) tại điểm thứ hai là

K,L tương ứng. Chứng minh rằng AB ≥ 2KL. Đẳng thức xảy ra khi

nào ?

Giải.

Hình 2.21 Hình 2.22

Đặt C1(O1, R1), C2(O2, R2), C(O,R), P,Q = (C1)∩(C2) (hình 2.21),

nếu (C1) tiếp xúc (C2) thì P ≡ Q (hình 2.22).

AB là trục đẳng phương của (C1), (C2) suy ra AK.AM = AL.AN ⇒∆AKL ∼ ∆ANM ⇒ AKL = ANM =

1

2sđ

_

AM .

Ta có O1KM = O1MK = 900 − 1

2MOA = 900 − 1

2sđ

_

AM⇒

O1KL = 900, suy raKL là tiếp tuyến của (C1), ta cóO1K//OA//O2L⇒KL⊥O2L, suy ra KL là tiếp tuyến của (C2).

Áp dụng định lí Casey cho bộ 4 đường tròn đường tròn

(A), (C1), (B), (C2) tiếp xúc với đường tròn (C), ta được√AP.AQ.

√BP.BQ +

√AP.AQ.

√BP.BQ = AB.KL suy ra

www.VNMATH.com

Page 70: Bất đẳng thức hình học

68

AB.KL = 2√AP.AQ.

√BP.BQ ≤ 2

AP +BP

2

AQ+BQ

2=AB2

2.

Do đó AB ≥ 2KL.

Dấu đẳng thức xảy ra khi và chỉ khi (C1) tiếp xúc (C2) và AP = BP .

2.5. Mở rộng bất đẳng thức Ptolemy trong không gian

Định lý 2.5. Cho ABCD là tứ diện bất kì, ta có

AC.BD + AD.BC > AB.CD. (2.32)

Chứng minh.

Hình 2.23

Trong mặt phẳng (BCD) ta

lấy điểm E sao cho B và E

khác phía với đường thẳng CD

và AC = CE,AD = DE. Từ đó

ta có ∆ACD = ∆ECD. Suy ra

AP = PE, trong đó P là giao

điểm của BE và CD. Áp dụng

bất đẳng thức Ptolemy cho tứ

giác BCED, ta có BE.CD ≤CE.BD +BC.DE.

Mặt khác, AB.CD ≤(AP + PB)CD = PE.CD +

PB.CD = BE.CD.

Vậy AB.CD ≤ BE.CD ≤ CE.BD + BC.DE = AC.BD + AD.BC.

Ngoài ra dấu đẳng thức không xảy ra.

Bài toán 2.23. (Olympic 30/4, Việt Nam 2000) Cho hình chóp tam

giác S.ABC. Giả sử các trung tuyến của các tam giác SAB, SBC, SCA

kẻ từ S tạo với những cạnh đáy AB,BC,CA các góc không tù bằng

nhau. Chứng minh diện tích một mặt bên của hình chóp nhỏ hơn tổng

diện tích các mặt bên còn lại.

www.VNMATH.com

Page 71: Bất đẳng thức hình học

69

Giải. Gọi α là góc tạo bởi các trung tuyến SM,SK, SL với các cạnh

đáy AB,BC,CA. Vì vai trò của các mặt bên là như nhau nên ta chỉ

cần chứng minh [SAB] < [SBC] + [SCA]. Thật vậy, áp dụng định lí

2.5 cho tứ diện SKLM ta có SM.KL < SK.LM + SL.KM . Bất đẳng

thức trên tương đương với các bất đẳng thức sau SM.AB < SK.BC +

SL.CA ⇔ SM.AB. sinα < SK.BC. sinα + SL.CA. sinα ⇔ [SAB] <

[SBC] + [SCA] .

Bài toán 2.24. (Tạp chí THTT, số 264) Trên cạnh CD của hình tứ diện

ABCD lấy điểm N (N khác C,D). Kí hiệu p(XY Z) là chu vi tam giác

XY Z. Chứng minh rằng NC.p(DAB) +ND.p(CAB) > CD.p(NAB).

Giải. Xét bất đẳng thức Ptolemy cho các bộ 4 điểm (N,A,C,D) và

(N,C,B,D) ta có

NC.DA+ND.CA > CD.NA, (2.33)

NC.DB +ND.CB > CD.NB. (2.34)

Mặt khác, vì N thuộc đoạn CD nên NC +ND = CD. Do đó

NC.AB +ND.AB = CD.AB. (2.35)

Cộng theo vế 3 bất đẳng thức (2.33),(2.34) và (2.35) ta được bất đẳng

thức cần chứng minh.

www.VNMATH.com

Page 72: Bất đẳng thức hình học

70

Chương 3Bất đẳng thức Erdos-Mordell vàcác mở rộng

Bất đẳng thức Erdos-Mordell là một bất đẳng thức nổi tiếng trong

tam giác, được nhà toán học Paul Erdos đề xuất năm 1935 và lời giải

đầu tiên đưa ra là của Louis Mordell sử dụng định lý hàm số cosin.

Chương này trình bày bất đẳng thức Erdos-Mordel và các bài toán

áp dụng. Ngoài ra còn trình bày một số mở rộng của bất đẳng thức này

trong tam giác và mở rộng trong đa giác. Nội dung chủ yếu được hình

thành từ các tài liệu [11 -13].

Ngoài các kí hiệu sử dụng ở chương 1, ta sử dụng thêm các kí hiệu

sau: Cho tam giác ABC và P là một điểm nằm trong tam giác. Kí hiệu

R1, R2, R3 lần lượt là khoảng cách từ P đến các đỉnh A,B,C và r1, r2, r3lần lượt là khoảng cách từ P đến các cạnh BC,CA,AB.

3.1. Bất đẳng thức Erdos-Mordell trong tam giác

Định lý 3.1. (Bất đẳng thức Erdos-Mordell) Cho tam giác ABC và P

là điểm nằm trong tam giác. Khi đó luôn có bất đẳng thức

R1 +R2 +R3 ≥ 2 (r1 + r2 + r3) . (3.1)

Đẳng thức xảy ra khi và chỉ khi tam giác ABC đều và P là trực tâm

của nó.

Bất đẳng thức Erdos-Mordell có rất nhiều cách chứng minh khác

nhau. Ngoài chứng minh bằng cách sử dụng định lý Ptolemy, ta trình

bày hai cách chứng minh đơn giản sau đây:

Chứng minh 1. Kẻ tia Ax đối xứng với tia AP qua phân giác của góc

A. Từ B và C kẻ các đường vuông góc BB′ và CC ′ tới tia Ax. Ta có

PAC = BAB′, PAB = CAC ′. Do đó a ≥ BB′+CC ′ = AB. sin BAB′+

AC. sin CAC ′ = c. sin PAC + b. sin PAB = c.r2R1

+ b.r3R1.

www.VNMATH.com

Page 73: Bất đẳng thức hình học

71

Hình 3.1

Suy ra

R1 ≥ r2c

a+ r3

b

a, (3.2)

tương tự, ta có

R2 ≥ r3a

b+ r1

c

b, (3.3)

R3 ≥ r1b

c+ r2

a

c. (3.4)

Cộng theo vế các bất đẳng thức

(3.2), (3.3), (3.4) và áp dụng bất

đẳng thức AM −GM ta được

R1 +R2 +R3 ≥ r1

(c

b+b

c

)+ r2

(ac

+c

a

)+ r3

(a

b+b

a

)≥ 2 (r1 + r2 + r3) .

Rõ ràng dấu đẳng thức xảy ra khi và chỉ khi a = b = c và trong (3.2),

(3.3), (3.4) dấu đẳng thức xảy ra. Điều này xảy ra khi và chỉ khi tam

giác ABC đều và P là trực tâm của nó.

Chứng minh 2. Đây là chứng minh đầu tiên được đưa ra bởi Mordell.

Hình 3.2

Áp dụng định lý hàm số sin và hàmsố cosin, ta có

R1. sinA = EF =√r22 + r23 − 2r2r3 cos (π −A),

R2. sinB = FD =√r23 + r21 − 2r3r1 cos (π −B),

R3. sinC = DE =√r21 + r22 − 2r1r2 cos (π − C).

Mặt khác, ta có

EF 2 =r22 + r23 − 2r2r3 cos (π − A)

=r22 + r23 − 2r2r3 (cosB cosC − sinB sinC) .

= (r2 sinC + r3 sinB)2 + (r2 cosC − r3 cosB)2 ≥ (r2 sinC + r3 sinB)2 .

Suy ra EF ≥ r2 sinC + r3 sinB. Do đó R1 ≥ r2sinC

sinA+ r3

sinB

sinA.

Tương tự R2 ≥ r3sinA

sinB+ r1

sinC

sinB, R3 ≥ r1

sinB

sinC+ r2

sinA

sinC.

www.VNMATH.com

Page 74: Bất đẳng thức hình học

72

Cộng theo vế 3 bất đẳng thức trên, rồi áp dụng bất đẳng thức AM−GMta được

R1 +R2 +R3 =r1

(sinB

sinC+

sinC

sinB

)+ r2

(sinA

sinC+

sinC

sinA

)+

+r3

(sinA

sinB+

sinB

sinA

)≥ 2 (r1 + r2 + r3) .

Dấu đẳng thức xảy ra khi và chỉ khi sinA = sinB = sinC và r1 = r2 =

r3, điều này có nghĩa tam giác ABC đều và P là trực tâm của nó.

Bài toán 3.1. Cho tam giác ABC và P là một điểm tùy ý trong tam

giác. Chứng minh rằng

2

(1

R1+

1

R2+

1

R3

)≤ 1

r1+

1

r2+

1

r3. (3.5)

Giải.

Hình 3.3

Xét phép nghịch đảo N cực P và

phương tích d = r2

N :A→ A′;B → B′;C → C ′,

A1 → A′1;B1 → B′1;C1 → C ′1.

Khi đó, ta có

PA.PA′ = PB.PB′ = PC.PC ′ = d2,

PA1.PA′1 = PB1PB

′1 = PC1.PC

′1 = d2.

Hơn nữa A′, B′, C ′ nằm trên

B′1C′1, C

′1A′1, A

′1B′1 tương ứng và

PA′, PB′, PC ′ vuông góc với B′1C′1, C

′1A′1, A

′1B′1 tương ứng.

Áp dụng bất đẳng thức Erdos-Mordell cho tam giác A′1B′1C′1 ta có

PA′1 + PB′1 + PC ′1 ≥ 2 (PA′ + PB′ + PC ′).

Vì PA′1 =d2

PA1, PB′1 =

d2

PB1, PC ′1 =

d2

PC1, PC ′ =

d2

PC, PB′ =

d2

PB,

PA′ =d2

PAnên d2

(1

PA1+

1

PB1+

1

PC1

)≥ 2d2

(1

PA+

1

PB+

1

PC

).

Do đó 2

(1

R1+

1

R2+

1

R3

)≤ 1

r1+

1

r2+

1

r3.

www.VNMATH.com

Page 75: Bất đẳng thức hình học

73

Bài toán 3.2. Cho tam giác ABC và P là một điểm tùy ý trong tam

giác. Chứng minh rằng

R1r1 +R2r2 +R3r3 ≥ 2 (r1r2 + r2r3 + r3r1) . (3.6)

Giải.

Hình 3.4

Gọi ha là độ dài đường cao xuất phát từ

A, ta có aha = 2[ABC] = ar1 + br2 + cr3.

Vì ha ≤ R1 + r1 nên a(R1 + r1) ≥ aha, suy

ra aR1 ≥ br2 + cr3 hay R1 ≥ r2b

a+ r3

c

a.

Do đó R1r1 ≥ r1r2b

a+ r3r1

c

a. Tương tự

R2r2 ≥ r2r3c

b+r1r2

a

b,R3r3 ≥ r3r1

a

c+r2r3

b

c.

Cộng theo vế 3 bất đẳng thức trên và áp

dụng bất đẳng thức AG-GM, ta được

R1r1 +R2r2 +R3r3 ≥(a

b+b

a

)r1r2 +

(b

c+c

b

)r2r3 +

( ca

+a

c

)r3r1

≥2 (r1r2 + r2r3 + r3r1) .

Bài toán 3.3. Cho tam giác ABC và P là một điểm tùy ý trong tam

giác. Chứng minh rằng

R1R2 +R2R3 +R3R1 ≥ 4 (r1r2 + r2r3 + r3r1) . (3.7)

Giải. Xét phép nghịch đảo N cực P và phương tích d = r2

N : A→ A′;B → B′;C → C ′.

Gọi r′1, r′2, r′3 là khoảng cách từ P tới các cạnh B′C ′, C ′A′, A′B′, tương

ứng. Ta có r′1.B′C ′ = 2 [PB′C ′] =

PB′.PC ′.B′C ′

PA′1=r1.PB

′.PC ′.B′C ′

d2,

hay r′1 =r1.PB

′.PC ′

d2, trong đó A′1 là ảnh của A1 (A1 là hình chiếu của

P trên BC). Tương tự r′2 =r2.PC

′.PA′

d2, r′3 =

r3.PA′.PB′

d2.

Áp dụng bất đẳng thức Erdos-Mordell cho tam giác A′B′C ′, ta có

PA′+PB′+PC ′ ≥ 2 (r′1 + r′2 + r′3). Vì PA.PA′ = PB.PB′ = PC.PC ′ =

www.VNMATH.com

Page 76: Bất đẳng thức hình học

74

d2 nên1

PA+

1

PB+

1

PC≥ 2

( r1PB.PC

+r2

PC.PA+

r3PA.PB

)Bất đẳng

thức trên tương đương với

R2R3 +R3R1 +R1R2 ≥ 2 (R1r1 +R2r2 +R3r3) . (3.8)

Từ bất đẳng thức (3.6) và (3.8) ta thu được

R1R2 +R2R3 +R3R1 ≥ 4 (r1r2 + r2r3 + r3r1) .

Bài toán 3.4. Cho tam giác ABC và P là một điểm tùy ý trong tam

giác. Chứng minh rằng

R1R2R3 ≥ 8r1r2r3. (3.9)

Giải. Gọi C1 là điểm trên BC sao cho BC1 = AB. Khi đó AC1 = 2c sin B2 .

Mặt khác, gọi K,H lần lượt là chân đường vuông góc hạ từ A,C1 lên

Hình 3.5

đường thẳng BP , vì A và

C1 không nằm cùng phía đối

với đường thẳng BP nên

AC1 ≥ C1H + AK, suy ra

R2.AC1 ≥ R2.C1H + R2.AK hay

R2.AC1 ≥ 2[BPC1] + 2[BAP ].

Do đó R2.2c sin B2 ≥ c.r1 + c.r3,

hay R2 ≥r1 + r3

2 sin B2

, tương tự

R1 ≥r2 + r3

2 sin A2

, R3 ≥r1 + r2

2 sin C2

. Nhân 3 bất đẳng thức trên, ta được

R1R2R3 ≥1

8 sin A2 . sin

B2 . sin

C2

(r1 + r2) (r2 + r3) (r3 + r1) . (3.10)

Vì sinA

2. sin

B

2. sin

C

2=

r

4Rvà R ≥ 2r nên từ (3.10), ta có

R1R2R3 ≥ (r1 + r2) (r2 + r3) (r3 + r1) . (3.11)

Áp dụng bất đẳng thức AM −GM cho vế phải của (3.11), ta được

R1R2R3 ≥ 8r1r2r3.

www.VNMATH.com

Page 77: Bất đẳng thức hình học

75

Nhận xét 3.1. Từ bài toán trên, ta có các kết quả sau

1.R2

1

r2r3+

R22

r3r1+

R23

r1r2≥ 12,

2.R1

r2 + r3+

R2

r3 + r1+

R3

r1 + r2≥ 3,

3.R1√r2r3

+R2√r3r1

+R3√r1r2

≥ 6.

Chứng minh. Áp dụng bất đẳng thức AM −GM , ta có

1.R2

1

r2r3+

R22

r3r1+

R23

r1r2≥ 3 3

√R2

1

r2r3

R22

r3r1

R23

r1r2≥ 3

3√

82 = 12.

2.R1

r2 + r3+

R2

r3 + r1+

R3

r1 + r2≥ 3 3

√R1

r2 + r3

R2

r3 + r1

R3

r1 + r2≥ 3.

3.R1√r2r3

+R2√r3r1

+R3√r1r2

≥ 3 3

√R1√r2r3

R2√r3r1

R3√r1r2

≥ 3 3√

8 = 6.

Bài toán 3.5. Cho tam giác ABC và P là một điểm tùy ý trong tam

giác. Chứng minh rằng

R1 +R2 +R3 ≥ 6r. (3.12)

Giải. Dễ thấy R1 + r1 ≥ ha, R2 + r2 ≥ hb, R3 + r3 ≥ hc. Cộng 3 bất đẳng

thức này theo vế và áp dụng bất đẳng thức Erdos-Modell ta được

ha + hb + hc ≤ (R1 +R2 +R3) + (r1 + r2 + r3) ≤3

2(R1 +R2 +R3) .

(3.13)

Bây giờ áp dụng bất đẳng thức AM - GM ta có

9 ≤ (ha + hb + hc)

(1

ha+

1

hb+

1

hc

)= (ha + hb + hc)

1

r. (3.14)

Từ (3.13) và (3.14) ta được 9r ≤ ha + hb + hc ≤3

2(R1 +R2 +R3).

Do đó R1 +R2 +R3 ≥ 6r.

www.VNMATH.com

Page 78: Bất đẳng thức hình học

76

Bài toán 3.6. (IMO, 1991). Cho tam giác ABC và P là điểm trong tam

giác. Chứng minh rằng có ít nhất một trong các góc PAB, PBC, PCA

nhỏ hơn hoặc bằng 300.

Giải. Giả sử D,E, F là chân các đường vuông góc hạ từ P tương ứng

xuống các cạnh BC,CA và AB. Giả sử các góc PAB, PBC, PCA đều

lớn hơn 300. Khi đó

PA+ PB + PC =PF

sin PAB+

PD

sin PBC+

PE

sin PCA

<PF

sin 300+

PD

sin 300+

PE

sin 300= 2 (PD + PE + PF ) ,

mâu thuẫn với bất đẳng thức Erdos - Mordell. Suy ra các góc

PAB, PBC, PCA đều lớn hơn 300 là sai. Từ đó được điều phải chứng

minh.

Bài toán 3.7. (Olympic 30/4, 1999, Việt Nam, đề đề nghị) Cho M là

một điểm bất kỳ trong tam giác ABC. Kí hiệu các khoảng cách từ M

đến 3 đỉnh A,B,C là Ra, Rb, Rc và các khoảng cách từ M đến 3 cạnh

BC,CA,AB là da, db, dc. Chứng minh rằng

da + db + dc ≥ 2

[dbdcRa

+dcdaRb

+dadbRc

].

Giải.

Hình 3.6

Gọi A1, B1, C1 lần lượt là chân

đường vuông góc của M xuống

BC,CA,AB. Ta có

B1C1 = MA. sinA = Ra. sinA,

C1A1 = MB. sinB = Rb. sinB,

A1B1 = MC. sinC = Rc. sinC.

Kẻ MA2⊥B1C1, MB2⊥C1A1,

MC2⊥A1B1, ta có

MA2 = MB1. sin MB1A2 = MB1. sin MAC1 =MB1.MC1

MA=dbdcRa

,

www.VNMATH.com

Page 79: Bất đẳng thức hình học

77

MB2 = MC1. sin MC1B2 = MC1. sin MBA1 =MC1.MA1

MB=dcdaRb

,

MC2 = MA1. sin MA1C2 = MA1. sin MCB1 =MA1.MB1

MC=dadbRc

.

Áp dụng bất đẳng thức Erdos-Mordell cho tam giác A1B1C1, ta có

MA1 +MB1 +MC1 ≥ 2 (MA2 +MB2 +MC2)

hay da + db + dc ≥ 2

[dbdcRa

+dcdaRb

+dadbRc

].

Bài toán 3.8. (IMO, 1996) Cho ABCDEF là một lục giác lồi thỏa mãn

AB song song với DE, BC song song với EF và CD song song với FA.

Kí hiệu RA, RC , RE tương ứng là các bán kính đường tròn ngoại tiếp các

tam giác FAB,BCD và DEF . Gọi p là chu vi của hình lục giác. Chứng

minh rằng

RA +RC +RE ≥p

2.

Giải.

Hình 3.7

Dựng các điểm M,N,P sao cho

MDEF,NFAB và PBCD là các

hình bình hành. Khi đó M,N,F

thẳng hàng; N,P,B thẳng hàng và

P,M,D cũng thẳng hàng. Giả sử

XY Z là tam giác tạo bởi các đường

thẳng qua B,D, F và vuông góc với

FA,BC,DE, tương ứng, trong đó B

nằm trên Y Z, D nằm trên ZX và

F nằm trên XY . Để ý rằng ∆MNP

đồng dạng với ∆XY Z.

Vì ∆DEF = ∆DMF nên chúng có bán kính đường tròn ngoại tiếp

bằng nhau. Hơn nữa, vì XM là đường kính của đường tròn ngoại tiếp

tam giác DMF , do đó XM = 2RE. Tương tự Y N = 2RA và ZP = 2RC .

Do đó, bất đẳng thức cần chứng minh có thể được viết lại là

XM + Y N + ZP ≥ BN +BP +DP +DM + FM + FN.

www.VNMATH.com

Page 80: Bất đẳng thức hình học

78

Trường hợp M ≡ N ≡ P bất đẳng thức trên là bất đẳng thức Erdos

- Mordell. Ta chứng minh các trường hợp còn lại.

Hình 3.8

Gọi Y ′, Z ′ đối xứng với Y và

Z qua đường phân giác trong của

góc X. Kí hiệu G,H lần lượt là

chân đường vuông góc của X và

M trên Y ′Z ′. Đặt x = Y ′Z ′, y =

XZ ′, z = XY ′, ta có XM ≥z

xDM +

y

xFM, Y N ≥ x

yFN +

z

yBN,ZP ≥ y

zBP +

x

zDP.

Cộng theo vế 3 bất đẳng thức

trên, ta được

XM + Y N + ZP ≥ z

xDM +

y

xFM +

x

yFN +

z

yBN +

y

zBP +

x

zDP.

(3.15)

Để ý rằng

y

zBP +

z

yBN =

(y

z+z

y

)(BP +BN

2

)+

(y

z− z

y

)(BP −BN

2

).

Vì hai tam giác XY Z và MNP đồng dạng, ta có thể đặt r như sau

r =FM − FN

XY=BN −BP

Y Z=DP −DM

ZX.

Áp dụng bất đẳng thức AM - GM, ta được

y

zBP +

z

yBN =

(y

z+z

y

)(BP +BN

2

)− r

2

(yx

z− zx

y

)≥BP +BN − r

2

(yx

z− zx

y

).

Tương tự, ta có các bất đẳng thức

x

yFN +

y

xFM ≥ FN + FM − r

2

(xz

y− yz

x

),

z

xDM +

x

zDP ≥ DM +DP − r

2

(zyx− xy

z

).

www.VNMATH.com

Page 81: Bất đẳng thức hình học

79

Nếu cộng các bất đẳng thức trên và thế vào (3.15), ta được

XM + Y N + ZP ≥ BN +BP +DP +DM + FM + FN.

Điều phải chứng minh.

3.2. Bất đẳng thức Erdos-Mordell trong tam giác mở rộng

Bổ đề 3.1. Giả sử x, y, z, θ1, θ2, θ3 là các số thực với θ1 + θ2 + θ3 = π.

Khi đó ta có

x2 + y2 + z2 ≥ 2 (yz cos θ1 + zx cos θ2 + xy cos θ3) (3.16)

Chứng minh. Vì θ3 = π − (θ1 + θ2) nên cos θ3 = − cos (θ1 + θ2) =

sin θ1 sin θ2 − cot θ1 cot θ2. Ta có

x2 + y2 + z2 − 2 (yz cos θ1 + zx cos θ2 + xy cos θ3)

= (z − (x cos θ2 + y cos θ1))2 + (x sin θ2 − y sin θ1)

2 ≥ 0.

Hệ quả 3.1. Giả sử p, q, r là các số dương, còn θ1, θ2, θ3 là các số thực

thỏa mãn θ1 + θ2 + θ3 = π. Khi đó ta có bất đẳng thức

p cos θ1 + q cos θ2 + r cos θ3 ≤1

2

(qr

p+rp

q+pq

r

)(3.17)

Chứng minh. Thay (x, y, z) =

(√qr

p,

√rp

q,

√pq

r

)vào (3.16).

Định lý 3.2. (Bất đẳng thức Barrow) Cho tam giác ABC, M là một

điểm nằm trong nó. Gọi R1, R2, R3 lần lượt là các khoảng cách từ M tới

các đỉnh A,B,C và l1, l2, l3 là độ dài các đường phân giác trong của các

góc BMC, CMA, AMB. Ta có bất đẳng thức

R1 +R2 +R3 ≥ 2 (l1 + l2 + l3) . (3.18)

Chứng minh. Đặt BMC = 2α, CMA = 2β, AMB = 2γ. Ta có α + β +

γ = π. Sử dụng công thức đường phân giác và định lý AM −GM có

l1 =2R2R3

R2 +R3cosα ≤

√R2R3 cosα,

www.VNMATH.com

Page 82: Bất đẳng thức hình học

80

l2 =2R3R1

R3 +R1cos β ≤

√R3R1 cos β,

l3 =2R1R2

R1 +R2cos γ ≤

√R1R2 cos γ.

Từ 3 bất đẳng thức trên và áp dụng bổ đề 3.1 ta được

2 (l1 + l2 + l3) ≤ 2√R2R3 cosα + 2

√R3R1 cos β + 2

√R1R2 cos γ

≤ R1 +R2 +R2.

Chú ý 3.1. Bất đẳng thức Erdos-Mordell chỉ là hệ quả của bất đẳng thức

Barrow. Thật vậy, ta có l1 ≥ r1, l2 ≥ r2, l3 ≥ r3 . Suy ra R1 +R2 +R3 ≥2 (r1 + r2 + r3).

Định lý 3.3. Cho P là một điểm tùy ý nằm trong tam giác ABC. Khi

đó với mọi số thực x, y, z ≥ 0 bất đẳng thức sau luôn đúng

R1x2 +R1y

2 +R3z2 ≥ 2 (r1yz + r2zx+ r3xy) .

Đẳng thức xảy ra khi và chỉ khi tam giác ABC đều, P là trực tâm và

x = y = z.

Chứng minh. Với P là điểm nằm trong tam giác ABC, theo (3.2), (3.3)

và (3.4) ta có các bất đẳng thức

R1x2 ≥ r2

c

ax2 + r3

b

ax2, R2y

2 ≥ r3a

by2 + r1

c

by2, R3z

2 ≥ r1b

cz2 + r2

a

cz2.

Cộng theo vế 3 bất đẳng thức trên, sau đó áp dụng bất đẳng thức

AM −GM ta được

R1x2 +R2y

2 +R3z2 ≥ r1

(c

by2 +

b

cz2)

+ r2

(acz2 +

c

ax2)

+ r3

(a

by2 +

b

ax2)

≥ 2 (r1yz + r2zx+ r3xy) .

Sau đây ta sẽ mở rộng bất đẳng thức Erdos-Mordell đối với hai điểm

tùy ý nằm trong tam giác, để có kết quả đó trước hết ta xét các bổ đề

sau.

www.VNMATH.com

Page 83: Bất đẳng thức hình học

81

Bổ đề 3.2. Với mọi số thực x1, x2, x3, y1, y2, y3 thỏa mãn x1x2 + x2x3 +

x3x1 ≥ 0 và y1y2 + y2y3 + y3y1 ≥ 0 ta luôn có bất đẳng thức sau

(x2 + x3) y1 + (x3 + x1) y2 + (x1 + x2) y3

≥ 2√

(x1x2 + x2x3 + x3x1) (y1y2 + y2y3 + y3y1)

Dấu đẳng thức xảy ra khi và chỉ khix1y1

=x2y2

=x3y3

.

Chứng minh. Áp dụng bất đẳng thức Cauchy-Schwarz ta được

(x2 + x3) y1 + (x3 + x1) y2 + (x1 + x2) y3

=(x1 + x2 + x3) (y1 + y2 + y3)− (x1y1 + x2y2 + x3y3)

=√[

x21 + x22 + x23 + 2 (x1x2 + x2x3 + x3x1)] [y21 + y22 + y23 + 2 (y1y2 + y2y3 + y3y1)

]− (x1y1 + x2y2 + x3y3)

≥2√

(y1y2 + y2y3 + y3y1) (x1x2 + x2x3 + x3x1) +√(

x21 + x22 + x23) (y21 + y22 + y23

)− (x1y1 + x2y2 + x3y3)

≥2√

(y1y2 + y2y3 + y3y1) (x1x2 + x2x3 + x3x1).

Bổ đề 3.3. Cho tam giác ABC và x, y, z là các số thực dương. Bất đẳng

thức sau luôn đúng

x sinA+ y sinB + z sinC ≤ 1

2(xy + yz + zx)

√x+ y + z

xyz. (3.19)

Đẳng thức xảy ra khi và chỉ khi x = y = z và ABC là tam giác đều.

Chứng minh. Trước hết ta có

x2 + y2 + z2 + 2yz cos 2A+ 2zx cos 2B + 2xy cos 2C ≥ 0. (3.20)

Vì x2 + y2 + z2 + 2yz cos 2A + 2zx cos 2B + 2xy cos 2C =

(x+ z cos 2B + y cos 2C)2 + (z sin 2B − y sin 2C)2 ≥ 0.

Trong (3.20) thay (x, y, z) bằng

(1

x,1

y,1

z

), ta được bất đẳng thức

x cos 2A+ y cos 2B + z cos 2C ≥ −1

2

(yz

x+zx

y+xy

z

). (3.21)

www.VNMATH.com

Page 84: Bất đẳng thức hình học

82

Thay cos 2A = 1− 2 sin2A, cos 2B = 1− 2 sin2B, cos 2C = 1− 2 sin2C

vào (3.21), ta được

2(xsin2A+ ysin2B + zsin2C

)≤ x+ y + z +

1

2

(yz

x+zx

y+xy

z

)hay

xsin2A+ ysin2B + zsin2C ≤ (yz + zx+ xy)2

4xyz. (3.22)

Bây giờ, áp dụng bất đẳng thức Cauchy-Schwarz và sử dụng (3.22), ta

được

(x sinA+ y sinB + z sinC)2 ≤(xsin2A+ ysin2B + zsin2C

)(x+ y + z)

≤(yz + zx+ xy)2

4xyz(x+ y + z) .

Suy ra x sinA+ y sinB + z sinC ≤ 1

2(yz + zx + xy)

√x+ y + z

xyz.

Đẳng thức xảy ra khi và chỉ khix+ z cos 2B + y cos 2C = 0z sin 2B − y sin 2C = 0sinA = sinB = sinC

⇔x = y = z

A = B = C⇔x = y = z

∆ABC đều.

Bổ đề 3.4. Cho tam giác ABC và điểm P tùy ý trong nó, ta luôn có

bất đẳng thức√abr1r2 + bcr2r3 + car3r1 ≥ 2 (r1r2 + r2r3 + r3r1) . (3.23)

Chứng minh. Đặt x = [PBC] , y = [PCA] , z = [PAB], ta có

x

x+ y + z=

[PBC]

[ABC]=

r1a

bc sinA=

2r1bc.

a

2 sinA=

2Rr1bc

.

Suy ra r1 =bc

2R.

x

x+ y + z.

Tương tự r2 =ca

2R.

y

x+ y + z, r3 =

ab

2R.

z

x+ y + z.

Do đó bất đẳng thức (3.23) tương đương với

abc

2R(x+ y + z)

√xy + yz + zx ≥ abc

R(x+ y + z)2

(a

2Ryz +

b

2Rzx+

c

2Rxy

)

www.VNMATH.com

Page 85: Bất đẳng thức hình học

83

hay

1

2(x+ y + z)

√xy + yz + zx ≥ yz sinA+ zx sinB + xy sinC. (3.24)

Bất đẳng thức (3.24) suy ra từ bổ đề 3.3 bằng cách thay (x, y, z) bằng(1x ,

1y ,

1z

). Hoàn thành chứng minh bổ đề 3.4.

Định lý 3.4. Cho P,Q là hai điểm tùy ý nằm trong tam giác ABC.

Gọi R1, R2, R3 và r1, r2, r3 lần lượt là các khoảng cách từ P tới các đỉnh

A,B,C và các cạnh BC,CA,AB. Đối với điểm Q định nghĩa D1, D2, D3

và d1, d2, d3 một cách tương tự. Khi đó ta có các bất đẳng thức sau.

R1D1 +R2D2 +R3D3 ≥ 4 (r1r2 + r2r3 + r3r1) , (3.25)

R1D1 +R2D2 +R3D3 ≥ 4 (d1d2 + d2d3 + d3d1) . (3.26)

Chứng minh.

Hình 3.9

Theo các bất đẳng thức (3.2), (3.3), (3.4), bổ đề 3.2, bổ đề 3.4 và định

lí 4.1 ta có

R1D1 +R2D2 +R3D3 = aR1.D1

a+ bR2.

D2

b+ cR3.

D3

c

≥ (cr2 + br3)D1

a+ (ar3 + cr1)

D2

b+ (br1 + ar2)

D3

c

≥2

√(abr1r2 + bcr2r3 + ca3r1)

(D1D2

ab+D2D3

bc+D3D1

ca

)≥2√abr1r2 + bcr2r3 + car3r1

≥4 (r1r2 + r2r3 + r3r1) .

www.VNMATH.com

Page 86: Bất đẳng thức hình học

84

Bất đẳng thức (3.25) được chứng minh. Còn bất đẳng thức (3.26) được

chứng minh tương tự.

Hệ quả 3.2. Với mọi tam giác ABC và hai điểm trong nó P,Q, ta có

R1D1 +R2D2 +R3D3 ≥ 4√

(r1r2 + r2r3 + r3r1) (d1d2 + d2d3 + d3d1).(3.27)

Chứng minh. Cộng theo vế hai bất đẳng thức (3.25) và (3.26), sau đó áp

dụng bất đẳng thức AM −GM , ta thu được bất đẳng thức (3.27).

Định lý 3.5. Cho tam giác ABC bất kỳ với độ dài các cạnh là a, b, c

và P là điểm nằm trong tam giác. Kí hiệu R1, R2, R3 là khoảng cách từ

P tương ứng đến các đỉnh A,B,C và r1, r2, r3 tương ứng là khoảng cách

từ P đến các cạnh BC,CA,AB. Với mọi số thực α ∈ [0, 1] ta luôn có

Rα1 +Rα

2 +Rα3 ≥ 2α (rα1 + rα2 + rα3 ) .

Trước hết ta xét bổ đề sau:

Bổ đề 3.5. Với mọi số dương x, y và với mọi số thực α thỏa mãn 0 <

α ≤ 1 ta có

(x+ y)α ≥ 2α−1 (xα + yα) . (3.28)

Chứng minh. Ta có bất đẳng thức (3.28) tương đương với(x

y+ 1

)α≥ 2α−1

((x

y

)α+ 1

). (3.29)

Đặt t =x

y> 0, bất đẳng thức (3.29) trở thành (t+ 1)α ≥ 2α−1 (tα + 1) .

Xét f(t) = (t+ 1)α− 2α−1 (tα + 1), t > 0. Ta chỉ cần xét với 0 < α < 1

(vì nếu α = 1 thì (3.28) hiển nhiên đúng).

Ta có f ′(t) = α (t+ 1)α−1 − αtα−12α−1 = α[(t+ 1)α−1 − (2t)α−1

].

f ′(t) = 0⇔ t = 1.

Từ bảng xét dấu suy ra f(t) ≥ 0, với mọi t > 0. Vậy bổ đề được chứng

minh.

Bây giờ ta chứng minh định lý 3.5: Áp dụng (3.2) và bổ đề (3.5) ta

có Rα1 ≥

(r2ca + r3b

a

)α ≥ 2α−1[(

r2ca

)α+(r3ba

)α],

www.VNMATH.com

Page 87: Bất đẳng thức hình học

85

Hình 3.10 Bảng xét dấu

tương tự Rα2 ≥ 2α−1

[(r3ab

)α+(r1cb

)α], Rα

3 ≥ 2α−1[(

r1bc

)α+(r2ac

)α].

Cộng theo vế ba bất đẳng thức trên, sau đó áp dụng bất đẳng thức

AM −GM ta được

Rα1 +Rα

2 +Rα3 ≥2α−1

[(r1cb

)α+

(r1b

c

)α]+ 2α−1

[(r2ca

)α+(r2ac

)α]+

+2α−1[(r3a

b

)α+

(r3b

a

)α]≥ 2α (rα1 + rα2 + rα3 ) .

3.3. Mở rộng bất đẳng thức Erdos-Mordell trong tứ giác

Định lý 3.2 cho ta ý tưởng xây dựng bất đẳng thức Erdos-Mordell mở

rộng trong tứ giác

Bổ đề 3.6. Giả sử x1, x2, x3, x4, θ1, θ2, θ3, θ4, là các số thực tùy ý và

θ1 + θ2 + θ3 + θ4 = π. Khi đó ta có bất đẳng thức

√2 (x1x2 cos θ1 + x2x3 cos θ2 + x3x4 cos θ3 + x4x1 cos θ4) ≤ x21 + x22 + x23 + x24.

(3.30)

Chứng minh. Từ nhận xét θ1+θ2+(θ3 + θ4) = π và θ3+θ4+(θ1 + θ2) = π,

áp dụng bổ đề 3.1 ta được

x1√2x2 cos θ1 + x2

x3√2

cos θ2 +x1√

2

x3√2

cos (θ3 + θ4) ≤1

2

(x212

+ x22 +x232

),

x3√2x4 cos θ3 + x4

x1√2

cos θ4 +x3√

2

x1√2

cos (θ1 + θ2) ≤1

2

(x232

+ x24 +x212

).

Cộng theo vế hai bất đẳng thức trên, với lưu ý rằng cos (θ3 + θ4) +

cos (θ1 + θ2) = 0, ta thu được bất đẳng thức cần chứng minh.

www.VNMATH.com

Page 88: Bất đẳng thức hình học

86

Định lý 3.6. Tứ giác lồi ABCD, P là một điểm nằm trong nó.

Kí hiệu R1, R2, R3, R4 lần lượt là khoảng cách từ P tới các đỉnh

A,B,C,D và l1, l2, l3, l4 độ dài các đường phân giác của các góc

APB, BPC, CPD, DPA, tương ứng. Khi đó

R1 +R2 +R3 +R4 ≥√

2 (l1 + l2 + l3 + l4) . (3.31)

Chứng minh. Đặt APB = 2α, BPC = 2β, CPD = 2γ, DPA = 2δ, ta có

α + β + γ + δ = π.

Hình 3.11

Sử dụng công thức đường phân giác

và bất đẳng thức AM −GM ta có

l1 =2R1R2

R1 +R2cosα ≤

√R1R2 cosα,

l2 =2R2R3

R2 +R3cos β ≤

√R2R3 cos β,

l3 =2R3R4

R3 +R4cos γ ≤

√R3R4 cos γ,

l4 =2R4R1

R4 +R1cos δ ≤

√R4R1 cos δ.

Từ 4 bất đẳng thức trên và áp dụng

bổ đề 3.6 ta có

√2 (l1 + l2 + l3 + l4) ≤

√2(√R1R2 cosα +

√R2R3 cos β+

+√R3R4 cos γ+

√R4R1 cos δ) ≤ R1 +R2 +R3 +R4.

Chú ý 3.2. Ta luôn có l1 ≥ r1, l2 ≥ r2, l3 ≥ r3, l4 ≥ r4 trong đó

r1, r2, r3, r4 lần lượt là khoảng cách từ P tới các cạnh AB,BC,CD,DA

của tứ giác lồi ABCD. Do đó, từ bất đẳng thức (3.31) ta có

R1 +R2 +R3 +R4 ≥√

2 (r1 + r2 + r3 + r4) . (3.32)

Bất đẳng thức (3.32) được xem là bất đẳng thức Erdos - Mordell

trong tứ giác.

www.VNMATH.com

Page 89: Bất đẳng thức hình học

87

3.4. Mở rộng bất đẳng thức Erdos-Mordell trong đa giác

Bổ đề 3.7. Với mọi dãy số thực x1, x2, · · · , xn ta luôn có

cosπ

n

n∑k=1

x2k −n−1∑k=1

xkxk+1 + x1xn =

=n−2∑k=1

1

2 sin kπn sin (k+1)π

n

(sin

(k + 1) π

nxk − sin

nxk+1 + sin

π

nxn

)2

.

Chứng minh. Bằng cách so sánh các hệ số của x2k và xkxk+1. Chẳng hạn,

hệ số của x2k là

sin (k+1)πn

2 sin kπn

+sin (k−1)π

n

2 sin kπn

=2 sin kπ

n cosπn2 sin kπ

n

= cosπ

n,

với k = 2, 3, · · · , n− 2 và hệ số của x21 làsin 2π

n

2 sin πn

= cos πn , hệ số của x2n−1

làsin (n−2)π

n

2 sin (n−1)πn

=sin 2π

n

2 sin πn

= cosπn và hệ số của x2n là

n−2∑k=1

sin2 πn

2 sin kπn sin (k+1)π

n

=n−2∑k=1

sin πn

2

(cot

n− cot

(k + 1) π

n

)=

sin πn

2

(cot

π

n− cot

(n− 1) π

n

)= cos

π

n.

Hệ số của xkxk+1 là− sin (k+1)π

n sin kπn

2 sin kπn sin (k+1)π

n

= −1 với k = 1, 2, · · · , n−2 và hệ

số của xn−1xn là−2 sin (n−2)π

n sin πn

2 sin (n−2)πn sin (n−1)π

n

=− sin π

n

sin πn

= −1. Hệ số của x1xn

là2 sin 2π

n sin πn

2 sin πn sin 2π

n

= 1 và hệ số của xkxn với k = 2, 3, · · · , n− 2 là

n−2∑k=2

2 sin (k+1)πn sin π

n

2 sin kπn sin (k+1)π

n

−n−3∑k=1

2 sin kπn sin π

n

2 sin kπn sin (k+1)π

n

=n−2∑k=2

sin πn

sin kπn

=n−3∑k=2

sin πn

sin (k+1)πn

= 0.

www.VNMATH.com

Page 90: Bất đẳng thức hình học

88

Hệ quả 3.3. Với mọi dãy số thực x1, x2, · · · , xn ta luôn có

cosπ

n

n∑k=1

x2k −n−1∑k=1

xkxk+1 + x1xn ≥ 0. (3.33)

Chứng minh. Vì sin kπn > 0, sin (k+1)π

n > 0 với mọi k = 1, 2, · · · , n nên

n−2∑k=1

1

2 sin kπn sin (k+1)π

n

(sin

(k + 1) π

nxk − sin

nxk+1 + sin

π

nxn

)2

≥ 0.

Từ bất đẳng thức trên và bổ đề 3.7 được bất đẳng thức (3.33)

Bổ đề 3.8. (Wolstenholme-Lenhard) Với mọi dãy số thực

R1, R2, · · · , Rn,Rn+1 = R1 và mọi dãy θ1, θ2, · · · , θn thỏa mãnn∑k=1

θk = (2r + 1) π, r ∈ N, bất đẳng thức sau luôn đúng

cosπ

n

n∑k=1

R2k ≥

n∑k=1

RkRk+1 cos θk. (3.34)

Chứng minh. Áp dụng hệ quả 3.3, tương tự ta có

cosπ

n

n∑k=1

y2k −n−1∑k=1

ykyk+1 + y1yn ≥ 0. (3.35)

với mọi dãy số thực y1, y2, · · · , yn.Đặt xk = Rk cos βk và yk = Rk sin βk trong đó Rk ∈ R và βk ∈ R .

Cộng theo vế hai bất đẳng thức (3.33) và (3.35) ta được

cosπ

n

n∑k=1

R2k −

n−1∑k=1

RkRk+1 [cosβkcosβk+1 + sin βk sin βk+1]

+R1Rn [cosβ1cosβn + sin β1 sin βn] ≥ 0

hay cos πn

n∑k=1

R2k −

n−1∑k=1

RkRk+1 cos (βk − βk+1) +R1Rncos (β1 − βn) ≥ 0.

Đặt βk − βk+1 = θk với k = 1, 2, · · · , n − 1 và β1 − βn = (2r + 1) π −θn, r ∈ N ta được bất đẳng thức cos

π

n

n∑k=1

R2k ≥

n∑k=1

RkRk+1 cos θk, trong

đó Rk ∈ R, θk ∈ R vàn∑k=1

θk = (2r + 1) π, r ∈ N.

www.VNMATH.com

Page 91: Bất đẳng thức hình học

89

Định lý 3.7. (Bất đẳng thức Erdos-Mordell cho đa giác) Cho

A1A2 · · ·An, (n ≥ 3) là đa giác lồi và P là một điểm tùy ý nằm

trong nó. Gọi R1, R2, · · · , Rn lần lượt là khoảng cách tù P tới các đỉnh

A1, A2, · · · , An và r1, r2, · · · , rn lần lượt là khoảng cách từ điểm P tới

các cạnh A1A2, A2A3, · · · , AnA1. Ta luôn có bất đẳng thức

n∑k=1

Rk ≥1

cosπ

n

n∑k=1

rk, (3.36)

Đẳng thức xảy ra khi và chỉ khi A1A2 · · ·An là đa giác đều và P là tâm

của nó.

Chứng minh. Gọi l1, l2, · · · , ln lần lượt là độ dài các đường phân giác

của các góc A1PA2, A2PA3, · · · , AnPA1. Đặt A1PA2 = 2θ1, A2PA3 =

2θ2, · · · , AnPA1 = 2θn, ta có θ1 + θ2 + · · ·+ θn = π.

Theo công thức đường phân giác và bất đẳng thức AM −GM ta có

l1 =2R1R2

R1 +R2cos θ1 ≤

√R1R2 cos θ1,

l2 =2R2R3

R2 +R3cos θ2 ≤

√R2R3 cos θ2,

........................................................

ln =2RnR1

Rn +R1cos θn ≤

√R4R1 cos θn.

Từ n bất đẳng thức trên và áp dụng bổ đề 3.8 ta có

1

cos1

n

n∑k=1

lk ≤1

cos1

n

n∑k=1

√RkRk+1. cos θk ≤

n∑k=1

Rk.

Vì l1 ≥ r1, l2 ≥ r2, · · · , ln ≥ rn nên từ bất đẳng thức trên, ta có

n∑k=1

Rk ≥1

cosπ

n

n∑k=1

rk.

Đẳng thức xảy ra khi và chỉ khi A1A2 · · ·An là đa giác đều và P là tâm

của nó.

www.VNMATH.com

Page 92: Bất đẳng thức hình học

90

3.5. Mở rộng bất đẳng thức Erdos-Mordell trong tứ diện

Bài toán 3.9. Cho tứ diện ABCD và P là một điểm tùy ý nằmtrong nó. Gọi d1, d2, d3, d4 lần lượt là khoảng cách từ P đến các đỉnhA,B,C,D và h1, h2, h3, h4 lần lượt là khoảng cách từ điểm P đến cácmặt (BCD), (CDA), (DAB), (ABC). Chứng minh rằng

d1 + d2 + d3 + d4 ≥ 2(√

h1h2 +√h1h3 +

√h1h4 +

√h2h3 +

√h2h4 +

√h3h4

). (3.37)

Giải. Gọi

Hình 3.12

ha là độ dài đường cao của tứ

diện hạ từ đỉnh A xuống mặt đáy

BCD. S1, S2, S3, S4 lần lượt là

diện tích của các tam giác BCD,

CDA,DAB, ABC. V1, V2, V3, V4,

V lần lượt là thể tích của các khối

tứ diện PBCD, PCDA, PDAB,

PABC, ABCD.

Ta có d1 + h1 ≥ ha ⇒ d1S1 +

h1S1 ≥ haS1 ⇒ d1S1 + 3V1 ≥3V = 3 (V1 + V2 + V3 + V4) ⇒d1S1 ≥ 3V2 + 3V3 + 3V4 = h2S2 +

h3S3 + h4S4 ⇒ d1 ≥ h2S2

S1+ h3

S3

S1+ h4

S4

S1. Tương tự d2 ≥ h1

S1

S2+ h3

S3

S2+

h4S4

S2, d3 ≥ h1

S1

S3+ h2

S3

S3+ h4

S4

S3, d4 ≥ h1

S1

S4+ h2

S3

S4+ h4

S3

S4.

Cộng theo vế 4 bất đẳng thức trên và áp dụng bất đẳng thức AM−GMđược

d1 + d2+d3 + d4 ≥(h1S1

S2+ h2

S2

S1

)+

(h1S1

S3+ h3

S3

S1

)+

(h1S1

S4+ h4

S4

S1

)+

(h2S2

S3+ h3

S3

S2

)+

(h2S2

S4+ h4

S4

S2

)+

(h3S3

S4+ h4

S4

S3

).

≥2(√

h1h2 +√h1h3 +

√h1h4 +

√h2h3 +

√h2h4 +

√h3h4

)

Nhận xét 3.2. Đặc biệt hóa điểm P và tứ diện ABCD ta thu được các

kết quả sau:

www.VNMATH.com

Page 93: Bất đẳng thức hình học

91

+ Nếu P ≡ I (I là tâm mặt cầu nội tiếp tứ diện ABCD) thì từ (3.37)

ta có

d1 + d2 + d3 + d4 ≥ 12r, (3.38)

trong đó r là bán kính mặt cầu nội tiếp tứ diện.

+ Nếu ABCD là tứ diện gần đều (AB = CD,BC = AD,AC = BD)

có diện tích một mặt là S thì từ (3.37) ta có

d1 + d2 + d3 + d4 ≥ 3h, (3.39)

trong đó h là chiều cao của tứ diện gần đều ABCD.

+ Nếu ABCD là tứ diện đều, cạnh a thì từ (3.39) ta có

d1 + d2 + d3 + d4 ≥ a√

6. (3.40)

+ Nếu ABCD là tứ diện vuông tại A (tức là AB⊥AC, AB⊥AD,AC⊥AD) và P ≡ I thì d1 = IA =

√3r kết hợp với (3.38) ta được

d2 + d3 + d4 ≥(

12−√

3)r. (3.41)

www.VNMATH.com

Page 94: Bất đẳng thức hình học

92

Chương 4Các bất đẳng thức có trọng

Chương này trình bày một số bất đẳng thức có trọng và làm đặc biệt

hóa để thu được các bất đẳng thức hình học mới liên quan đến khoảng

cách từ một điểm nằm trong tam giác đến các đỉnh và các cạnh. Nội

dung chủ yếu của chương được hình thành từ các tài liệu [9], [13] và [14].

4.1. Bất đẳng thức dạng Hayashi và các hệ quả

4.1.1. Bất đẳng thức Hayashi

Định lý 4.1. (Bất đẳng thức Hayashi) Cho M là một điểm tùy ý trong

mặt phẳng chứa tam giác ABC với độ dài các cạnh là a, b, c. Khi đó

aMB.MC + bMC.MA+ cMA.MB ≥ abc. (4.1)

Đây là một định lý có cách chứng minh hình học khá phức tạp. Sau

đây bằng cách dùng số phức sẽ cho ta một cách chứng minh định lý rất

ngắn gọn và ấn tượng. Chỉ cần dùng một đồng nhất thức đại số quen

thuộc và biểu diễn hình học của số phức.

Chứng minh. Ta đặt tam giác ABC lên mặt phẳng phức và gọi m,α, β, γ

tương ứng là toạ vị của M,A,B,C. Từ đồng nhất đại số

(m− β)(m− γ)

(α− β)(α− γ)+

(m− γ)(m− α)

(β − γ)(β − α)+

(m− α)(m− β)

(γ − α)(γ − β)= 1, (4.2)

đúng với mọi α, β, γ đôi một khác nhau và với mọi m. Đồng nhất thức

này có thể chứng minh bằng cách để ý vế trái là một tam thức bậc hai

theo m và có giá trị bằng 1 tại ba điểm phân biệt α, β, γ , do đó đồng

nhất 1.

Từ đồng nhất (4.2) chuyển qua modul ta được

|m− β| |m− γ||α− β| |α− γ|

+|m− γ| |m− α||β − γ| |β − α|

+|m− α| |m− β||γ − α| |γ − β|

≥ 1. (4.3)

www.VNMATH.com

Page 95: Bất đẳng thức hình học

93

Theo cách đặt trên ta được MA = |m − α|,MB = |m − β|, |MC| =

|m− γ|, c = AB = |α− β|, a = BC = |β − γ|, b = CA = |γ − α|. Khi đó

(4.3) tương đương vớiMB.MC

bc+MC.MA

ca+MA.MB

ab≥ 1.

Chú ý 4.1. Trường hợp xảy ra dấu đẳng thức ta sẽ xét đến trong bài

toán 4.1 ở mục dưới.

Định lý 4.2. ChoM là một điểm tùy ý trong mặt phẳng chứa tam giác

ABC với độ dài các cạnh là a, b, c. Khi đó

aMA2 + bMB2 + cMC2 ≥ abc. (4.4)

Chứng minh. Xét gốc của mặt phẳng phức ở M và gọi α, β, γ là tọa vị

các đỉnh A,B,C của tam giác ABC. Từ đồng nhất thức

α2

(α− β) (α− γ)+

β2

(β − α) (β − γ)+

γ2

(γ − α) (γ − β)= 1, (4.5)

đúng với mọi α, β, γ đôi một khác nhau. Chuyển qua modul ta được

1 =

∣∣∣∣∣∑cyc

α2

(α− β) (α− γ)

∣∣∣∣∣ ≤∑cyc

|α|2

|α− β| |α− γ|. (4.6)

Theo cách đặt trên ta được |α| = PA, |β| = PB, |γ| = PC và |β − γ| =a, |γ − α| = b, |α− β| = c, bất đẳng thức (4.6) tương đương với bất

đẳng thức aMA2 + bMB2 + cMC2 ≥ abc.

Định lý 4.3. ChoM là một điểm tùy ý trong mặt phẳng chứa tam giác

ABC với độ dài các cạnh là a, b, c. Khi đó

aMA3 + bMB3 + cMC3 ≥ abc.MG. (4.7)

Trong đó G là trọng tâm của tam giác ABC.

Chứng minh. Từ đồng nhất thức

x3 (y − z) + y3 (z − x) + z3 (x− y) = (x− y) (y − z) (z − x) (x+ y + z) , (4.8)

đúng với mọi số phức x, y, z. Chuyển qua modul, ta được

|x|3 |y − z|+ |y|3 |z − x|+ |z|3 |x− y| ≥ |x− y| |y − z| |z − x| |x+ y + z| . (4.9)

Xét gốc của mặt phẳng phức ở G và gọi α, β, γ,m là tọa vị của các điểm

A,B,C,M tương ứng. Trong bất đẳng thức (4.9) xét x = m − α, y =

m− β, z = m− γ ta thu được bất đẳng thức (4.7).

www.VNMATH.com

Page 96: Bất đẳng thức hình học

94

4.1.2. Các hệ quả của bất đẳng thức hyashi

Hệ quả 4.1. (Bất đẳng thức Euler) Kí hiệu R, r lần lượt là bán kính

đường tròn ngoại tiếp và nội tiếp tam giác ABC. Khi đó R ≥ 2r.

Chứng minh. Xét trường hợp M ≡ O (trong đó O là tâm đường

tròn ngoại tiếp tam giác ABC), bất đẳng thức (4.1) tương đương với

R2 (a+ b+ c) ≥ abc. Do đó R2 ≥ abc

a+ b+ c=abc

2p=

4R.S

2.Sr= 2Rr, từ đó

R ≥ 2r.

Hệ quả 4.2. Kí hiệuma,mb,mc lần lượt là độ dài các đường trung tuyến

xuất phát từ các đỉnh A,B,C của tam giác ABC. Khi đó

mamb

ab+mbmc

bc+mcma

ca≥ 9

4.

Chứng minh. Xét trường hợp M ≡ G (G là trọng tâm của tam giác

ABC), từ bất đẳng thức (4.1) ta suy ra bất đẳng thức cần chứng minh.

Dấu đẳng thức xảy ra khi và chỉ khi tam giác ABC đều.

4.1.3. Bài toán áp dụng

Bài toán 4.1. (Olympic toán Trung Quốc 1998) Cho ABC là tam giác

nhọn và cho P là một điểm trong nó. Chứng minh rằng

aPB.PC + bPC.PA+ bPA.PB ≥ abc. (4.10)

Đẳng thức xảy ra nếu và chỉ nếu P là trực tâm của tam giác ABC.

Giải. Giả sử P là gốc của mặt phẳng phức và đặt α, β, γ là tọa vị củaA,B,C, tương ứng. Khi đó với mọi α, β, γ ta có đồng nhất thức sau

αβ (α− β) + βγ (β − γ) + γα (γ − α) = − (α− β) (β − γ) (γ − α) . (4.11)

Lấy modul hai vế đồng nhất trên ta được bất đẳng thức

|αβ (α− β)|+ |βγ (β − γ)|+ |γα (γ − α)| ≥ |(α− β) (β − γ) (γ − α)| . (4.12)

Theo cách đặt trên ta có PA = |α| , PB = |β| , PC = |γ| , c =

|α− β| , a = |β − γ| , b = |γ − α|. Do đó được bất đẳng thức

aPB.PC + bPC.PA+ bPA.PB ≥ abc.

www.VNMATH.com

Page 97: Bất đẳng thức hình học

95

Bây giờ, ta xác định xem khi nào đẳng thức xảy ra. Đặt

z1 =αβ

(α− γ) (β − γ), z2 =

βγ

(β − α) (γ − α), z3 =

γα

(γ − β) (α− β),

với cách đặt này, từ (4.11) và (4.12) ta có các đẳng thức và bất đẳng

thức sau z1 + z2 + z3 = 1, |z1|+ |z2|+ |z3| ≥ 1.

Vậy đẳng thức xảy ra khi và chỉ khi z1, z2, z3 là các số thực dương.

Ta chứng minh z1, z2, z3 là các số thực dương khi và chỉ khi P là trực

tâm của tam giác ABC. Thật vậy, giả sử z1, z2, z3 là các số thực dương.

−z1z2z3

=

γ − α

)2

,−z2z3z1

=

α− β

)2

,−z3z1z2

=

β − γ

)2

.

Từ số phức αβ−γ có bình phương là số một số thực âm nên là các số thuần

ảo; vậy góc tạo bởi giữa các véc tơ biểu diễn α là−→PA và véc tơ biểu diễn

β − γ là−−→BC là góc có số đo 900, suy ra PA⊥BC. Tương tự β

γ−α là số

thuần ảo, suy ra PB⊥CA. Vậy P là trực tâm của tam giác ABC.

Đảo lại, nếu P là trực tâm của tam giác nhọn ABC. Điểm P ở bên

trong tam giác và PA⊥BC nên góc tạo bởi giữa véc tơ−→PA và

−−→BC có

số đo −900, suy ra tồn tại số thực dương r1 sao cho αβ−γ = −r1i. Tương

tự, tồn tại các số thực dương r2, r3 sao cho βγ−α = −r2i, γ

α−β = −r3i. Vậyz1, z2, z3 là các số thực dương. Từ các kết quả trên ta kết luận đẳng thức

xảy ra khi và chỉ khi P là trực tâm của tam giác ABC.

Bài toán 4.2. Cho G là trọng tâm của tam giác ABC . Gọi R1, R2, R3, R

là bán kính đường tròn ngoại tiếp các tam giác GBC,GCA,GAB,ABC

tương ứng. Chứng minh rằng R1 +R2 +R3 ≥ 3R.

Giải. Áp dụng định lí 4.1 với G là trọng tâm của tam giác ABC. Ta có

aGB.GC + bGC.GA+ cGA.GB ≥ abc, (4.13)

trong đó a, b, c là độ dài các cạnh của tam giác ABC.

Mặt khác, ta có aGB.GC = 4R1. [GBC] = 43R1. [ABC]. Tương tự

bGC.GA = 43R2. [ABC] , cGA.GB = 4

3R3. [ABC] . Từ đó (4.13) tương

đương với 43 (R1 +R2 +R3) . [ABC] ≥ 4R. [ABC]. Suy ra R1+R2+R3 ≥

3R.

www.VNMATH.com

Page 98: Bất đẳng thức hình học

96

Bài toán 4.3. (Chọn đội tuyển IMO Rumani 2004) Cho tam giác ABC

và P là một điểm trong nó. Gọi R1, R2, R3, R là bán kính đường tròn

ngoại tiếp các tam giác PBC,PCA, PAB,ABC tương ứng. Các đường

thẳng PA, PB, PC lần lượt cắt các cạnh BC,CA,AB tại A1, B1, C1. Ký

hiệu k1 =PA1

AA1, k2 =

PB1

BB1, k2 =

PC1

CC1. Chứng minh rằng k1R1 + k2R2 +

k3R3 ≥ R.

Giải.

Hình 4.1

Ta có k1 = PA1

AA1= [PA1B]

[AA1B] = [PA1C][AA1C]

=[PA1B]+[PA1C][AA1B]+[AA1C]

= [PBC][ABC] , tương tự k2 =

[PCA][ABC] , k3 = [PAB]

[ABC] .

Mặt khác [ABC] = abc4R , [PBC] =

aPB.PC4R1

, [PCA] = bPC.PA4R2

, [PAB] =cPA.PB

4R3.

Bất đẳng thức Hayashi áp dụng cho

tam giác ABC và điểm P nằm trong

nó tương đương với bất đẳng thức sau

aPB.PC

4R1.R1 +

bPC.PA

4R2.R2 +

cPA.PB

4R3.R3 ≥ R.

abc

4R.

Do đó, ta được [PBC] .R1 + [PCA] .R2 + [PAB] .R3 ≥ R. [ABC] hay[PBC][ABC] .R1 + [PCA]

[ABC] .R2 + [PAB][ABC] .R3 ≥ R. Từ đó, được bất đẳng thức cần

chứng minh k1R1 + k2R2 + k3R3 ≥ R.

4.2. Bất đẳng thức Weizenbock suy rộng và các hệ quả

4.2.1. Bất đẳng thức Weizenbock suy rộng

Chúng ta đã biết các bất đẳng thức sau đây: Giả sử a, b, c là độ dài

ba cạnh, còn S là diện tích của một tam giác thì

a2 + b2 + c2 ≥ (a− b)2 + (b− c)2 + (c− a)2 + 4√

3S. (4.14)

a2 + b2 + c2 ≥ 4√

3S. (4.15)

Bất đẳng thức (4.14) được gọi là bất đẳng thức Hadwiger - Finsler, còn

bất đẳng thức (4.15) là bất đẳng thức Weizenbock là hệ quả của bất

www.VNMATH.com

Page 99: Bất đẳng thức hình học

97

đẳng thức Hadwiger - Finsler. Trong mục này xét sự mở rộng của bất

đẳng thức Weizenbock.

Định lý 4.4. Cho x, y, z là các số thực thỏa mãn các điều kiện x+y, y+

z, z + x, xy + yz + zx ≥ 0. Đặt a, b, c là độ dài ba cạnh và S là diện tích

của tam giác ABC. Khi đó

xa2 + yb2 + zc2 ≥ 4√xy + yz + zx.S. (4.16)

Chứng minh. Áp dụng định lý hàm số cosin c2 = a2 + b2 − 2ab cosC và

công thức diện tích S = 12ab sinC, ta có

xa2 + yb2 + zc2 ≥ 4√xy + yz + zx.S

⇔xa2 + yb2 + z(a2 + b2 − 2ab cosC

)≥√xy + yz + zx.2ab sinC

⇔ (x+ z) a2 + (y + z) b2 ≥ 2ab[√xy + yz + zx sinC + z cosC

]⇔ (x+ z)

a

b+ (y + z)

b

a≥ 2

[sinC

√xy + yz + zx+ z cosC

]. (4.17)

Áp dụng bất đẳng thức Cauchy - Schwarz, ta có[sinC

√xy + yz + zx+ z cosC

]2 ≤ (xy + yz + zx+ z2) (

sin2C + cos2C)

= xy + yz + zx+ z2 = (x+ z) (y + z) .

Mặt khác

[(x+ z)

a

b+ (y + z)

b

a

]2≥ 4 (x+ z) (y + z) .

Do đó (4.17) đúng.

Dấu đẳng thức xảy ra khi và chỉ khi(x+ z)

a

b= (y + z)

b

acosC

z=

sinC√xy + yz + zx

a√y + z

=b√x+ z

cos2C

z2=

sin2C

xy + yz + zx=

sin2C + cos2C

xy + yz + zx+ z2=

1

(x+ z) (y + z).

www.VNMATH.com

Page 100: Bất đẳng thức hình học

98

Thay b và cosC tương ứng vào biểu thức c2 = a2 + b2 − 2bc cosC ta

được

c2 = a2 + a2x+ z

y + z− 2a2

√x+ z

y + z

z√(x+ z) (y + z)

⇔( ca

)2= 1 +

x+ z

y + z− 2

z

y + z⇔ c

a=

√x+ y

y + z⇔ a√

y + z=

c√x+ y

.

Vậy đẳng thức xảy ra khi và chỉ khi a√y+z

=b√x+ z

=c√x+ y

.

Bất đẳng thức (4.16) được gọi là bất đẳng thức weizenbock suy rộng.

Tiếp theo, ta sẽ mở rộng bất đẳng thức Hadwiger - Finsler.

Định lý 4.5. Cho x, y, z là các số thực dương. Bất đẳng thức sau luôn

đúng

(y + z) a2 + (z + x) b2 + (x+ y) c2 ≥8√xy + yz + zxS + (x+ y) (a− b)2

+ (y + z) (b− c)2 + (z + x) (c− a)2 . (4.18)

Chứng minh.

Hình 4.2

Gọi A′, B′, C ′ lần lượt là

tâm đường tròn bàng tiếp

các góc A,B,C. Dễ dàng

chứng minh được bộ ba điểm

(A′B,C ′) , (B′, C, A′) , (C ′, A,B′)

thẳng hàng. Gọi K,H là

chân đường cao hạ từ C ′

xuống AB và từ B′ xuống

AC. Áp dụng định lí 4.4

vào tam giác A′B′C ′ ta có

zA′B′2 + xB′C ′2 + yC ′A′2 ≥4√xy + yz + zxSA′B′C ′.

Ta có

B′C ′ =B′A+ AC ′ =AH

cos(π2 −

A2

) +AK

cos(π2 −

A2

) =AH +HK

cos(π2 −

A2

)

www.VNMATH.com

Page 101: Bất đẳng thức hình học

99

=p− c+ p− bcos(π2 −

A2

) =a

sin A2

=2R sinA

sin A2

= 4R cosA

2.

Tương tự A′B′ = 4R cosC

2, C ′A′ = 4R cos

B

2. Do đó

SA′B′C ′ =1

2.B′C ′.A′B′. sinB′ =

1

2.4R cos

A

2.4R cos

C

2. cos

B

2

=8R2 cosA

2cos

C

2cos

B

2=R2 sin A sinB sinC

sin A2 sin B

2 sin C2

=1

8R.2R sinA.2R sinB.2R sinC

sin A2 sin B

2 sin C2

=abc

2r=abc

4R.2R

r=

2S.R

r.

Theo công thức vòng tròn bàng tiếp, ta có

ra + rb = p

[tan

A

2+ tan

B

2

]= p

(r

p− a+

r

p− b

)=

prc

(p− a) (p− b).

Mặt khác, từ công thức diện tích tam giác lại có

pr.abc

4R= p (p− a) (p− b) (p− c)⇒ prc

(p− a) (p− b)= 4R

p (p− c)ab

,

nên ra + rb =prc

(p− a) (p− b)= 4R

p (p− c)ab

= 4R cos2C

2.

Tương tự rb + rc = 4R cos2A

2, rc + ra = 4R cos2

B

2.

Từ các kết quả trên ta có

zA′B′2 + xB′C ′2 + yC ′A′2

=z.16R2 cos2C

2+ x.16R2 cos2

A

2+ y.16R2 cos2

B

2

=4R

(z.4R. cos2

C

2

)+ 4R

(x.4R. cos2

A

2

)+ 4R

(y.4R. cos2

B

2

)=4R [z (ra + rb) + x (rb + rc) + y (rc + ra)] .

Suy ra 4R [z (ra + rb) + x (rb + rc) + y (rc + ra)] ≥ 4√xy + yz + zx.2SRr .

Biết rằng ra(p− a) = rb(p− b) = rc(p− c) = pr = S, nên bất đẳng thức

trên tương đương với

⇔r [ra (y + z) + rb (z + x) + rc (z + x)] ≥ 2S√xy + yz + zx

www.VNMATH.com

Page 102: Bất đẳng thức hình học

100

⇔4 (y + z)S2

p (p− a)+ 4 (z + x)

S2

p (p− b)+ 4 (x+ y)

S2

p (p− c)≥ 8S

√xy + yz + zx

⇔4 (y + z) (p− b) (p− c) + 4 (z + x) (p− c) (p− a) + 4 (x+ y) (p− a) (p− b)≥ 8S

√xy + yz + zx

⇔ (y + z) a2 + (z + x) b2 + (x+ y) c2 ≥ 8S√xy + yz + zx+ (x+ y) (a− b)2

+ (y + z) (b− c)2 + (z + x) (c− a)2.

Nhận xét 4.1. Ở bất đẳng thức (4.18) nếu đặt y + z = x1, z + x =

y1, x+ y = z1 thì bất đẳng thức sẽ trở thành

x1a2 + y1b

2 + z1c2 ≥4

√x21 + y21 + z21 − (x1 − y1)2 − (y1 − z1)2 − (z1 − x1)2S

+x1(b− c)2 + y1(c− a)2 + z1(a− b)2. (4.19)

Như vậy bất đẳng thức (4.19) đúng với x1, y1, z1 là độ dài ba cạnh của

một tam giác. Vấn đề đặt ra là liệu bất đẳng thức (4.19) có còn đúng với

mọi x1, y1, z1 là các số thực dương hay không ? Câu trả lời là có, được

thể hiện thông qua định lí sau đây:

Định lý 4.6. Với x, y, z là các số thực dương. Chứng minh rằng

xa2 + yb2 + zc2 ≥4

√x2 + y2 + z2 − (x− y)2 − (y − z)2 − (z − x)2S

+x(b− c)2 + y(c− a)2 + z(a− b)2. (4.20)

Chứng minh. Áp dụng bất đẳng thức (4.16) cho tam giác A′B′C ′ ta có

(−x+ y + z)B′C ′2 + (x− y + z)C ′A′2 + (x+ y − z)A′B′2

≥4

√x2 + y2 + z2 − (x− y)2 − (y − z)2 − (z − x)2.SA′B′C ′.

Mặt khác, ta lại có

(−x+ y + z)B′C ′2 + (x− y + z)C ′A′2 + (x+ y − z)A′B′2

=(−x+ y + z) 16R2 cos2A

2+ (x− y + z) 16R2 cos2

B

2+ (x+ y − z) 16R2 cos2

C

2=4R [(−x+ y + z) (rb + rc) + (x− y + z) (rc + ra) + (x+ y − z) (ra + rb)]

=8R (xra + yrb + zrc) =8R

r(xrra + yrrb + zrrc)

www.VNMATH.com

Page 103: Bất đẳng thức hình học

101

=8R

r[x (p− b) (p− c) + y (p− c) (p− a) + z (p− a) (p− b)] .

4

√x2 + y2 + z2 − (x− y)2 + (y − z)2 + (z − x)2.SA′B′C ′

=4

√x2 + y2 + z2 − (x− y)2 + (y − z)2 + (z − x)2.

2S.R

r.

Suy ra

xa2 + yb2 + zc2 ≥ 4

√x2 + y2 + z2 − (x− y)2 − (y − z)2 − (z − x)2S

+x(b− c)2 + y(c− a)2 + z(a− b)2.

4.2.2. Các hệ quả của bất đẳng thức Weizenbock suy rộng

Hệ quả 4.3. Với mọi số thực dương x, y, z. Khi đó bất đẳng thức sau

luôn đúng

xab+ ybc+ zca ≥ 4√xy + yz + zx.S (4.21)

Chứng minh. Vì bất đẳng thức Weizenbock suy rộng (4.16) đúng với mọi

số thực dương x, y, z nên thay x→ x ba , y → y cb , z → z ac ta được

xab+ ybc+ zca ≥ 4

√xyc

a+ yz

a

b+ zx

b

c.S

hay

(xab+ ybc+ zca)2 ≥ 16S2

(xyc

a+ yz

a

b+ zx

b

c

). (4.22)

Lại thay x → zc

a, y → x

a

b, z → y

b

cvào bất đẳng thức Weizenbock suy

rộng, sau đó bình phương hai vế , ta được bất đẳng thức

(xab+ ybc+ zca)2 ≥ 16S2

(xya

c+ yz

b

a+ zx

c

b

). (4.23)

www.VNMATH.com

Page 104: Bất đẳng thức hình học

102

Cộng theo vế 2 bất đẳng thức (4.22) và (4.23), rồi áp dụng bất đẳng

thức AM −GM , ta được

2 (xab+ ybc+ zca)2 ≥ 16S2

[xy(ac

+c

a

)+ yz

(b

a+a

b

)+ zx

(b

c+c

b

)]≥ 2.16S2 (xy + yz + zx) .

Suy ra xab+ ybc+ zca ≥ 4√xy + yz + zx.S.

Trong các bất đẳng thức (4.16) và (4.21) có xuất hiện những biến số

x, y, z dương tùy ý. Ta nghĩ đến việc thay các số x, y, z bằng các biến

MA,MB,MC sẽ thu được một số kết quả sau

Hệ quả 4.4. Cho M là một điểm tùy ý trong tam giác ABC. Khi đó

bất đẳng thức sau luôn đúng

aMA+ bMB + cMC ≥ 4S. (4.24)

Chứng minh. Áp dụng bất đẳng thức Weizenbock suy rộng với x =MAa , y = MB

b , z = MCc , ta có aMA + bMB + cMC = MA

a a2 + MB

b b2 +

MCc c

2 ≥ 4√

MAa

MBb + MB

bMCc + MC

cMAa .S. Mặt khác, ta có MA

aMBb +

MBb

MCc + MC

cMAa ≥ 1 nên aMA+ bMB + cMC ≥ 4S.

Hệ quả 4.5. Cho M là một điểm tùy ý trong tam giác ABC. Khi đó

bất đẳng thức sau luôn đúng

bc

aMA+

ca

bMB +

ab

cMC ≥ 4S. (4.25)

Chứng minh. Áp dụng bất đẳng thức (4.21) hệ quả 4.3 với x =MCc , y = MA

a , z = MBb , ta có bc

aMA + cabMB + ab

cMC ≥4√

MAa

MBb + MB

bMCc + MC

cMAa .S ≥ 4S.

Hệ quả 4.6. Cho tam giác ABC với độ dài các cạnh a, b, c diện tích S

và tam giác A′B′C ′ với độ dài các cạnh a′, b′, c′ diện tích S ′ ; M là một

điểm tùy ý trong mặt phẳng. Khi đó bất đẳng thức sau luôn đúng

a′2

aMA+

b′2

bMB +

c′2

cMC ≥ 4S ′. (4.26)

www.VNMATH.com

Page 105: Bất đẳng thức hình học

103

Chứng minh. Áp dụng bất đẳng thức Weizenbock suy rộng cho tam giácA′B′C ′ với x = MA

a , y = MBb , z = MC

c , ta có

a′2

aMA+

b′2

bMB +

c′2

cMC ≥ 4

√MA

a

MB

b+MB

b

MC

c+MC

c

MA

a.S′ ≥ 4S′.

Các bất đẳng thức trên là khá khó và đẹp. Thế nhưng ta để ý rằng các

hệ số tổng quát x, y, z không xuất hiện. Các hệ số đã được thế bởi các

biến MA,MB,MC. Để vẫn xuất hiện các hệ số x, y, z, ta sử dụng bất

đẳng thức Hyashi MA.MBab + MB.MC

bc + MC.MAca ≥ 1 thu được kết quả sau

Hệ quả 4.7. Cho tam giác ABC với độ dài các cạnh a, b, c và tam giác

A′B′C ′ với độ dài các cạnh a′, b′, c′ diện tích S ′. Với M là một điểm tùy

ý trong mặt phẳng và x, y, z là các số thực dương, ta có

xa′2

a2MA2 + y

b′2

b2MB2 + z

c′2

c2MC2 ≥ 4

√xyz

x+ y + z.S ′.

Chứng minh. Áp dụng bất đẳng thức Weizenbock suy rộng cho tam giác

A′B′C ′, ta có

xa′2

a2MA2 + y

b′2

b2MB2 + z

c′2

c2MC2 =

x

a2MA2.a′2 +

y

b2MB2.b′2 +

z

c2MC2.c′2

≥4

√yzMB2

b2.MC2

c2+ zx

MC2

c2.MA2

a2+ xy

MA2

a2.MB2

b2.S ′. (4.27)

Mặt khác áp dụng bất đẳng thức Cauchy - Schwarz, ta có(yzMB2

b2.MC2

c2+ zx

MC2

c2.MA2

a2+ xy

MA2

a2.MB2

b2

)(1

yz+

1

zx+

1

xy

)≥(MB.MC

bc+MC.MA

ca+MA.MB

ab

)2

≥ 1.

Suy ra

yzMB2

b2.MC2

c2+ zx

MC2

c2.MA2

a2+ xy

MA2

a2.MB2

b2≥ xyz

x+ y + z. (4.28)

Từ (4.27) và (4.28) ta được bất đẳng thức cần chứng minh.

www.VNMATH.com

Page 106: Bất đẳng thức hình học

104

Hệ quả 4.8. Cho tam giác ABC, M là một điểm tùy ý trong mặt

phẳng. Khi đó bất đẳng thức sau luôn đúng

(MA.MB +MB.MC +MC.MA)

(a

MA+

b

MB+

c

MC

)≥ 12S.

Chứng minh. Áp dụng bất đẳng thức (4.21), ta có

MA.MB +MB.MC +MC.MA

=MA

a.MB

b.ab+

MB

b.MC

c.bc+

MC

c.MA

a.ca

≥4

√MA.MB.MC

abc

(MA

a+MB

b+MC

c

).S.

Để chứng minh bất đẳng thức đã cho ta cần chứng minh

MA.MB.MC

abc

(MA

a+MB

b+MC

c

)(a

MA+

b

MB+

c

MC

)2

≥ 9.

Thật vậy, ta có(MA

a+MB

b+MC

c

)(a

MA+

b

MB+

c

MC

)≥ 9 (4.29)

MA.MB.MC

abc

(a

MA+

b

MB+

c

MC

)=MB.MC

bc+MC.MA

ca+MA.MB

ab≥ 1. (4.30)

Từ (4.29) và (4.30) ta có điều phải chứng minh.

Hình 4.3

Ta thử mở rộng theo hướng khác

để xuất hiện các hệ số x, y, z. Với chú

ý rằng: Nếu gọi A′, B′, C ′ là hình chiếu

của M trên BC,CA,AB thì

MA =2R.B′C ′

a, MB =

2R.C ′A′

b,

MC =2R.A′B′

c.

www.VNMATH.com

Page 107: Bất đẳng thức hình học

105

Như thế, ta đã quy các biến MA,MB,MC về độ dài 3 cạnh của một

tam giác. Ta có kết quả sau

Hệ quả 4.9. Cho tam giác ABC với độ dài các cạnh a, b, c nội tiếp

đường tròn (O,R). M là một điểm bất kì trong tam giác. Khi đó với

mọi số thực dương x, y, z ta có

xMB.MC

a+ y

MC.MA

b+ z

MA.MB

c≥√xy + yz + zx.

(R2 −OM 2

)R

.

Chứng minh. Gọi A′, B′, C ′ là hình chiếu của M trên BC,CA,AB theo

định lý Euler ta có

[A′B′C ′] =1

4

(1− OM 2

R2

). [ABC]

Áp dụng hệ quả 4.3 cho tam giác A′B′C ′ với lưu ý MA =2R.B′C ′

a,

MB =2R.C ′A′

b, MC =

2R.A′B′

c, ta có

√xy + yz + zx.

(R2 −OM 2

)R

= 4R.√xy + yz + zx.

[A′B′C ′]

[ABC]

≤R.xA′B′.A′C ′ + yB′C ′.B′A′ + zC ′A′.C ′B′

[ABC]

=xMC.MB.cb+ yMA.MC.ac+ zMB.MA.ba

4R [ABC]

=xMC.MB.cb+ yMA.MC.ac+ zMB.MA.ba

abc

=xMB.MC

a+ y

MC.MA

b+ z

MA.MB

c.

4.3. Bất đẳng thức Klamkin và các hệ quả

4.3.1. Bất đẳng thức Klamkin

Vào năm 1975, M.S. Klamkin đã thiết lập định lý sau đây:

Định lý 4.7. Cho ABC là một tam giác tùy ý với độ dài các cạnh lần

lượt là a, b, c và P là điểm bất kỳ trong mặt phẳng chứa tam giác. Với

các số thực x, y, z ta có

(x+ y + z)(xPA2 + yPB2 + zPC2

)≥ yza2 + zxb2 + xyc2. (4.31)

www.VNMATH.com

Page 108: Bất đẳng thức hình học

106

Chứng minh. Ta có

x−→PA+ y

−−→PB + z

−→PC ≥ 0⇔

(x2PA2 + y2PB2 + z2PC2

)+

+(

2xy−→PA.−−→PB + 2yz

−−→PB.−→PC + 2zx

−→PC.−→PA)≥ 0. (4.32)

Theo định lí hàm số cosin ta có

2−→PA.−−→PB = 2PA.PB cos

(−→PA,−−→PB)

= PA2 + PB2 − c2,

2−−→PB.−→PC = 2PB.PC cos

(−−→PB,

−→PC)

= PB2 + PC2 − a2,

2−→PC.−→PA = 2PC.PA cos

(−→PC,−→PA)

= PC2 + PA2 − b2.

thay các bất đẳng thức trên vào (4.32) ta được thu được bất đẳng thức

(x+ y + z)(xPA2 + yPB2 + zPC2

)≥ yza2 + zxb2 + xyc2.

Đẳng thức trong (4.31) xảy ra khi và chỉ khi x−→PA+y

−−→PB+z

−→PC =

−→0 ,

tức là P là tâm tỉ cự của hệ điểm A,B,C.

4.3.2. Các hệ quả của bất đẳng thức Klamkin

Hệ quả 4.10. Trong tam giác ABC với G là trọng tâm, ta có các bất

đẳng thức sau

1. PA2 + PB2 + PC2 ≥ a2 + b2 + c2

3. (4.33)

2. PA2 + PB2 + PC2 ≥ 4

9

(m2a +m2

b +m2c

). (4.34)

3. PA2 + PB2 + PC2 ≥ GA2 +GB2 +GC2. (4.35)

Chứng minh. Khi x = y = z, bất đẳng thức 4.31 trở thành

3(PA2 + PB2 + PC2

)≥ a2 + b2 + c2.

Suy ra bất đẳng thức (4.33). Tiếp theo ta biến đổi bất đẳng thức (4.33)

như sau

PA2 + PB2 + PC2 ≥2(b2 + c2

)− a2

9+

2(c2 + a2

)− b2

9+

2(a2 + b2

)− c2

9.

Từ đây suy ra các bất đẳng thức (4.34) và (4.35).

www.VNMATH.com

Page 109: Bất đẳng thức hình học

107

Hệ quả 4.11. Trong tam giác ABC, ta có các bất đẳng thức sau

1.PA2

a2+PB2

b2+PC2

c2≥ a4 + b4 + c4

a2b2 + b2c2 + c2a2≥ 1. (4.36)

2.PA2

b2+PB2

c2+PC2

a2≥ 1. (4.37)

3.PA2

c2+PB2

a2+PC2

b2≥ 1. (4.38)

Chứng minh. Áp dụng bất đẳng thức Klamkin với x =1

a2, y =

1

b2, z =

1

c2

ta thu được bất đẳng thức (4.36). Nếu cho x =1

b2, y =

1

c2, z =

1

a2thì ta

có bất đẳng thức (4.37), còn nếu cho x =1

c2, y =

1

a2, z =

1

b2ta sẽ có bất

đẳng thức (4.38).

Hệ quả 4.12. Trong tam giác ABC, ta có các bất đẳng thức sau

1.m2a

a2+m2b

b2+m2c

c2≥ 9

4. (4.39)

2.mamb

ab+mbmc

bc+mcma

ca≥ 9

4. (4.40)

Chứng minh. Áp dụng bất đẳng thức Klamkin với P ≡ G, x =1

a2,

y =1

b2, z =

1

c2ta thu được bất đẳng thức (4.39). Nếu cho P ≡ G,

x =a

ma, y =

b

mb, z =

c

mc, ta sẽ có bất đẳng thức (4.40).

Hệ quả 4.13. Trong tam giác ABC, ta có các bất đẳng thức sau

1.cos2A

sinB sinC+

cos2B

sinC sinA+

cos2C

sinA sinB≥ 1. (4.41)

2. 4R2 ≥ a3 + b3 + c3 + abc

a+ b+ c. (4.42)

3.2R− rr

≥ a3 + b3 + c3

abc. (4.43)

Chứng minh. Áp dụng bất đẳng thức Klamkin với x = a, y = b, z = c,

P ≡ H (H là trực tâm của tam giác). Để ý rằng HA = |2R cosA| ,

www.VNMATH.com

Page 110: Bất đẳng thức hình học

108

HB = |2R cosB| , HC = |2R cosC| và sử dụng định lý hàm số sin ta

dễ dàng nhận được bất đẳng thức (4.41). Sử dụng các biến đổi HA2 =

4R2 cos2A = 4R2(1− sin2A

)= 4R2 − a2, HB2 = 4R2 − b2, HC2 =

4R2 − c2, sau một số biến đổi ta nhận được bất đẳng thức (4.42). Sử

dụng công thức abc = 4Rrp ta biến đổi bất đẳng thức (4.42) về bất đẳng

thức (4.43).

4.4. Bất đẳng thức Jian Liu và các hệ quả

4.4.1. Bất đẳng thức Jian Liu

Jian Liu đã chứng minh định lí 4.8 dưới đây. Để chứng minh định lí

4.8 ta cần bổ đề sau:

Bổ đề 4.1. Cho ABC là một tam giác tùy ý và P là một điểm bất kỳ

trong mặt phẳng chứa tam giác ABC. Nếu bất đẳng thức sau đây

f (a, b, c, R1, R2, R3) ≥ 0 (4.44)

đúng, thì bất đẳng thức sau cũng đúng

f (aR1, bR2, cR3, R2R3, R3R1, R1R2) ≥ 0, (4.45)

trong đó R1, R2, R3 lần lượt là khoảng cách từ P đến các đỉnh A,B,C.

Chứng minh. Xét phép nghịch đảo N tâm P , hệ số R1.R2.R3, ta có

N : A→ A′, B → B′, C → C ′

Khi đó ta có PA′ = PB.PC = R2R3, tương tự PB′ = R3R1,

PC ′ = R1R2 và B′C ′ = R1R2R3.BC

PB.PC = aR1, tương tự C ′A′ =

bR2, A′B′ = cR3. Vì f (a, b, c, R1, R2, R3) ≥ 0 đúng với mọi tam

giác ABC và mọi P nên áp dụng điều kiện này cho tam giác

A′B′C ′ và điểm P ta được f (B′C ′, C ′A′, A′B′, PA′, PB′, PC ′) ≥ 0 hay

f (aR1, bR2, cR3, R2R3, R3R1, R1R2) ≥ 0.

Định lý 4.8. Cho x, y, z là các số dương. Khi đó, với tam giác ABC

tùy ý và P là điểm bất kỳ trong mặt phẳng chứa ∆ABC, bất đẳng thức

www.VNMATH.com

Page 111: Bất đẳng thức hình học

109

sau đúng

R21

x+R2

2

y+R2

3

z≥ aR1 + bR2 + cR3√

xy + yz + zx, (4.46)

trong đó R1 = PA,R2 = PB,R3 = PC.

Đẳng thức xảy ra khi và chỉ khi ABC là tam giác nhọn, P trùng với

trực tâm H và x : y : z = cotA : cotB : cotC.

Chứng minh. Nếu P trùng với một trong các đỉnh của ∆ABC, chẳng

hạn, P ≡ A, thì PA = 0, PB = c, PC = b và bất đẳng thức (4.46) trở

thành tầm thường. Trong trường hợp này, dấu đẳng thức trong (4.46)

rõ ràng là không xảy ra.

Tiếp theo, giả sử P không trùng với một trong các đỉnh của ∆ABC.

Nếu x, y, z là các số dương thì bất đẳng thức (4.31) tương đương với bất

đẳng thức sau(xR2

1 + yR22 + zR2

3

)( 1

yz+

1

zx+

1

xy

)≥ a2

x+b2

y+c2

z.

Mặt khác, theo bất đẳng thức Cauchy - Schwarz ta có

a2

x+b2

y+c2

z≥ (a+ b+ c)2

x+ y + z.

Đẳng thức xảy ra khi và chỉ khi x : y : z = a : b : c.

Kết hợp hai bất đẳng thức trên ta có bất đẳng thức(xR2

1 + yR22 + zR2

3

)( 1

yz+

1

zx+

1

xy

)≥ (a+ b+ c)2

x+ y + z. (4.47)

Dấu đẳng thức trong (4.47) xảy ra khi và chỉ khi x : y : z = a : b : c vàP là tâm của đường tròn nội tiếp tam giác ABC.Bây giờ, áp dụng phép nghịch đảo trong bổ đề 4.1 cho bất đẳng thức(4.47), ta thu được[

x(R2R3)2 + y(R3R1)

2 + z(R1R2)2]( 1

yz+

1

zx+

1

xy

)≥ (aR1 + bR2 + cR3)

2

x+ y + z,

hay

(R2R3)2

yz+

(R3R1)2

zx+

(R1R2)2

xy≥(aR1 + bR2 + cR3

x+ y + z

)2

. (4.48)

www.VNMATH.com

Page 112: Bất đẳng thức hình học

110

Thay x→ xR21, y → yR2

2, z → zR23, ta được

1

yz+

1

zx+

1

xy≥(aR1 + bR2 + cR3

xR21 + yR2

2 + zR23

)2

. (4.49)

Lại một lần nữa thay x → 1

x, y → 1

y, z → 1

z, ta được bất đẳng thức

(4.46) cần chứng minh.

Nếu đẳng thức trong (4.44) chỉ xảy ra khi P là tâm của đường tròn nội

tiếp ∆ABC, thì đẳng thức trong (4.45) xảy ra chỉ khi ∆ABC là nhọn

và P là trực tâm của nó. Theo điều này và điều kiện để đẳng thức trong

(4.47) xảy ra, ta có đẳng thức trong (4.46) xảy ra khi và chỉ khi ∆ABC

là nhọn, P là trực tâm của nó và

R1

xa=R2

yb=R3

zc. (4.50)

Khi P là trực tâm của tam giác nhọn ABC, ta có R1 : R2 : R3 = cosA :

cosB : cosC. Do đó, trong trường hợp này, từ (4.50) ta có x : y : z =

cotA : cotB : cotC. Vì vậy, đẳng thức trong (4.46) xảy ra khi và chỉ khi

∆ABC là nhọn, P trùng với trực tâm của nó và xcotA = y

cotB = zcotC .

4.4.2. Các hệ quả của bất đẳng thức Jian Liu

Hệ quả 4.14. Với P tùy ý nằm trong mặt phẳng chứa tam giác ABC

và với mọi số dương x, y, z, bất đẳng thức sau luôn đúng

R21

x+R2

2

y+R2

3

z≥ 4S√

xy + yz + zx. (4.51)

Đẳng thức xảy ra khi và chỉ khi x : y : z = cotA : cotB : cotC và P là

trực tâm của tam giác nhọn ABC.

Chứng minh. Từ bất đẳng thức (4.24) và bất đẳng thức Jian Liu (4.46)

ta thu được bất đẳng thức cần chứng minh (4.51).

Hệ quả 4.15. Cho tam giác ABC và P là một điểm tùy ý trong mặt

phẳng chứa tam giác. Khi đó bất đẳng thức sau đúng

R21 +R2

2 +R23 ≥

3√3S. (4.52)

www.VNMATH.com

Page 113: Bất đẳng thức hình học

111

Chứng minh. Trong bất đẳng thức (4.51) cho x = y = z sẽ thu được bất

đẳng thức (4.52).

Hệ quả 4.16. Với P tùy ý nằm trong mặt phẳng chứa tam giác ABC

và với mọi số dương x, y, z, bất đẳng thức sau luôn đúng

aR21 + bR2

2 + cR23

aR1 + bR2 + cR3≥√

2Rr. (4.53)

Đẳng thức xảy ra khi và chỉ khi tam giác ABC đều và P là trực tâm

của nó.

Chứng minh. Trong bất đẳng thức Jian Liu (4.46) ta thay x =1

a, y =

1

b,

z =1

cvà sử dụng kết quả

1

bc+

1

ca+

1

ab=

1

2Rr, ta thu được bất đẳng

thức cần chứng minh (4.53).

Hệ quả 4.17. (Bất đẳng thức Hayashi) Nếu P là một điểm tùy ý và

không trùng với các đỉnh của tam giác ABC, thì

R2R3

bc+R3R1

ca+R1R2

ab≥ 1. (4.54)

Đẳng thức xảy ra khi và chỉ khi tam giác ABC nhọn và P là trực tâm

của nó.

Chứng minh. Trong bất đẳng thức (4.48), thay x = aR1, y = bR2,

z = cR3 ta được bất đẳng thức (4.54).

Hệ quả 4.18. Nếu P là một điểm tùy ý và không trùng với các đỉnh

của tam giác ABC, thì

(R2R3 +R3R1 +R1R2)2

(1

R2R3+

1

R3R1+

1

R1R2

)≥ 4p2. (4.55)

Trong đó p là nửa chu vi của tam giác ABC. Đẳng thức xảy ra khi và

chỉ khi tam giác ABC nhọn và P là trực tâm của nó.

Chứng minh. Trong (4.48) thay x → xaR1, y → ybR2, z → zcR3 . Khi

đó ta được

R2R3

yzbc+R3R1

zxca+R1R2

xyab≥(

aR1 + bR2 + cR3

xaR1 + ybR2 + zcR3

)2

. (4.56)

www.VNMATH.com

Page 114: Bất đẳng thức hình học

112

Tiếp tục thay x =1

a, y =

1

b, z =

1

c, ta được

(R2R3 +R3R1 +R1R2) (R1 +R2 +R3)2 ≥ (aR1 + bR2 + cR3)

2. (4.57)

Áp dụng phương pháp biến đổi nghịch đảo của bổ đề 4.1 cho bất đẳng

thức (4.57), sau đó chia cả hai vế cho (R1R2R3)2 ta được bất đẳng thức

(4.55).

Hệ quả 4.19. Cho P là một điểm nằm trong tam giác ABC, khi đó bất

đẳng thức sau luôn đúng

(R2R3)2

r2r3+

(R3R1)2

r3r1+

(R1R2)2

r1r2≥ 16

9p2. (4.58)

Trong đó p là nửa chu vi của tam giác ABC.

Chứng minh. Với P là một điểm tùy ý nằm trong tam giác ABC, ta có

các bất đẳng thức sau

aR1 ≥ br3 + cr2, bR2 ≥ cr1 + ar3, cR3 ≥ ar2 + br1.

Cộng theo vế 3 bất đẳng thức trên với lưu ý là a+ b+ c = 2p, ar1 + br2 +

cr3 = 2rp, ta thu được bất đẳng thức

aR1 + bR2 + cR3 ≥ 2p (r1 + r2 + r3)− 2rp.

Nhân cả hai vế với bất đẳng thức trên với 2, sau đó cộng với bất đẳng

thức aR1 + bR2 + cR3 ≥ 4S, với lưu ý là S = rp ta thu được

3 (aR1 + bR2 + cR3) ≥ 4p (r1 + r2 + r3) , (4.59)

hay

aR1 + bR2 + cR3

r1 + r2 + r3≥ 4

3p. (4.60)

Mặt khác, trong bất đẳng thức (4.48) cho x = r1, y = r2, z = r3 ta thu

được bất đẳng thức

(R1R2)2

r2r3+

(R3R1)2

r3r1+

(R1R2)2

r1r2≥(aR1 + bR2 + cR3

r1 + r2 + r3

)2

. (4.61)

Từ hai bất đẳng thức (4.60) và (4.61) ta được bất đẳng thức (4.58).

www.VNMATH.com

Page 115: Bất đẳng thức hình học

113

Hệ quả 4.20. Cho P là một điểm nằm trong tam giác ABC, khi đó bất

đẳng thức sau luôn đúng

(R1 +R2 +R3)2

r1 + r2 + r3≥ 4√

3p. (4.62)

Chứng minh. Từ bất đẳng thức (4.57) và (4.60) ta suy ra

(R2R3 +R3R1 +R1R2) (R1 +R2 +R3)2 ≥ 16

9p2 (r1 + r2 + r3)

2 .

Mặt khác, ta có

3 (R2R3 +R3R1 +R1R2) ≤ (R1 +R2 +R3)2 .

Từ hai bất đẳng thức trên ta được bất đẳng thức (4.62).

Hệ quả 4.21. Cho P là một điểm nằm trong tam giác ABC, khi đó bất

đẳng thức sau luôn đúng

R21

ra+R2

2

rb+R2

3

rc≥ 4

3(r1 + r2 + r3) . (4.63)

Chứng minh. Trong bất đẳng thức (4.46) thay x → ra, y → rb, z → rcvới lưu ý là rbrc + rcra + rarb = p2, ta có

R21

ra+R2

2

rb+R2

3

rc≥ 1

p(aR1 + bR2 + cR3) . (4.64)

Từ bất đẳng thức (4.60) và (4.64) ta được bất đẳng thức (4.63).

Hệ quả 4.22. Cho P là một điểm nằm trong tam giác ABC, khi đó bất

đẳng thức sau luôn đúng

R21

ra+R2

2

rb+R2

3

rc≥ r1 + r2 + r3 + r. (4.65)

Chứng minh. Cộng theo vế các bất đẳng thức (4.59) và aR1+bR2+cR3 ≥4S, sau đó chia cả hai vế cho 4, ta có

aR1 + bR2 + cR3 ≥ p (r1 + r2 + r3 + r) . (4.66)

Từ bất đẳng thức (4.64) và (4.66) ta được bất đẳng thức (4.65).

www.VNMATH.com

Page 116: Bất đẳng thức hình học

114

Hệ quả 4.23. Cho P là một điểm nằm trong tam giác ABC. Gọi

ha, hb, hc lần lượt là độ dài đường cao hạ từ A,B,C. Khi đó

R21

ha+R2

2

hb+R2

3

hc≥ 4r. (4.67)

Trong đó r là bán kính đường tròn nội tiếp tam giác ABC.

Chứng minh. Ta biết rằng 1ha

+ 1hb

+ 1hc

= 1r hay hahb + hbhc + hcha =

hahbhcr = 8S3

abcr = 2S2

rR . Vì R ≥ 2r nên

hahb + hbhc + hcha ≤S2

r2. (4.68)

Trong (4.51) cho x = ha, y = hb, z = hc và sử dụng (4.68) ta đượcR2

1

ha+R2

2

hb+R2

3

hc≥ 4S√

S2

r2

= 4r.

Hệ quả 4.24. Cho P là một điểm nằm trong tam giác ABC. Khi đó

bất đẳng thức sau luôn đúng

aR21 + bR2

2 + cR23 ≥

2√S3

4√

27(4.69)

Chứng minh. Theo bài toán 1.6 ta có 4√

3 ≤ 9abc

a+ b+ chay

1

ab+

1

bc+

1

ca≤ 3√

3

4S. (4.70)

Bây giờ trong (4.51) cho x =1

a, y =

1

b, z =

1

cvà sử dụng (4.4.2.) ta được

aR21 + bR2

2 + cR23 ≥ 4S√

3√3

4S

= 2√S3

4√27.

Hệ quả 4.25. Cho P là một điểm nằm trong tam giác ABC. Gọi

ma,mb,mc lần lượt là độ dài trung tuyến xuất phát từ A,B,C. Khi

đó bất đẳng thức sau luôn đúng

maR21 +mbR

22 +mcR

23 ≥

4S√S

4√

3. (4.71)

www.VNMATH.com

Page 117: Bất đẳng thức hình học

115

Chứng minh. Gọi A1, B1, C1 lần lượt là trung điểm của BC,CA,AB.

Dựng tam giác AA1M sao cho M đối xứng với C1 qua B1. Dễ thấy

ma,mb,mc là độ dài các cạnh của tam giác MAA1 (ma = AA1,mb =

MA1,mc = MA) và [MAA1] = 34S. Áp dụng bất đẳng thức ta được

1

mamb+

1

mbmc+

1

mcma≤ 4S

√S

4√

3=

√3

S. (4.72)

Bây giờ trong (4.51) cho x =1

ma, y =

1

mb, z =

1

mcvà sử dụng bất đẳng

thức (4.72) ta được maR21 +mbR

22 +mcR

23 ≥

4S√√3S

=4S√S

4√

3.

www.VNMATH.com

Page 118: Bất đẳng thức hình học

116

Kết luận

Luận văn đã trình bày và đạt được một số kết quả sau:

1. Trình bày một số bất đẳng thức trong tam giác và trong tứ giác từ

cơ bản đến nâng cao.

2. Giới thiệu hai bất đẳng thức quen thuộc và có nhiều áp dụng trong

các đề thi học sinh giỏi đó là bất đẳng thức Erdos-modell và bất đẳng

thức Ptolemy. Đồng thời cũng xây dựng một số mở rộng của hai bất

đẳng thức này.

3. Trình bày một số bất đẳng thức có trọng, các mở rộng và áp dụng,

đồng thời đặc biệt hóa để có các bài toán bất đẳng thức hình học mới

liên quan đến khoảng cách từ một điểm đến các đỉnh và các cạnh của

tam giác.

4. Tác giả đã nêu và chứng minh được một số bổ đề giúp cho việc chứng

minh các bất đẳng quan trọng trong luận văn như bất đẳng thức hình

bình hành, bất đẳng thức Erdos-Modell mở rộng đối với 2 điểm trong

tam giác, mở rộng trong tứ giác và đa giác, bất đẳng thức Jian Liu v.v..

Ngoài ra, khai thác một số bất đẳng thức tác giả đã đặc biệt hóa để thu

được nhiều bất đẳng thức hay và đẹp.

5. Luận văn đã chọn lọc giới thiệu một số đề thi học sinh giỏi trong

nước, khu vực và quốc tế liên quan đến các bất đẳng thức trong tam

giác và trong tứ giác. Các bất đẳng thức hình học áp dụng trực tiếp hai

bất đẳng thức nổi tiếng đó là bất đẳng thức Erdos-Modell và bất đẳng

thức Ptolemy.

www.VNMATH.com

Page 119: Bất đẳng thức hình học

117

Tài liệu tham khảo

[1] Bộ giáo dục và đào tạo-Hội toán học Việt Nam (2009), Các bài toánchọn lọc - 45 năm Tạp chí Toán học và Tuổi trẻ, NXB Giáo Dục.

[2] Nguyễn Minh Hà, Suy nghĩ mới từ một bài toán quen thuộc, Tuyểntập 5 năm tạp chí toán học và tuổi trẻ, NXB Giáo Dục (1995).

[3] Vũ Đình Hòa (2005), Bất đẳng thức hình học, NXB Giáo Dục.

[4] Phan Huy Khải, Nguyễn Đạo Phương (1994), Hệ thức lượng trongtam giác và tứ giác, NXB Giáo Dục.

[5] Nguyễn Văn Nho (2011), Những định lí chọn lọc trong hình họcphẳng qua các kì thi Olympic, NXB Đại Học Sư Phạm.

[6] Nguyễn Đức Tấn (2000), Chuyên đề bất đẳng thức và cực trị tronghình học phẳng, NXB Giáo dục.

[7] Nguyễn Thượng Võ (1989), 200 bài toán chọn lọc về hệ thức lượngtrong tam giác, NXB Giáo dục.

[8] Trần Nam Dũng, Ptolemy’s inequality and its applications, Kỷ yếuhội nghị khoa học, Việt Trì (2011).

[9] Titu Andreescu and Dorin Andrica, Proving some geometric in-equalities by using complex numbers, Educatia Matematica Vol.1,N2, (2005), 19-26.

[10] Dusan Djukic, Vladimir Jankovic, Ivan Matic, Nikola Petrovic, TheIMO Compendium A Collection of Problems Suggested for the Inter-national Mathematical Olympiads:1959–2004, Springer Publishers,2004.

[11] Yu-Dong Wu, Chun-Lei Yu Zhi-Hua Zhang, A geometric inequalityof the generalized Erdos-Modell type, Journal of inequalities in pureand applied mathematics, Vol.10, Iss.4, Ar.106, 2009.

[12] Radmila Bulajich Manfrino, José Antonio Gómez Ortega, RogelioValdez Delgado, Inequalities A Mathematical Olympiad Approach,Birkhauser Publishers, 2009.

www.VNMATH.com

Page 120: Bất đẳng thức hình học

118

[13] D.S Mitrinovic, J.E.Pecaric and V.Volenec, Recent Advances inGeometric Inequalities, Kluwer Academic publishers, Dordrecht,Netherlands, 1989.

[14] Jian Liu, A weighted geometric inequality and its applications, Jour-nal of inequalities in pure and applied mathematics, Vol.9, Iss.2,Ar.58, 2008.

[15] József Sándor, On the geometry of equilateral triangles, Forum Ge-ometricorum, Vol.5 (2005) 107–117.

www.VNMATH.com